Sie sind auf Seite 1von 38

Airline Pilots Association of the Phil. v.

CIR HELD: As stated in the findings of fact in the questioned resolution of


Director Pura Ferrer-Calleja, on October 23, 1950, a group of deck officers
(LABOR ORGANIZATIONS – UNIONS organized the Cebu Seamen's Association, Inc., (CSAI), a non-stock
:Definitions) corporation and registered it with the Securities and Exchange
Commission (SEC). The same group registered the organization with the
FACTS: A group of pilots of a particular airline, allegedly anticipation their Bureau of Labor Relations (BLR) as Seamen's Association of the
forced retirement or resignation on account of strained relations with the Philippines (SAPI). It is the registration of the organization with the BLR
airline arising from unfulfilled economic demands, decided to adopt an are not with the SEC which made it a legitimate labor organization with
amendment to their organization's constitution and by-laws in order to rights and privileges granted under the Labor Code.
enable them to retain their membership standing therein even after the
termination of their employment with the employer concerned. We gathered from the records that CSAI, the corporation was already
According to the Court of Industrial Relations in its decision regarding the inoperational before the controversy in this case arose. In fact, on August
certification election of the labor organization, the organization's 24, 1984 the SEC ordered the CSAI to show cause why its certificate of
constitution is illegal because member of labor organization cannot adopt registration should not be revoked for continuous inoperation. There is
an amendment to their fundamental charter so as to include non nothing in the records which would show that CSAI answered said show-
employees as member. cause order.

ISSUE: W/N the Constitution is illegal for including non employees of the Also, before the controversy, private respondent Dominica Nacua was
employer airline as member. elected president of the labor union, SAPI. It had an existing CBA with
Aboitiz Shipping Corporation. Before the end of the term of private
HELD: No. Labor organization was defined by Section 2(e) of R.A. 875 respondent Nacua, some members of the union which included Domingo
defines as any union or association of employees which exist, in whole or Machacon and petitioner Manuel Gabayoyo showed signs of
in part, for the purpose of the collective bargaining or dealing with discontentment with the leadership of Nacua. This break-away group
employers concerning terms and conditions of employment." The revived the moribund corporation and issued an undated resolution
absence of the condition which the court below would attach to the expelling Nacua from association (pp. 58-59, Rollo). Sometime in
statutory concept of a labor organization, as being limited to the February, 1987, it held its own election of officers supervised by the
employees of particular employer, is quite evident from the law. Securities and Exchange Commission. It also filed a case of estafa against
Nacua sometime in May, 1986.
Cebu Seaman's Association vs Ferrer-Calleja
The expulsion of Nacua from the corporation, of which she denied being a
(LABOR ORGANIZATIONS – UNIONS
member, has however, not affected her membership with the labor union.
:DOLE registration as Basis)
In fact, in the elections of officers for 1987-1989, she was re-elected as
FACTS: A group of deck officers and marine engineers on board vessels the president of the labor union. In this connections, We cannot agree
plying Cebu and other ports of the Philippines organized themselves into with the contention of Gabayoyo that Nacua was already expelled from
an association and registered the same as a non-stock corporation known the union. Whatever acts their group had done in the corporation do not
as Cebu Seamen's Association, Inc. (CSAI), with the Securities and bind the labor union. Moreover, Gabayoyo cannot claim leadership of the
Exchange Commission (SEC). Later, the same group registered its labor group by virtue of his having been elected as a president of the
dormant corporation CSAI.
association with this Bureau as a labor union known as the Seamen's
Association of the Philippines, Incorporated (SAPI).

SAPI has an existing collective bargaining agreement (CBA) with the Progressive Development Corp vs Sec of DOLE
Aboitiz Shipping Corporation which will expire on 31 December 1988. In
consonance with the CBA said company has been remitting checked-off (LABOR ORGANIZATIONS – UNIONS
union dues to said union until February, 1987 when a group composed of :DOLE Registration as Basis)
members of said union, introducing itself to be its new set of officers,
went to the company and claimed that they are entitled to the remittance FACTS: Pambansang Kilusan ng Paggawa (KILUSAN) -TUCP (hereinafter
and custody of such union dues. This group, headed by Manuel Gabayoyo referred to as Kilusan) filed with the Department of Labor and
claims that they were elected as such on January 20, 1987 under the Employment (DOLE) a petition for certification election among the rank-
supervision of the SEC. and-file employees of the petitioner alleging that it is a legitimate labor
federation and its local chapter, Progressive Development Employees
On 26 May 1987, another group headed by Dominica C. Nacua, claiming Union, was issued charter certificate No. 90-6-1-153. Kilusan claimed that
as the duly elected set of officers of the union in an election held on 20 there was no existing collective bargaining agreement and that no other
December 1986, filed a complaint, for and in behalf of the union, against legitimate labor organization existed in the bargaining unit.
the Cebu Seamen's Association, Inc. (CSAI) as represented by Manuel
Gabayoyo for the security of the aforementioned CBA, seeking such relief, Petitioner PDC filed its motion to dismiss dated July 11, 1990 contending
among others, as an order restraining the respondent from acting on that the local union failed to comply with Rule II Section 3, Book V of the
behalf of the union and directing the Aboitiz Shipping Corp. to remit the Rules Implementing the Labor Code, as amended, which requires the
checked-off union dues for the months of March and April 1987. submission of: (a) the constitution and by-laws; (b) names, addresses and
list of officers and/or members; and (c) books of accounts.
On 10 June 1987, respondent CSAI filed its Answer/Position Paper
alleging that the complainant union and CSAI are one and the same union; On July 16 , 1990, respondent Kilusan submitted a rejoinder to PDC's
that Dominica C. Nacua and Atty. Prospero Paradilla who represented the motion to dismiss claiming that it had submitted the necessary
union had been expelled as members/officers as of November 1984 for documentary requirements for registration, such as the constitution and
lawful causes; and, that its set of officers headed by Manuel Gabayoyo has by-laws of the local union, and the list of officers/members with their
the lawful right to the remittance and custody of the corporate funds addresses. Kilusan further averred that no books of accounts could be
(otherwise known as union does) in question pursuant to the resolution submitted as the local union was only recently organized.
of the SEC dated 22 April 1987.
In its "Supplemental Position Paper" dated September 3, 1990, the
The controversy in this case is between the two sets of officers in the petitioner insisted that upon verification with the Bureau of Labor
union. Both sets of officers claim to be entitled to the release of the union Relations (BLR), it found that the alleged minutes of the organizational
dues collected by the company with whom it had an existing CBA. The meeting was unauthenticated, the list of members did not bear the
controversy involves claims of different members/officers to certain corresponding signatures of the purported members, and the constitution
rights granted under the labor code. and by-laws did not bear the signature of the members and was not duly
subscribed. It argued that the private respondent therefore failed to
ISSUE: Who is entitled to the collection and custody of the union dues? substantially comply with the registration requirements provided by the
rules. Additionally, it prayed that Med-Arbiter Edgardo dela Cruz inhibit
himself from handling the case for the reason that he allegedly had being one, an organization which carries a mixture of rank-and-file and
prejudged the same. supervisory employees cannot possess any of the rights of a legitimate
labor organization, including the right to file a petition for certification
In his September 5, 1990 resolution, Med Arbiter dela Cruz held that election for the purpose of collective bargaining. It becomes necessary,
there was substantial compliance with the requirements for the therefore, anterior to the granting of an order allowing a certification
formation of the chapter. He further stated that mere issuance of the election, to inquire into the composition of any labor organization
charter certificate by the federation was sufficient compliance with the whenever the status of the labor organization is challenged on the basis
rules. Considering that the establishment was unorganized, he of Article 245 of the Labor Code.
maintained that a certification election should be conducted to resolve
the question of representation.

ISSUE: W/N the Kilusan is a legitimate labor organization. PROGRESSIVE DEVELOPMENT CORPORATION, petitioner, v. THE
HONORABLE SECRETARY, DEPARTMENT OF LABOR AND
HELD: A local or chapter therefore becomes a legitimate labor EMPLOYMENT, MED-ARBITER EDGARDO DELA CRUZ, AND
organization only upon submission of the following to the BLR: PAMBANSANG KILUSAN NG PAGGAWA (KILUSAN)-TUCP,
respondents.
1) A charter certificate, within 30 days from its issuance by the
labor federation or national union, and (LABOR ORGANIZATIONS – UNIONS
: DOLE Registration as Basis)
2) The constitution and by-laws, a statement on the set of officers,
and the books of accounts all of which are certified under oath by the FACTS:
secretary or treasurer, as the case may be, of such local or chapter, and
attested to by its president.

Absent compliance with these mandatory requirements, the local or Pambansang Kilusan ng Paggawa –TUCP (KILUSAN) filed with the DOLE a
chapter does not become a legitimate labor organization. petition for certification election among the rank-and-file employees of
Progressive Development Corporation (PDC), alleging that: (1) it is a
In the case at bar, the failure of the secretary of PDEU-Kilusan to certify legitimate labor federation; (2) its local chapter, Progressive
the required documents under oath is fatal to its acquisition of a Development Employees Union, was issued charter certificate; (3) there
legitimate status. was no existing collective bargaining agreement; and (4) no other
legitimate labor organization existed in the bargaining unit.

Toyota Motor Phils. Corp v. Tyota Motors Phils Corp Labor Union
PDC filed its motion to dismiss contending that KILUSAN failed to comply
(LABOR ORGANIZATIONS – UNIONS with Rule II, Section 3, Book V of the Rules Implementing the Labor Code,
:Mixed Membership) as amended, which requires the submission of: (a) the constitution and
by-laws; (b) names, addresses and list of officers and/or members; and
FACTS: The Toyota Motor Philippines Corporation Labor Union
(c) books of accounts. KILUSAN submitted a rejoinder claiming that it had
(TMPCLU) filed a petition for certification election with the Department
submitted the necessary documentary requirements for registration and
of Labor, National Capital Region, for all rank-and-file employees of the
that since the local union was recently organized, no books of accounts
Toyota Motor Corporation.
could be submitted.
In response, petitioner filed a Position Paper on seeking the denial of the
Med-Arbiter held that there was substantial compliance with the
issuance of an Order directing the holding of a certification election on
requirements and the mere issuance of the charter certificate by the
two grounds: first, that the respondent union, being "in the process of
federation was sufficient compliance with the rules. PDC’s MR was
registration" had no legal personality to file the same as it was not a
treated as an appeal, as such, the Secretary of Labor took cognizance
legitimate labor organization as of the date of the filing of the petition;
thereof and denied PDC’s MR. Thereafter, TRO was issued enjoining the
and second, that the union was composed of both rank-and-file and
public respondents from carrying out the resolutions and orders or from
supervisory employees in violation of law.
proceeding with the certification election.
ISSUE: W/N there can be a legitimate labor organization with mixed
membership (rank and file and supervisory employees).
ISSUE:
HELD: No. An appropriate bargaining unit is a group of employees of a
given employer, composed of all or less than the entire body of Whether PDC failed to substantially comply with the requirements
employees, which the collective interests of all the employees, consistent prescribed under Rule II, Section 3, Book V of the Rules Implementing the
with equity to the employer indicate to be best suited to serve reciprocal Labor Code, as amended.
rights and duties of the parties under the collective bargaining provisions
of law. In Belyca Corporation v. Ferrer Calleja, 18 we defined the
bargaining unit as "the legal collectivity for collective bargaining
purposes whose members have substantially mutual bargaining interests RULING:
in terms and conditions of employment as will assure to all employees
their collective bargaining rights." This in mind, the Labor Code has made The Petition is GRANTED and the resolution and orders of the Med-
it a clear statutory policy to prevent supervisory employees from joining Arbiter and Secretary of Labor and Employment are set aside. The TRO is
labor organizations consisting of rank-and-file employees as the concerns made permanent.
which involve members of either group are normally disparate and
contradictory. Article 245 provides:

Art. 245 Ineligibility of managerial employees to join any labor YES.


organization; right of supervisory employees. — Managerial Employees
A labor organization acquires legitimacy only upon registration with the
are not eligible to join, assist or form any labor organization. Supervisory
BLR. Article 234 of the Labor Code prescribes the requirements for
employees shall not be eligible for membership in a labor organization of
registration. Prescribing those requirements does not limit the right of
the rank-and-file employees but may join, assist or form separate labor
organizations of their own. assembly or association but rather a condition sine qua non for the
acquisition of legal personality by labor organizations, associations or
Clearly, based on this provision, a labor organization composed of both unions, and the possession of the rights and privileges granted by law to
rank-and-file and supervisory employees is no labor organization at all. It legitimate labor organizations. Besides, registration, as a valid exercise of
cannot, for any guise or purpose, be a legitimate labor organization. Not police power, is required to protect both labor and the public against
abuses, fraud, or impostors who pose as organizers, although not truly sharing bonus within a period of sixty (60) days from the date of receipt
accredited agents of the union the purport to represent. by [it] from the [Company] of the profit-sharing bonus. If a laborer or
employee of the [Company] does not want to accept the profit-sharing
Further, when an unregistered union becomes a branch, local or chapter bonus which the said employee or laborer is entitled under this
of a federation, some of the aforementioned requirements for registration Agreement, it shall be the duty of the [Associated Labor Union] to return
are no longer required. A local or chapter need not be independently the money received by [it] as profit-sharing bonus to the [Company]
registered. A local or chapter becomes a legitimate labor organization within a period of sixty (60) days from the receipt by the [Union] from
upon compliance with the provisions of Section 3, Rule II of Book V of the the [Company] of the said profit-sharing bonus.'"
Implementing Rules, i.e., submission of the following to the BLR: 1) A
charter certificate, within 30 days from its issuance by the labor In March, 1965 the defendant Cebu Shipyard & Engineering Works, Inc.
federation or national union, and 2) The constitution and by-laws, a delivered to the ALU for distribution to the laborers or employees
statement on the set of officers, and the books of accounts all of which are working with the defendant corporation to the profit-sharing bonus
certified under oath by the secretary or treasurer, as the case may be, of corresponding to the first installment for the year 1965. Again in June
such local or chapter, and attested to by its president. Absent compliance 1965 the defendant corporation delivered to the Associated Labor Union
with these mandatory requirements, the local or chapter does not the profit-sharing bonus corresponding to the second installment for
become a legitimate labor organization. 1965. The members of the Mactan Workers Union failed to receive their
shares in the second installment of bonus because they did not like to go
Furthermore, in case of union registration, the rationale for requiring that to the office of the ALU to collect their shares. In accordance with the
the submitted documents and papers be certified under oath by the terms of the collective bargaining after 60 days, the uncollected shares of
secretary or treasurer, as the case may be, and attested to by the the plaintiff union members was returned by the ALU to the defendant
president is that such submission becomes the BLR’s basis for the corporation. At the same time the defendant corporation was advised by
approval of the application for registration. Also, the certification and the ALU not to deliver the said amount to the members of the Mactan
attestation requirements are preventive measures against the Workers Union unless ordered by the Court, otherwise the ALU will take
commission of fraud. such step to protect the interest of its members… Because this warning
given by the intervenor union the defendant corporation did not pay to
It may be noted as well that the mother union, acting for and in behalf of the plaintiffs the sum of P4,035.82 which was returned by the Associated
its affiliate, had the status of an agent while the local union remained the Labor Union, but instead, deposited the said amount with the Labor
basic unit of the association, free to serve the common interest of all its Administrator.
members, subject only to the restraints imposed by the constitution and
by-laws of the association. Thus, where as in this case the petition for Issue:
certification election was filed by the federation which is merely an agent;
the petition is deemed to be filed by the chapter, the principal, which WON the benefits of a collective bargaining agreement extend to the
must be a legitimate labor organization. laborers and employees in the collective bargaining unit who do not
belong to the labor organization?
In this case, KILUSAN failed to substantially comply with the
aforementioned statutory requirements. The SC noted that big Held:
federations and national unions of workers should take the lead in
requiring their locals and chapters to faithfully comply with the law and Yes. It extends also to them.
the rules instead of merely snapping union after union into their folds in a
furious bid with rival federations to get the most number of members. The labor union that gets the majority vote as the exclusive bargaining
representative does not act for its members alone. It represents all the
employees in such a bargaining unit. It is not to be indulged in any
attempt on its part to disregard the rights of non-members.
MACTAN WORKERS UNION and TOMAS FERRER, as President
thereof, plaintiffs-appellees, The right to be the exclusive representative of all the employees in an
appropriate collective bargaining unit is vested in the labor union
vs. 'designated or selected' for such purpose 'by the majority of the
employees' in the unit concerned.
DON RAMON ABOITIZ, President, Cebu Shipyard & Engineering
Works, Inc.; EDDIE LIM, as Treasurer; JESUS DIAGO, Superintendent Guirano v. CIR
of the aforesaid corporation; WILFREDO VIRAY, as Resident
Manager of the Shipyard & Engineering Works, Inc.; and the CEBU (LABOR ORGANIZATIONS – UNIONS
SHIPYARD & ENGINEERING WORKS, INC., defendants-appellees; : Union Rationale)
ASSOCIATION LABOR UNION, intervenor-appellant.

(LABOR ORGANIZATIONS – UNIONS


: Union Rationale) Facts:

Facts: Three unfair labor practice cases for unlawful dismissal allegedly based
on legitimate union activity were filed against respondent Central Santos
The defendant Cebu Shipyard & Engineering Works, Inc. in Lapulapu City Lopez Co., Inc. and respondent
is employing laborers and employees belonging to two rival labor unions.
Seventy-two of these employees or laborers whose names appear in the United Sugar Workers Union-ILO. In view of a closed-shop provision in
complaint are affiliated with the Mactan Workers Union while the rest are the then existing collective bargaining contract, respondent Central
members of the intervenor Associated Labor Union. On November 28, Santos Lopez Co., Inc. assumed it had to dismiss them. So it was noted in
1964, the defendant Cebu Shipyard & Engineering Works, Inc. and the the decision of the then associate Judge Joaquin M. Salvador of
Associated Labor Union entered into a 'Collective Bargaining Agreement' respondent Court. Thus: &quote; The respondent company, in its answer,
... the pertinent part of which, Article XIII thereof, [reads thus]: alleged that the only reason for the dismissal of the complainants herein
is because their said dismissal was asked by the USWU-ILO of which
'... The [Company] agrees to give a profit-sharing bonus to its employees union respondent company has a valid and existing collective bargaining
and laborers to be taken from ten per cent (10%) of its net profits or net contract with a closed-shop provision to the effect that those laborers
income. who are no longer members of good standing in the union may be
dismissed by the respondent company if their dismissal is sought by the
Said profit-sharing bonus shall be paid by the [Company] to [Associated union; that respondent company has never committed acts of unfair labor
Labor Union] to be delivered by the latter to the employees and laborers practice against its employees or workers much less against the
concerned and it shall be the duty of the Associated Labor Union to complainants herein but that it has a solemn obligation to comply with
furnish and deliver to the [Company] the corresponding receipts duly the terms and conditions of the contract; and that a closed-shop
signed by the laborers and employees entitled to receive the profit- agreement is sanctioned under this jurisdiction for such kind of
agreement is expressly allowed under the provisions of Republic Act 875 representatives of respondent firm, including their counsels of record and
known as the Industrial Peace Act and the dismissal of complainants is the President of the union and 8 directors of the union. Four of these nine
merely an exercise of a right allowed by said law.&quote; There was no union representatives, including the union president himself, had no
question, however, as to petitioners having been employed by such claims or awards whatever under the judgment. Said union officials were
respondent Company long before the collective bargaining contract. not assisted by counsel, as petitioner Mary Concepcion, counsel of record
of the union, was not present, not having been notified of the conference.
In the decision of respondent Court, there was an acknowledgment of the
prior existence of such employment relationship. Nonetheless, the In this conference respondent firm made again the same offer to settle
conclusion reached, both by the trial judge and then by respondent and quitclaim the judgment in favor of the union members for the same
Court en banc was that the dismissal was justifiable under the closed- amount of P110,000.00, which offer had already been ‘rejected by the
shop provision of the collective bargaining agreement. union at the earlier conference held on June 25, 1963. But this time,
respondent and the directors of the union decided to settle the case
Issue: amicably with the payment by the firm of the same amount of
P110,000.00 which was deposited with the Court’s disbursing officer
“immediately upon the signing of the settlement which will be prepared
by the respondent firm through its counsel.”
WON the closed-shop provision of the union is compelling and will not
apply to its dismissed members; and whether the dismissal (by the One of the union director together with 49 of its members questioned the
company) is justifiable. amicable settlement that took place. They claim that the Board of
Directors did not have any express authority of the members of the
Held:
Santiago Labor Union to enter into any compromise for the sum of
The dismissal is unjustifiable and the employees should be reinstated. P110,000.00, that it was tainted by apparent bad faith on the part of the
The authoritative doctrine that a closed-shop provision in a collective President of the Union, that the amount of P110,000.00 is
bargaining agreement is not to be given a retroactive effect so as to unconscionable, considering, that the total claims of the members of the
preclude its being applied to employees already in the service. The union is more than P400,000.00.
closed-shop agreement authorized under sec. 4, subsec. a(4) of the
Industrial Peace Act above quoted should however, apply to persons to be
hired or to employees who are not yet members of any labor ISSUE:
organization. It is inapplicable to those already in the service who are
members of another union. To hold otherwise, i. e., that the employees in
a company who are members of a minority union may be compelled to
disaffiliate from their union and join the majority or contracting union, Whether the amicable settlement is valid.
would render nugatory the right of all employees to self- organization
and to form, join or assist labor organizations of their own choosing, a
right guaranteed by the Industrial Peace Act (sec. 3, Rep. Act No. 875) as
well as by the Constitution (Art. III, sec. 1[6]). It is well settled that such RULING:
unions are not entitled to arbitrarily exclude qualified applicants for
NO. Petitioners were not accorded due process of law. The union was
membership, and a closed-shop provision would not justify the employer
deprived of the assistance of its counsel. The lack or due deliberation and
in discharging, or a union in insisting upon the discharge of, an employee
caution in the trial judge’s instant approval of the settlement is seen from
whom the union thus refuses to admit to membership, without any
the stipulations therein that the union thereby waived and quitclaimed
reasonable ground therefore.
any and all claims which it may have against the respondent, as well as
the claim of each and every one of the members of the union against
respondent, when precisely the authority of the union board members to
enter into any such compromise or settlement was under express
Heirs of Teodulo M. Cruz v. CIR challenge by petitioner Magalpo, a board member herself which the trial
judge completely disregarded.
(LABOR ORGANIZATIONS – UNIONS
: Union Rationale) Petitioners were deprived of the formal conference on the and of their
right to be assisted by the union counsel as expressly requested, so that a
fair hearing could be accorded petitioners and an opportunity afforded
them to air their serious charges of bad faith and lack authority against
FACTS: the Union leadership. Certainly, all these serious questions and charges
made by petitioners could have been threshed out and verified, if the
June 21, 1952, the Santiago Labor Union, composed of workers of the formal conference had been held with the presence of union counsel.
Santiago Rice Mill, a business enterprises engaged in the, buying. and
milling of palay at Santiago, Isabela, and owned and operated by King
Hong Co., Inc., filed before the respondent Court of Industrial Relations
(CIR) cases for petition for overtime pay, premium pay for night, Sunday The transcript of the conference is deficient and does not reflect the
and holiday work, and for reinstatement of workers illegally laid off. CIR actual discussions and proceedings. This is to be deplored, for in a matter
favored the union by a split decision of 3-2 vote. The case reached the of such great importance, especially where the union officials were
Supreme Court but the SC still favored the union. The SC remanded the unassisted by counsel in an unscheduled conference, care should be taken
records for enforcement by respondent CIR. In examination of books, said by the trial judge that the proceedings are faithfully recorded.
CIR’s Chief Examiner filed his Partial Report regarding the computation of
the benefits rendered in the case in favor of the Union. We find the forcing through of the settlement arbitrary, unfair and
unconscionable.
Petitioners claim that in this computation of the Examiner did not,
include the claims of 70 other laborers whose total claims (for back Another reason for striking down the settlement is the lack of any express
wages), at the rate of P6,300.00 each and would be P441,000.00. or specific authority of the president and majority. of the union board of
Therefore, the correct. grand total amount due the laborers would be directors to execute the same and scale down the estimated P423,756.74
864,756.74. judgment liability of respondent firm in favor of the individual union
members to P110,000.00. On the contrary, petitioner board member
The trial judge took no action on the latest Urgent Motion of the union, Magalpo timely challenged the authority or the union board to execute
wherein it emphasized that respondent, with Court’s action rejecting its any such settlement, expressly informing the trial judge that the union
appeal, no longer had any excuse for refusing to comply with the deposit had specifically appointed an entity in Manila, the “CREAM, Inc.,” as its
Order. Instead, an unscheduled conference was called and held on attorney in-fact and “exclusive authorized representative for the
October 31, 1963 in the chambers of the trial judge, and attended by evaluation, adjustment and -liquidation or its claim against respondent.
These union members have repudiated the former union president, (LABOR ORGANIZATIONS – UNIONS
Maylem and his board of directors, for having betrayed the union : Union Rationale)
members, and the new union leadership.

The authority of the union, to execute a settlement of the judgment award


in favor of the individual union members, cannot be presumed but must
be expressly granted.
FACTS: On August 4, 1959, On August 4, 1959, USUP presented a set of
Just as this Court has stricken down unjust exploitation of laborers by demand to USUP presented a set of demand to DSA, representing the
oppressive employers, so will it strike down their unfair treatment by respondent shipping companies, for union DSA, representing the
their own unworthy leaders. The Constitution enjoins the State to afford respondent shipping companies, for union recognition, union security,
protection to labor. Fair dealing is equally demanded of unions as well as standardization of wages and other benefits. In response, the ship owners
of employers in their dealings with employees. The union has been brought to the recognition, union security, standardization of wages and
evolved as an organization of collective strength for the protection of other benefits. In response, the ship owners brought to the attention of
labor against the unjust exactions of capital, but equally important is the the USUP the existence of a CBA with the Davao Marine Association
requirement of fair dealing between the union and its members, which is (DMA), where all of the crewmen of attention of the USUP the existence of
fiduciary in nature, and arises out of two factors: “one is the degree of a CBA with the Davao Marine Association (DMA), where all of the
dependence of the individual employee on the union organization; the crewmen of their launches belonged. They suggested to the USUP that
other, a corollary of the first, is the comprehensive power vested in the they first take the necessary steps for certification as their launches
union with respect to the individual.” The union may, be considered but belonged. They suggested to the USUP that they first take the necessary
the agent of its members for the purpose of securing for them fair and steps for certification as the collective bargaining agent, as the ship
just wages and good working conditions and is subject to the obligation of owners were bound by the CBA until 1959. However, even before
giving the members as its principals all information relevant to union and receiving the collective bargaining agent, as the ship owners were bound
labor matters entrusted to it. by the CBA until 1959. However, even before receiving the ship owner’s
response to its demands, the USUP had filed a Notice of Strike against the
The union leadership in the case at bar was recreant in its duty towards individual ship owners at the ship owner’s response to its demands, the
the union members in apparently having failed to disclose to the union USUP had filed a Notice of Strike against the individual ship owners at the
members the full situation of their judgment credit against respondent, to Department of Labor Davao Department of Labor Davao Office.
wit, that they were in the advantageous position of being able to require
enforcement of the respondent court’s P200,000.00-deposit order, and in The Chief of the Labor Operations Section of the Davao the Chief of the
presuming that it had authority to waive and quitclaim the estimated Labor Operations Section of the Davao Office requested for a conference
P423,756.74-judgment credit of the union members for the to solve the conflict. On August, Office requested for a conference to solve
unconscionable amount of P110,000.00, which had already been the conflict. On August20, 1959, both parties reached a covenant stating
previously rejected by the workers. Respondent firm could not claim that the withdrawal of the Notice of Strike, as well as the observance of the20,
it dealt in good faith with the union officials, for it hastily executed the 1959, both parties reached a covenant stating the withdrawal of the
purported settlement notwithstanding the serious charges of bad faith Notice of Strike, as well as the observance of the status quo regarding the
against the union leadership, and the non-holding of the scheduled jobs incident to the businesses of the DSA and the withdrawal of the civil
conference where the union leaders, at their express request, could be case of the DMA status quo regarding the jobs incident to the businesses
duly assisted by union counsel. The interests of the individual worker can of the DSA and the withdrawal of the civil case of the DMA against the
be better protected on the whole by a strong union aware of its moral and USUP. It was also stipulated in the contract that the USUP will respect the
legal obligations to represent the rank and file faithfully and secure for existing CBA between DSA and against the USUP. It was also stipulated in
them the best wages and working terms and conditions in the process of the contract that the USUP will respect the existing CBA between DSA and
collective bargaining. As has been aptly pointed out, the will of the DMA, but USUP will DMA, but USUP will file a petition for certification
majority must prevail over that of the minority in the process, for “under election for file a petition for certification election for determination of
the philosophy of collective responsibility, an employer who bargains in union representation. Determination of union representation. As
good faith should be entitled to rely upon the promises and agreements of stipulated, the USUP filed with the CIR a petition for certification election.
the union representatives with whom he must deal. under the Subsequent to the covenant, the shipping
compulsion of, law and contract. The ‘collective bargaining process
should be carried on between parties who can mutually respect and rely As stipulated, the USUP filed with the CIR a petition for certification
upon the authority of each other.” Where, however, collective bargaining election. Subsequent to the covenant, the shipping companies separately
process is not involved, and what is at stake are back wages already served notices of termination upon 64 employees, effecting December 31,
earned by the individual workers by way of overtime, premium and 1959, due to different companies separately served notices of
differential pay, and final judgment has been rendered in their favor, as in termination upon 64 employees, effecting December 31, 1959, due to
the present case, the real parties in interest with direct material interest, different reasons (from stoppage of operations to the death of one of the
as against the union which has only served as a vehicle for collective partners of the shipping companies due to business reasons (from
action to enforce their just claims, are the individual workers themselves. stoppage of operations to the death of one of the partners of the shipping
Authority of the union to waive or quitclaim all or part of the judgment companies due to business losses). As a result, USUP reported the
award in favor of the individual workers cannot be lightly presumed but terminations to the Department of Labor, which called for a conference.
must be expressly granted, and the employer, as judgment debtor, must Losses). As a result, USUP reported the terminations to the Department of
deal in all good faith with the union as the agent of the individual Labor, which called for a conference Nevertheless, on ss, on December 29,
workers. The Court in turn should certainly verify and assure itself of the 1959, the December 29, 1959, the USUP notified the Philippine
fact and extent of the authority of the union leadership to execute any Constabulary, City Mayor, Bureau of Customs USUP notified the
compromise or settlement of the judgment on behalf of the individual Philippine Constabulary, City Mayor, Bureau of Customs the general
workers who are the real judgment creditors. public of a strike on January 1, 1960.the general public of a strike on
January 1, 1960.
The settlement in the case at bar was precipitately approved without
verification of the union boards authority to execute the compromise On February 11, 1960, the shipping companies filed a petition for writ of
settlement and that there was no such authority. injunction, as a necessity due to irreparable On February 11, 1960, the
shipping companies filed a petition for writ of injunction, as a necessity
WHEREFORE, the respondent Court’s Orders are hereby declared null due to irreparable damage to properties due to “coercion, violence and
and void and set aside. illegal picketing”. On the other hand, on February 24, 1960, the damage to
properties due to “coercion, violence and illegal picketing”. On the other
hand, on February 24, 1960, the USUP filed a ULP case against the ship
owners and DSA, alleging that the ship owners interfered, and continued
United Seaman’s Union of the Phil. V. Davao Ship-owners Assn., 20 to USUP filed a ULP case against the ship owners and DSA, alleging that
SCRA 1226 the ship owners interfered, and continued to interfere with their right to
self-organization by discrimination against employees. CIR however
sided with the DSA Interfere with their right to self-organization by public respondents from carrying out the resolutions and orders or from
discrimination against employees. CIR however sided with the DSA proceeding with the certification election.
Dismissing the USUP’s ULP case while declaring the dismissing the
USUP’s ULP case while declaring the strike as illegal. Strike as illegal.

ISSUE:

ISSUE: WON CIR gravely abused discretion by declaring the strike illegal? Whether PDC failed to substantially comply with the requirements
prescribed under Rule II, Section 3, Book V of the Rules Implementing the
Labor Code, as amended.

HELD: The Supreme Court agreed with the lower court’s findings that the
USUP’s Notice of Strike was but a direct offshoot of the “losing effort” to
compel the DSA and the ship owners to recognize USUP as the sole RULING:
collective bargaining agent of the employees, to the exclusion of the DMA
as the existing collective bargaining agent of the DSA The Petition is GRANTED and the resolution and orders of the Med-
Arbiter and Secretary of Labor and Employment are set aside. The TRO is
First, the Court noted that the USUP filed its Notice of Strike even before made permanent.
its receipt of the ship owner’s answers to its set of demands, thereby
showing that the USUP was already set on continuing the strike with or
without the answer of the ship owners. This, according to the Court was a
YES.
clear showing that USUP was aware of the existence of DMA as a valid
collective bargaining agent, operating as a legal bar to entertaining A labor organization acquires legitimacy only upon registration with the
USUP’s demands. BLR. Article 234 of the Labor Code prescribes the requirements for
Second, the Court stated that USUP completely disturbed the status quo registration. Prescribing those requirements does not limit the right of
assembly or association but rather a condition sine qua non for the
– the return to normal and original operating practices through acquisition of legal personality by labor organizations, associations or
the strike that was done by USUP. By striking, USUP impaired unions, and the possession of the rights and privileges granted by law to
existing CBA between the ship owners and the DMA which legitimate labor organizations. Besides, registration, as a valid exercise of
recognized "the right of the Employer to hire, promote and police power, is required to protect both labor and the public against
transfer and for legal cause suspend, lay-off or discharge abuses, fraud, or impostors who pose as organizers, although not truly
employees subject to the right of the union (referring to the accredited agents of the union the purport to represent.
DMA) to notification and to ask reconsideration of any action
Further, when an unregistered union becomes a branch, local or chapter
of the Employer in the premises."
of a federation, some of the aforementioned requirements for registration
are no longer required. A local or chapter need not be independently
registered. A local or chapter becomes a legitimate labor organization
upon compliance with the provisions of Section 3, Rule II of Book V of the
Implementing Rules, i.e., submission of the following to the BLR: 1) A
PROGRESSIVE DEVELOPMENT CORPORATION, petitioner, v. THE charter certificate, within 30 days from its issuance by the labor
HONORABLE SECRETARY, DEPARTMENT OF LABOR AND federation or national union, and 2) The constitution and by-laws, a
EMPLOYMENT, MED-ARBITER EDGARDO DELA CRUZ, AND statement on the set of officers, and the books of accounts all of which are
PAMBANSANG KILUSAN NG PAGGAWA (KILUSAN)-TUCP, certified under oath by the secretary or treasurer, as the case may be, of
respondents. such local or chapter, and attested to by its president. Absent compliance
with these mandatory requirements, the local or chapter does not
(LABOR UNION AND GOVERNMENT REGULATION: Requirements and become a legitimate labor organization.
Rationale)
Furthermore, in case of union registration, the rationale for requiring that
the submitted documents and papers be certified under oath by the
secretary or treasurer, as the case may be, and attested to by the
FACTS: president is that such submission becomes the BLR’s basis for the
approval of the application for registration. Also, the certification and
attestation requirements are preventive measures against the
Pambansang Kilusan ng Paggawa –TUCP (KILUSAN) filed with the DOLE a commission of fraud.
petition for certification election among the rank-and-file employees of
It may be noted as well that the mother union, acting for and in behalf of
Progressive Development Corporation (PDC), alleging that: (1) it is a
its affiliate, had the status of an agent while the local union remained the
legitimate labor federation; (2) its local chapter, Progressive
basic unit of the association, free to serve the common interest of all its
Development Employees Union, was issued charter certificate; (3) there
members, subject only to the restraints imposed by the constitution and
was no existing collective bargaining agreement; and (4) no other
by-laws of the association. Thus, where as in this case the petition for
legitimate labor organization existed in the bargaining unit.
certification election was filed by the federation which is merely an agent;
the petition is deemed to be filed by the chapter, the principal, which
must be a legitimate labor organization.
PDC filed its motion to dismiss contending that KILUSAN failed to comply
with Rule II, Section 3, Book V of the Rules Implementing the Labor Code, In this case, KILUSAN failed to substantially comply with the
as amended, which requires the submission of: (a) the constitution and aforementioned statutory requirements. The SC noted that big
by-laws; (b) names, addresses and list of officers and/or members; and federations and national unions of workers should take the lead in
(c) books of accounts. KILUSAN submitted a rejoinder claiming that it had requiring their locals and chapters to faithfully comply with the law and
submitted the necessary documentary requirements for registration and the rules instead of merely snapping union after union into their folds in a
that since the local union was recently organized, no books of accounts furious bid with rival federations to get the most number of members.
could be submitted.

Med-Arbiter held that there was substantial compliance with the


Phoenix Iron and Steel Corp. v. Sec. of Labor, 244 SCRA 173 [G.R. No.
requirements and the mere issuance of the charter certificate by the 112141, May 16, 1995]
federation was sufficient compliance with the rules. PDC’s MR was
treated as an appeal, as such, the Secretary of Labor took cognizance (LABOR UNION AND GOVERNMENT REGULATION: Requirements and
thereof and denied PDC’s MR. Thereafter, TRO was issued enjoining the Rationale)
Facts o In the case at bar, the failure of the secretary of PDEU-Kilusan
to certify the required documents under oath is fatal to its acquisition of a
o Private respondent PISCOR Workers Union—Alliance of legitimate status.
Nationalist and Genuine Labor Organizations (PISCOR- ANGLO) asserting
to be a legitimate labor organization filed on 13 October 1992 a petition o Compared with what happened in the Progressive case, this
for certification election with the Med-Arbiter. situation before us now is even worse.

o On 3 December 1992 petitioner Phoenix Iron and Steel o There are no books of account filed before the BLR, the
Corporation (PHOENIX) sought clarification of the legal personality of constitution, by- laws and the list of members who supposedly ratified
PISCOR-ANGLO (UNION). On 19 January 1993, finding that the UNION the same were not attested to by the union president, and the
had not complied with the requisites of law, Med-Arbiter Napoleon V. constitution and by-laws were not verified under oath.
Fernando dismissed the petition.

- Issue
SAN MIGUEL UNION VS. LAGUESMA
o Is the failure of the secretary of PDEU-Kilusan to certify the
required documents under oath fatal to its acquisition of legitimate (LABOR UNION AND GOVERNMENT REGULATION: Requirements and
status? Rationale)

- Held

o We agree with petitioner Phoenix Iron and Steel Corporation FACTS: Petitioner union filed before DOLE a Petition for Direct
and the Solicitor General that our ruling that Progressive Development Certification or Certification Election among the supervisors and exempt
Corp. v. Sec. of Labor, 205 SCRA 802 [G.R. No. 96425, Feb. 4, 1992] applies employees of the SMC Magnolia Poultry Products Plants of Cabuyao, San
in this case, to wit: Fernando and Otis.

A local or chapter x x x becomes a legitimate labor organization Med-Arbiter Danilo L. Reynante issued an Order ordering the conduct of
only upon submission of the following to the BLR: certification election among the abovementioned employees of the
different plants as one bargaining unit.
• 1) A charter certificate, within 30 days from its issuance by the
labor federation or national union, and San Miguel Corporation filed a Notice of Appeal with Memorandum on
Appeal, pointing out, among others, the Med-Arbiter’s error in grouping
• 2) The constitution and by-laws, a statement on the set of together all three (3) separate plants, into one bargaining unit, and in
officers, and the books of accounts all of which are certified under oath by including supervisory levels 3 and above whose positions are confidential
the secretary or treasurer, as the case may be, of such local or chapter, in nature.
and attested to by its president.
The public respondent, Undersecretary Laguesma, granted respondent
Absent compliance with these mandatory requirements, the company’s Appeal and ordered the remand of the case to the Med-Arbiter
local or chapter does not become a legitimate labor organization. of origin for determination of the true classification of each of the
employees sought to be included in the appropriate bargaining unit.
o In the case at bar, the failure of the secretary of PDEU-Kilusan
to certify the required documents under oath is fatal to its acquisition of a Upon petitioner-union’s motion, Undersecretary Laguesma granted the
legitimate status. reconsideration prayed for and directed the conduct of separate
certification elections among the supervisors ranked as supervisory
o Compared with what happened in the Progressive case, this levels 1 to 4 (S1 to S4) and the exempt employees in each of the three
situation before us now is even worse. plants at Cabuyao, San Fernando and Otis.

o There are no books of account filed before the BLR, the


constitution, by- laws and the list of members who supposedly ratified
the same were not attested to by the union president, and the ISSUE:
constitution and by-laws were not verified under oath.

Issue
1. Whether Supervisory employees 3 and 4 and the exempt
o Is the failure of the secretary of PDEU-Kilusan to certify the employees of the company are considered confidential employees, hence
required documents under oath fatal to its acquisition of legitimate ineligible from joining a union.
status?

Held
2. If they are not confidential employees, do the employees of the
o We agree with petitioner Phoenix Iron and Steel Corporation three plants constitute an appropriate single bargaining unit.
and the Solicitor General that our ruling that Progressive Development
Corp. v. Sec. of Labor, 205 SCRA 802 [G.R. No. 96425, Feb. 4, 1992] applies
in this case, to wit:
RULING:
A local or chapter x x x becomes a legitimate labor organization
only upon submission of the following to the BLR: (1) On the first issue, this Court rules that said employees do not fall
within the term “confidential employees” who may be prohibited from
• 1) A charter certificate, within 30 days from its issuance by the joining a union.
labor federation or national union, and
They are not qualified to be classified as managerial employees who,
• 2) The constitution and by-laws, a statement on the set of under Article 245 of the Labor Code, are not eligible to join, assist or form
officers, and the books of accounts all of which are certified under oath by any labor organization. In the very same provision, they are not allowed
the secretary or treasurer, as the case may be, of such local or chapter, membership in a labor organization of the rank-and-file employees but
and attested to by its president. may join, assist or form separate labor organizations of their own.

Absent compliance with these mandatory requirements, the


local or chapter does not become a legitimate labor organization.
Confidential employees are those who (1) assist or act in a confidential
capacity, (2) to persons who formulate, determine, and effectuate
management policies in the field of labor relations. The two criteria are Held: No. As long as an applicant union complies with all of the legal
cumulative, and both must be met if an employee is to be considered a requirements for registration, it becomes the BLR’s ministerial duty to so
confidential employee — that is, the confidential relationship must exist register the union. It suffices then to order that petitioner Union be
between the employee and his supervisor, and the supervisor must registered, there being no legal obstacle to such a step and the duty of the
handle the prescribed responsibilities relating to labor relations. Bureau of Labor Relations being clear. Then there is this ruling in
Philippine Labor Alliance Council v. Bureau of Labor Relations that calls
The exclusion from bargaining units of employees who, in the normal for application that “once the fact of disaffiliation has been demonstrated
course of their duties, become aware of management policies relating to beyond doubt, as in this case, a certification election is the most
labor relations is a principal objective sought to be accomplished by the expeditious way of determining which labor organization is to be the
”confidential employee rule.” The broad rationale behind this rule is that exclusive bargaining representative.” In the meanwhile, if as contended
employees should not be placed in a position involving a potential conflict by private respondent labor union the interim collective bargaining
of interests. “Management should not be required to handle labor agreement which was entered during the pendency of the petition of the
relations matters through employees who are represented by the union petitioner, has much more favorable terms for the workers of private
with which the company is required to deal and who in the normal respondent Vassar Industries, then it should continue in full force and
performance of their duties may obtain advance information of the effect until the appropriate bargaining representative is chosen and
company’s position with regard to contract negotiations, the disposition negotiations for a new collective bargaining agreement thereafter
of grievances, or other labor relations matters.” concluded. This is one way of assuring that both the social justice, and the
protection to labor provisions would be effectively implemented without
The Court held that “if these managerial employees would belong to or be sanctioning an attempt to frustrate the exercise of this Court’s jurisdiction
affiliated with a Union, the latter might not be assured of their loyalty to in a pending case.
the Union in view of evident conflict of interest. The Union can also
become company-dominated with the presence of managerial employees Villar v. Inciong
in Union membership.”
(LABOR UNION AND GOVERNMENT REGULATION: Effect of Freedom
An important element of the “confidential employee rule” is the of Association)
employee’s need to use labor relations information. Thus, in determining
the confidentiality of certain employees, a key question frequently Facts: Petitioners were members of the Amigo Employees Union-PAFLU,
considered is the employee’s necessary access to confidential labor a duly registered labor organization which, at the time of the present
relations information. dispute, was the existing bargaining agent of the employees in private
respondent Amigo Manufacturing, Inc. (hereinafter referred to as
(2) The fact that the three plants are located in three different places, Company). The Company and the Amigo Employees Union-PAFLU had a
namely, in Cabuyao, Laguna, in Otis, Pandacan, Metro Manila, and in San collective bargaining agreement governing their labor relations, which
Fernando, Pampanga is immaterial. Geographical location can be agreement was then about to expire on February 28, 1977. Within the last
completely disregarded if the communal or mutual interests of the sixty (60) days of the CBA, events transpired giving rise to the present
employees are not sacrificed. dispute.
An appropriate bargaining unit may be defined as “a group of employees Upon written authority of at least 30% of the employees in the company,
of a given employer, comprised of all or less than all of the entire body of including the petitioners, the Federation of Unions of Rizal (hereinafter
employees, which the collective interest of all the employees, consistent referred to as FUR) filed a petition for certification election with the Med-
with equity to the employer, indicate to be best suited to serve the Arbiter's Office, of the Ministry of Labor and Employment. The petition
reciprocal rights and duties of the parties under the collective bargaining was, however, opposed by the Philippine Association of Free Labor
provisions of the law.” Unions (hereinafter referred to as PAFLU) with whom, as stated earlier,
the Amigo Employees Union was at that time affiliated. PAFLU's
A unit to be appropriate must effect a grouping of employees who have opposition cited the "Code of Ethics" governing inter-federation disputes
substantial, mutual interests in wages, hours, working conditions and among and between members of the Trade Unions Congress of the
other subjects of collective bargaining. Philippines (hereinafter referred to as TUCP). Consequently, the Med-
Arbiter indorsed the case to TUCP for appropriate action but before any
such action could be taken thereon, the petitioners disauthorized FUR
VASSAR INDUSTRIES EMPLOYEES UNION (VIEU), petitioner, from continuing the petition for certification election for which reason
FUR withdrew the petition. The same employees who had signed the
vs. petition filed by FUR signed a joint resolution disaffiliating from PAFLU.

HON. FRANCISCO L. ESTRELLA; as Acting Director of the Bureau of Dolores Villar, representing herself to be the authorized representative of
Labor Relations, ASSOCIATED LABOR UNIONS (ALU), and VASSAR the Amigo Employees Union, filed a petition for certification election in
INDUSTRIES, INC., respondents. the Company. The Amigo Employees Union-PAFLU intervened and moved
for the dismissal of the petition for certification election filed by Villar, on
(LABOR UNION AND GOVERNMENT REGULATION: Action or Denial of the ground, among others that Villar had no legal personality to sign the
Application and Remedy) petition since she was not an officer of the union nor is there factual or
legal basis for her claim that she was the authorized representative of the
Facts: There was in existence a collective bargaining agreement between local union. Med-Arbiter dismissed the petition filed by Villar, which
private respondents Associated Labor Unions and Vassar Industries, Inc. dismissal is still pending appeal before BLR. Amigo Employees Union-
which expired on May 15, 1977. Prior to such date, 111 of a total number PAFLU called a special meeting of its general membership. A Resolution
of 150 employees of such firm disaffiliated from the former labor was thereby unanimously approved which called for the investigation by
organization and formed their own union. Thereafter, they filed an the PAFLU national president, of all of the petitioners and one Felipe
application for registration of their union with the Bureau of Labor Manlapao, for continuously maligning the union spreading false
Relations, complying with and the requirements of both the Labor Code propaganda that the union officers were merely appointees of the
and its implementing regulations. While such application was pending, management; and for causing divisiveness in the union.
petitioner Union filed a petition for certification as bargaining agent for
the rank-and-file employees of the company. The Med-Arbiter, on May 24, PAFLU formed a Trial Committee to investigate the local union's charges
1977, denied their plea on the ground that the union was not duly against the petitioners for acts of disloyalty. PAFLU and the Company
registered with the Department of Labor. concluded a new CBA which also reincorporated the same provisions of
the existing CBA, including the union security clause. PAFLU President
ISSUE: Whether or not an application for registration should be denied rendered a decision finding the petitioners guilty of the charges. PAFLU
just because there is already a registered collective bargaining agent in demanded the Company to terminate the employment of the petitioners
the company. pursuant to the security clause of the CBA. Acting on PAFLU's demand,
the Company informed PAFLU that it will first secure the necessary
clearances to terminate petitioners. PAFLU requested the Company to put
petitioners under preventive suspension pending the application for said
clearances to terminate the petitioners. The Company filed the request
for clearance to terminate the petitioners before DOLE which was FACTS: The rank and file workers of Formey Plastic, Inc. (FORMEY),
granted. DOLE Secretary Inciong denied the appeal, hence, this petition formed a local union known as Pambansang Kapatiran ng mga Anak
for review. Pawis sa Formey Plastic (KAPATIRAN) under the auspices of the National
WorkersBrotherhood (NWB). They ratified their Constitution and By-
Laws on 4 April 1993.

Issue: WON the termination of petitioners are proper. On 22 April 1993 KAPATIRAN filed a Petition for Certification Election
alleging that there was no existing andeffective CBA between FORMEY
Held: YES. It is true that disaffiliation from a labor union is not open to and any union; neither was there any recognized union within the
legal objection. It is implicit in the freedom of association ordained by the company. FORMEY moved to dismiss the petition while Kalipunan ng
Constitution. But this Court has laid down the ruling that a closed shop is Manggagawang Pilipino (KAMAPI) intervened and likewise moved to
a valid form of union security, and such provision in a collective dismiss on the ground that there was already a duly registered CBA
bargaining agreement is not a restriction of the right of freedom of covering period Jan. 1, 1992 to Dec. 31, 1996, therefore the contract bar
association guaranteed by the Constitution. rule will apply. KAPATIRAN opposed both motion to dismiss claiming
that the CBA executed between FORMEY and KAMAPI was fraudulently
In the case at bar, the Company and the Amigo Employees Union-PAFLU registered with the DOLE and that it was defective since what was
entered into a Collective Bargaining Agreement with a union security certified as bargaining agent was KAMAPI which as federation only
clause provided for in Article XII thereof which is a reiteration of the served as mere agent of the local union and without any legal personality
same clause in the old CBA. The quoted stipulation for closed-shop is to sign in behalf of the latter. Med-Arbiter found that there is a valid and
clear and unequivocal and it leaves no room for doubt that the employer existing CBA between FORMEY and KAMAPI which effectively barred the
is bound, under the collective bargaining agreement, to dismiss the filing for petition for certification election. KAPATIRAN appealed
employees, herein petitioners, for non- union membership. Petitioners imputing grave abuse of discretion to the Med-Arbiter in applying the
became non-union members upon their expulsion from the general contract bar rule. Secretary of Labor upheld the decision of Med-Arbiter.
membership of the Amigo Employees Union-PAFLU on March 15, 1977 KAPATIRAN filed a motion for reconsideration which was likewise
pursuant to the Decision of the PAFLU national president. denied.
The Court rejected petitioners' theory that their expulsion was not valid
upon the grounds adverted to earlier in this Decision. That PAFLU had the
authority to investigate petitioners on the charges filed by their co- ISSUES:
employees in the local union and after finding them guilty as charged, to
expel them from the roll of membership of the Amigo Employees Union- 1. Whether or not the petition for certification election was properly filed.
PAFLU is clear under the constitution of the PAFLU to which the local
union was affiliated. And pursuant to the security clause of the new CBA, 2. Whether or not there was a valid CBA between FORMEY and KAMAPI.
reiterating the same clause in the old CBA, PAFLU was justified in
applying said security clause.

Petitioners insist that their disaffiliation from PAFLU and filing a petition HELD:
for certification election are not acts of disloyalty but an exercise of their
1. No, the petition for certification election was not properly filed. The
right to self-organization. They contend that these acts were done within
CBA entered into between FORMEY and KAMAPI was made effectively
the 60-day freedom period when questions of representation may freely
Jan. 1, 1992 and will expire Dec. 31, 1996. The petition for certification
be raised. Under the peculiar facts of the case, We find petitioners'
election was filed on April 22, 1993 which was filed before the so-called
insistence untenable.
60-day freedom period
In the first place, had petitioners merely disaffiliated from the. Amigo
Employees Union-PAFLU, there could be no legal objections thereto for it
was their right to do so. But what petitioners did by the very clear terms 2. Yes, the court affirmed that there was a valid CBA between FORMEY
of their "Sama-Samang Kapasiyahan" was to disaffiliate the Amigo and KAMAPI. Art. 253-A of the labor code provides that “no petition
Employees Union-PAFLU from PAFLU, an act which they could not have questioning the majority status of the incumbent bargaining agent shall
done with any effective consequence because they constituted the be entertained and no certification election shall be conducted by the
minority in the Amigo Employees Union-PAFLU. DOLE outside the 60-dayperiod immediately before the date of expiry of
such 5 year term of the CBA.
Extant from the records is the fact that petitioners numbering ten (10),
were among the ninety-six (96) who signed the "Sama-Samang
Kapasiyahan" whereas there are two hundred thirty four (234) union
members in the Amigo Employees Union-PAFLU. Hence, petitioners WHEREFORE, the petition is DENIED. The decision of the Secretary of
constituted a small minority for which reason they could not have Labor and Employment dated 15 August1993 sustaining the order of the
successfully disaffiliated the local union from PAFLU. Since only 96 Med-Arbiter dated 31 May 1993 is AFFIRMED.
wanted disaffiliation, it can be inferred that the majority wanted the
union to remain an affiliate of PAFLU and this is not denied or disputed
by petitioners. The action of the majority must, therefore, prevail over
that of the minority members FURUSAWA RUBBER PHILS VS SECRETARY OF LABOR

A closed-shop is a valid form of union security, and a provision therefor in (LABOR UNION AND GOVERNMENT REGULATION: Effect of
a collective bargaining agreement is not a restriction of the right of Registration)
freedom of association guaranteed by the Constitution. (Manalang, et al.
vs. Artex Development Co., Inc., et al., L-20432, October 30, 1967, 21 SCRA Facts:
561). Where in a closed-shop agreement it is stipulated that union
members who cease to be in good standing shall immediately be FEU-IND filed a petition for certification election among rank and file
dismissed, such dismissal does not constitute an unfair labor practice employees of Furusawa. The petitioner moved to dismiss the petition as
exclusively cognizable by the Court of Industrial Relations. the respondents was not a legitimate labor organization for having
submitted a photocopy of the certificate which has not been duly
PAMBANSANG KAPATIRAN NG MGA ANAK PAWIS vs. SECRETARY OF authenticated and not supported by other documentary evidence
LABOR contrary to law. The Med-Arbiter found for FEU-IND. Furusawa appealed
to the Secretary of Labor which affirmed the order of theMed Arbiter.
(LABOR UNION AND GOVERNMENT REGULATION: Effect of Furusawa’s motion was reconsideration was subsequently denied.
Registration)
for clearance to terminate the petitioners before DOLE which was
granted. DOLE Secretary Inciong denied the appeal, hence, this petition
Issue: for review.

Whether or not failure of respondent to submit anoriginal copy of its


certificate of registration disqualifies it to file a certificationelection.
Issue: WON the termination of petitioners are proper.

Held: YES. It is true that disaffiliation from a labor union is not open to
Held: legal objection. It is implicit in the freedom of association ordained by the
Constitution. But this Court has laid down the ruling that a closed shop is
NO. The fact that FEU-IND was issuedthe certificate of registration is a valid form of union security, and such provision in a collective
sufficient proof of itslegitimacy. Failure to submitphotocopy thereof is not bargaining agreement is not a restriction of the right of freedom of
afatal defect and does not affect thelegitimate statis of thelabor association guaranteed by the Constitution.
organization. The granting of the certificate showsthat FEU-IND has
complied with the requirements of Art.134 of the LC.The assertion of In the case at bar, the Company and the Amigo Employees Union-PAFLU
Furusawa that FEU-IND was not able tocomply with the 20% entered into a Collective Bargaining Agreement with a union security
requirement is untenable as it hasalready been found by the Med-Arbiter clause provided for in Article XII thereof which is a reiteration of the
that it did. same clause in the old CBA. The quoted stipulation for closed-shop is
clear and unequivocal and it leaves no room for doubt that the employer
Villar v. Inciong is bound, under the collective bargaining agreement, to dismiss the
employees, herein petitioners, for non- union membership. Petitioners
(LABOR UNION AND GOVERNMENT REGULATION: Effect of non-
became non-union members upon their expulsion from the general
registration)
membership of the Amigo Employees Union-PAFLU on March 15, 1977
Facts: Petitioners were members of the Amigo Employees Union-PAFLU, pursuant to the Decision of the PAFLU national president.
a duly registered labor organization which, at the time of the present
The Court rejected petitioners' theory that their expulsion was not valid
dispute, was the existing bargaining agent of the employees in private
upon the grounds adverted to earlier in this Decision. That PAFLU had the
respondent Amigo Manufacturing, Inc. (hereinafter referred to as
authority to investigate petitioners on the charges filed by their co-
Company). The Company and the Amigo Employees Union-PAFLU had a
employees in the local union and after finding them guilty as charged, to
collective bargaining agreement governing their labor relations, which
expel them from the roll of membership of the Amigo Employees Union-
agreement was then about to expire on February 28, 1977. Within the last
PAFLU is clear under the constitution of the PAFLU to which the local
sixty (60) days of the CBA, events transpired giving rise to the present
union was affiliated. And pursuant to the security clause of the new CBA,
dispute.
reiterating the same clause in the old CBA, PAFLU was justified in
Upon written authority of at least 30% of the employees in the company, applying said security clause.
including the petitioners, the Federation of Unions of Rizal (hereinafter
Petitioners insist that their disaffiliation from PAFLU and filing a petition
referred to as FUR) filed a petition for certification election with the Med-
for certification election are not acts of disloyalty but an exercise of their
Arbiter's Office, of the Ministry of Labor and Employment. The petition
right to self-organization. They contend that these acts were done within
was, however, opposed by the Philippine Association of Free Labor
the 60-day freedom period when questions of representation may freely
Unions (hereinafter referred to as PAFLU) with whom, as stated earlier,
be raised. Under the peculiar facts of the case, We find petitioners'
the Amigo Employees Union was at that time affiliated. PAFLU's
insistence untenable.
opposition cited the "Code of Ethics" governing inter-federation disputes
among and between members of the Trade Unions Congress of the In the first place, had petitioners merely disaffiliated from the. Amigo
Philippines (hereinafter referred to as TUCP). Consequently, the Med- Employees Union-PAFLU, there could be no legal objections thereto for it
Arbiter indorsed the case to TUCP for appropriate action but before any was their right to do so. But what petitioners did by the very clear terms
such action could be taken thereon, the petitioners disauthorized FUR of their "Sama-Samang Kapasiyahan" was to disaffiliate the Amigo
from continuing the petition for certification election for which reason Employees Union-PAFLU from PAFLU, an act which they could not have
FUR withdrew the petition. The same employees who had signed the done with any effective consequence because they constituted the
petition filed by FUR signed a joint resolution disaffiliating from PAFLU. minority in the Amigo Employees Union-PAFLU.
Dolores Villar, representing herself to be the authorized representative of Extant from the records is the fact that petitioners numbering ten (10),
the Amigo Employees Union, filed a petition for certification election in were among the ninety-six (96) who signed the "Sama-Samang
the Company. The Amigo Employees Union-PAFLU intervened and moved Kapasiyahan" whereas there are two hundred thirty four (234) union
for the dismissal of the petition for certification election filed by Villar, on members in the Amigo Employees Union-PAFLU. Hence, petitioners
the ground, among others that Villar had no legal personality to sign the constituted a small minority for which reason they could not have
petition since she was not an officer of the union nor is there factual or successfully disaffiliated the local union from PAFLU. Since only 96
legal basis for her claim that she was the authorized representative of the wanted disaffiliation, it can be inferred that the majority wanted the
local union. Med-Arbiter dismissed the petition filed by Villar, which union to remain an affiliate of PAFLU and this is not denied or disputed
dismissal is still pending appeal before BLR. Amigo Employees Union- by petitioners. The action of the majority must, therefore, prevail over
PAFLU called a special meeting of its general membership. A Resolution that of the minority members
was thereby unanimously approved which called for the investigation by
the PAFLU national president, of all of the petitioners and one Felipe A closed-shop is a valid form of union security, and a provision therefor in
Manlapao, for continuously maligning the union spreading false a collective bargaining agreement is not a restriction of the right of
propaganda that the union officers were merely appointees of the freedom of association guaranteed by the Constitution. (Manalang, et al.
management; and for causing divisiveness in the union. vs. Artex Development Co., Inc., et al., L-20432, October 30, 1967, 21 SCRA
561). Where in a closed-shop agreement it is stipulated that union
PAFLU formed a Trial Committee to investigate the local union's charges members who cease to be in good standing shall immediately be
against the petitioners for acts of disloyalty. PAFLU and the Company dismissed, such dismissal does not constitute an unfair labor practice
concluded a new CBA which also reincorporated the same provisions of exclusively cognizable by the Court of Industrial Relations.
the existing CBA, including the union security clause. PAFLU President
rendered a decision finding the petitioners guilty of the charges. PAFLU
demanded the Company to terminate the employment of the petitioners
pursuant to the security clause of the CBA. Acting on PAFLU's demand, Tropical Hut Employee Union v. Tropical Hut Food Market, Inc. (181
the Company informed PAFLU that it will first secure the necessary SCRA 173)
clearances to terminate petitioners. PAFLU requested the Company to put
petitioners under preventive suspension pending the application for said (LABOR UNION AND GOVERNMENT REGULATION: Effect of non-
clearances to terminate the petitioners. The Company filed the request registration)
Further, there is no merit in the contention of the respondents that the
act of disaffiliation violated the union security clause of the CBA and that
FACTS: their dismissal as a consequence thereof is valid. A perusal of the
collective bargaining agreements shows that the THEU-NATU, and not the
• The rank and file workers of the Tropical Hut Food Market NATU federation, was recognized as the sole and exclusive collective
Incorporated organized a local union called the Tropical Hut Employees bargaining agent for all its workers and employees in all matters
Union (THEU) and immediately sought affiliation with the National concerning wages, hours of work and other terms and conditions of
Association of Trade Unions (NATU). employment. Although NATU was designated as the sole bargaining agent
in the check-off authorization form attached to the CBA, this simply
• The NATU accepted the THEU application for affiliation. They means it was acting only for and in behalf of its affiliate.
were accordingly registered and it appears, however, that NATU itself as
a labor federation was not registered with the Department of Labor. The NATU possessed the status of an agent while the local union
remained the basic principal union which entered into contract with the
• After several negotiations between the union and herein
respondent company. When the THEU disaffiliated from its mother
respondent a CBA was concluded containing the following security
federation, the former did not lose its legal personality as the bargaining
clause:
union under the CBA.
Union Membership and Union Check-off
Moreover, the union security clause embodied in the agreements cannot
Sec. 1 —. . . Employees who are already members of the UNION at the be used to justify the dismissals meted to petitioners since it is not
time of the signing of this Agreement or who become so thereafter shall applicable to the circumstances obtaining in this case. The CBA imposes
be required to maintain their membership therein as a condition of dismissal only in case an employee is expelled from the union for joining
continued employment. another federation or for forming another union or who fails or refuses to
maintain membership therein. The case at bar does not involve the
And check-off Authorization Form, the terms of which are as follows: withdrawal of merely some employees from the union but of the whole
THEU itself from its federation. Clearly, since there is no violation of the
We, the undersigned, hereby designate the NATIONAL Association of union security provision in the CBA, there was no sufficient ground to
Trade Unions, of which the TROPICAL HUT EMPLOYEES UNION is an terminate the employment of petitioners.
affiliate as sole collective bargaining agent in all matters relating to salary
rates, hours of work and other terms and conditions of employment in
the Tropical Hut Food Market, Inc.

• When their existing CBA ended, they entered into a new CBA
incorporating the previous union-shop security clause and the attached PROGRESSIVE DEVELOPMENT CORPORATION, petitioner, v. THE
check-off authorization form. HONORABLE SECRETARY, DEPARTMENT OF LABOR AND
EMPLOYMENT, MED-ARBITER EDGARDO DELA CRUZ, AND
• THEU disaffiliated from the NATU federation and affiliated PAMBANSANG KILUSAN NG PAGGAWA (KILUSAN)-TUCP,
with the Confederation of General Workers (CGW). respondents.

• Upon the request of NATU, respondent company applied for (LABOR UNION AND GOVERNMENT REGULATION: Effect of non-
clearance with the Secretary of Labor to dismiss the other officers and registration)
members of THEU-CGW. The company also suspended them effective that
day for violation of the union security clause. Consequently, complaints FACTS:
for unfair labor practices were filed by members of the THEU-CGW.
Pambansang Kilusan ng Paggawa –TUCP (KILUSAN) filed with the DOLE a
• NLRC ruled for the respondent company. petition for certification election among the rank-and-file employees of
Progressive Development Corporation (PDC), alleging that: (1) it is a
• On appeal, the Secretary of Labor affirmed NLRC’s decision. legitimate labor federation; (2) its local chapter, Progressive
Development Employees Union, was issued charter certificate; (3) there
ISSUE: WON the disaffiliation of the local union from the national was no existing collective bargaining agreement; and (4) no other
federation was valid legitimate labor organization existed in the bargaining unit.

HELD: YES, it is valid. PDC filed its motion to dismiss contending that KILUSAN failed to comply
with Rule II, Section 3, Book V of the Rules Implementing the Labor Code,
We held that the validity of the dismissals pursuant to the union security as amended, which requires the submission of: (a) the constitution and
clause in the collective bargaining agreement hinges on the validity of the by-laws; (b) names, addresses and list of officers and/or members; and
disaffiliation of the local union from the federation. (c) books of accounts. KILUSAN submitted a rejoinder claiming that it had
submitted the necessary documentary requirements for registration and
The right of a local union to disaffiliate from its mother federation is well- that since the local union was recently organized, no books of accounts
settled. A local union, being a separate and voluntary association, is free could be submitted.
to serve the interest of all its members including the freedom to
disaffiliate when circumstances warrant. This right is consistent with the Med-Arbiter held that there was substantial compliance with the
constitutional guarantee of freedom of association. requirements and the mere issuance of the charter certificate by the
federation was sufficient compliance with the rules. PDC’s MR was
The inclusion of the word NATU after the name of the local union THEU in treated as an appeal, as such, the Secretary of Labor took cognizance
the registration with the Department of Labor is merely to stress that the thereof and denied PDC’s MR. Thereafter, TRO was issued enjoining the
THEU is NATU's affiliate at the time of the registration. It does not mean public respondents from carrying out the resolutions and orders or from
that the said local union cannot stand on its own. Neither can it be proceeding with the certification election.
interpreted to mean that it cannot pursue its own interests independently
of the federation. A local union owes its creation and continued existence ISSUE:
to the will of its members and not to the federation to which it belongs.
Whether PDC failed to substantially comply with the requirements
When the local union withdrew from the old federation to join a new prescribed under Rule II, Section 3, Book V of the Rules Implementing the
federation, it was merely exercising its primary right to labor Labor Code, as amended.
organization for the effective enhancement and protection of common
interests. In the absence of enforceable provisions in the federation's
constitution preventing disaffiliation of a local union a local may sever its
relationship with its parent. RULING:
The Petition is GRANTED and the resolution and orders of the Med- Two of the CBAs were about to expire in May and June 1967. The other
Arbiter and Secretary of Labor and Employment are set aside. The TRO is one faced conflict as there was a rival union.
made permanent.
On March 14, 1967, the management of Marcelo Steel received a letter
requesting negotiation of a new CBA from PSSLU in behalf of UNWU.
There were also proposals from the unions in Marcelo Tire and Marcelo
YES. Rubber as the existing CBA was about to expire. Same day, the union oin
Marcelo Tire disauthorized PSSLU as their agent. Afterwards, the rival
A labor organization acquires legitimacy only upon registration with the union submitted ita own proposals.
BLR. Article 234 of the Labor Code prescribes the requirements for
registration. Prescribing those requirements does not limit the right of Another requests were received on May 3, 1967 and May 23, 1967 from
assembly or association but rather a condition sine qua non for the two different unions.
acquisition of legal personality by labor organizations, associations or
unions, and the possession of the rights and privileges granted by law to As the management was confused as to which of the union really
legitimate labor organizations. Besides, registration, as a valid exercise of represents the workers, the president asked for the proof of
police power, is required to protect both labor and the public against authorization from the unions and they were informed of the conflicting
abuses, fraud, or impostors who pose as organizers, although not truly claims and suggested that they file for certification election and the
accredited agents of the union the purport to represent. decision of the court shall be followed and respected.

Further, when an unregistered union becomes a branch, local or chapter PSSLU refused the suggestion of the management and said that they will
of a federation, some of the aforementioned requirements for registration file ULP for refusing to bargain with them. All of the unuons subsequently
are no longer required. A local or chapter need not be independently filed a Notice of Strike.
registered. A local or chapter becomes a legitimate labor organization
upon compliance with the provisions of Section 3, Rule II of Book V of the MUEWA was certified as the bargaining agent as it represents the
Implementing Rules, i.e., submission of the following to the BLR: 1) A majority of the workers in Marcelo Tire and that there were no
charter certificate, within 30 days from its issuance by the labor oppositions from the other union or interested persons.
federation or national union, and 2) The constitution and by-laws, a
statement on the set of officers, and the books of accounts all of which are Notices of Strike were withdrawn and the management agreed to sit
certified under oath by the secretary or treasurer, as the case may be, of down in a conference for the bargaining. On the fourth conference, Lakas
such local or chapter, and attested to by its president. Absent compliance declared a strike against Marcelo Companies. Acts of violence and
with these mandatory requirements, the local or chapter does not vandalism attended by picketing, the premises were blocked, windows of
become a legitimate labor organization. the plants were bad.y damaged.

Furthermore, in case of union registration, the rationale for requiring that Cases were filed against the strikers and a Return to Work order was
the submitted documents and papers be certified under oath by the agreed upon. Marcelo Companies resumed its operations and strikers
secretary or treasurer, as the case may be, and attested to by the went back to work.
president is that such submission becomes the BLR’s basis for the
Marcelo Companies and Lakas resumed their bargaining negotiations.
approval of the application for registration. Also, the certification and
attestation requirements are preventive measures against the On Oct. 13, 1967 the negotiations reached its final stage. Then Lakas
commission of fraud. declared another strike without filing a notice of strike resulting to
complete paralyzation of the business.
It may be noted as well that the mother union, acting for and in behalf of
its affiliate, had the status of an agent while the local union remained the Notices to return to work were posted and some of the strikers started
basic unit of the association, free to serve the common interest of all its working again. The management required the workers to fill up forms so
members, subject only to the restraints imposed by the constitution and that they may be given a schedule. However, the workers refused and
by-laws of the association. Thus, where as in this case the petition for insisted that they be admitted without complying to the said requirement.
certification election was filed by the federation which is merely an agent;
the petition is deemed to be filed by the chapter, the principal, which Lakas then filed a ULP case based on the alleged fact of non readmission
must be a legitimate labor organization. of striking members.
In this case, KILUSAN failed to substantially comply with the The trial court ruled that the Marcelo Companies were not remiss in their
aforementioned statutory requirements. The SC noted that big obligation to bargain and that the strikes conducted were illegal.
federations and national unions of workers should take the lead in However, it was decided that there was ULP in not readmitting all the
requiring their locals and chapters to faithfully comply with the law and strikers.
the rules instead of merely snapping union after union into their folds in a
furious bid with rival federations to get the most number of members. ISSUE:

-Whether or not Marcelo Companies are guilty of ULP

LAKAS NG MANGAGAGAWANG MAKABAYAN VS MARCELO -legitimate representation


ENTERPRISE
HELD:
GR. NO. L-38258
The SC ruled in favor of Marcelo Companies. Lakas was not the bargaining
J. GUERERO representative, yet the management did not ignore the demand for
collective bargaining neither it was refused.
(LABOR UNION AND GOVERNMENT REGULATION: Effect of Non-
registration) Marcelo Companies may rightfully demand for reasonable proof of
majority representation on the part of the supposed or putative
bargaining agent as it is a natural consequence of the employer’s duty to
bargain with the bargaining agent who represents the majority of the
FACTS:
workers. It is, however, necessary that such demand is made in good faith
and not as a pretext of delay or evasion.
On May 23, 1967, the Lakas had existing CBAs within the bargaining units
in the respective companies comprising Marcelo Companies. The said Marcelo Companies did not commit ULP. The facts of the case shows that
CBAs were entered into while they were affiliated with a national the strikers were readmitted to work and the form required was intended
federation, Phil Social Security Labor Union. for proper scheduling and not to prevent workers from coming back to
work. It is only those who did not report back to work who are not In this certiorari proceeding, it was alleged that public respondents
readmitted. should have cancelled the registration and permit of private respondent
labor organization as private respondent labor union had engaged in an
illegal strike. That was the novel issue raised in this petition. Solicitor
General Estelito P. Mendoza, in his exhaustive Comment, considered as
Alliance of Labor Organization v. Laguesma, the answer, found no merit in such an allegation and sustained the action
of respondent public officials.
(LABOR UNION AND GOVERNMENT REGULATION: Cancellation of
Union Certificate Registration) Petitioner, nonetheless, would seek to impart a semblance of plausibility
to its claim by the assertion that the Labor Code itself provides, in another
FACTS:
section, that cancellation of registration follows from "any activity
prohibited by law." The argument is false and misleading according to
the Comment of the Solicitor General. Thus: "By this amendatory law, it is
The Alliance of Democratic Free Labor Organization (ADFLO) filed an evident that no cause of action exists which will warrant the cancellation
application for registration as a national federation alleging, among of [Association of Democratic Labor Organization's] permit and
others that it has twelve (12) affiliates. After proper evaluation of its registration. Of course, petitioner tried to evade said issue by relying on
application, it was issued a Certificate of Registration to the federation. Article 240 (e) and Article 242 (p) of the Labor Code of the Philippines, as
The Confederation of Labor and Allied Social Services (CLASS) filed a amended. Let us examine its legal contention on this matter.
petition for the cancellation of the Registration Certificate issued to
ADFLO.

The first hearing conducted by the BLR after the case was remanded to it Issue: WON the strike is illegal?
for further proceedings. However, since CLASS was not yet ready with its
evidence, the hearing was postponed. CLASS then filed its Formal Offer of
Evidence. ADFLO filed an Objection to Admission of Exhibits based on the Held: No. The following shall constitute grounds for cancellation of union
grounds that the exhibits were not marked nor identified by any witness registration: * * * (e) Acting as a labor contractor or engaging in the
during the hearing of the case where ADFLO had been properly notified. "cabo" system, or otherwise engaging in any activity prohibited by law.'
In the meantime, at the hearing of the case, CLASS failed to appear and Suppletory to the above provision is Section 6 (c) of Rule II, Book V of the
only ADFLO’s President Antonio Cedilla appeared. Unaware that an Rules and Regulations implementing the Labor Code of the Philippines, as
objection had already been filed by ADFLO’s counsel, Cedilla manifested amended, which reads as follows: 'Section 6. Denial of Registration of
that ADFLO will file its answer to CLASS’ offer of evidence within thirty local unions. - The Regional Office may deny the application for
(30) days. BLR Director without first ruling on the admissibility of the registration on any of the following grounds: * * * (c) Engaging in the
exhibits of CLASS and without any further hearing then cancelled the "cabo" system or other illegal practices.' It is a fact that [Association of
registration of ADFLO. Democratic Labor Organization] is not a labor contractor or is it engaged
in the 'cabo' system or is it otherwise engaged in any activity of such
nature which is prohibited by law. The above-quoted article should not be
interpreted or construed to include an illegal strike engaged into by any
union. This is so because the phrase 'or otherwise engaging in any activity
ISSUE: prohibited by laws' should be construed to mean such activity engaged
into by a union that partakes of the nature of a labor contractor or 'cabo'
Whether or not a certificate of registration can be cancelled without system. The law does not intend to include in the said phrase illegally
hearing. declared strike simply because strike per se is legal. Also, if the law
intends to include illegally declared strike, the same could have been
expressly placed therein as had been previously done in Presidential
Decree No. 823."[11] Clearly, an awareness of the relevance of the
RULING: maxims noscitur a sociis and ejusdem generis ought to have cautioned
counsel for petitioner to shy away from this approach.
Subject to the requirements of notice and due process, the registration of
any legitimate labor union, chartered local and worker’s association may The realization must have dawned on petitioner's counsel, Ramos L. Cura,
be cancelled by the Regional Director, or in the case of federations, whose abilities could have been enlisted for a more worthwhile cause,
national or industry unions and trade union centers, by the Bureau that the petition filed by him hardly has any prospect for success. The
Director, by filing of an independent complaint or petition for Comment of Solicitor General Mendoza was filed on July 12, 1978. Then
cancellation. came, less than a month later, August 3, 1978 to be exact, a joint motion
to dismiss filed by petitioner and private respondent. It alleges: "1. That,
The cancellation of a certificate of registration is the equivalent of on February 27, 1978, petitioners filed with this Honorable Court a
snuffing out the life of a labor organization. For without such registration, petition for certiorari and mandamus; 2. That, after the filing of the
it loses — as a rule — its rights under the Labor Code. Under the aforesaid petition, the parties through their respective
circumstances, petitioner was indisputably entitled to be heard before a representatives/counsel, met for the purpose of amicable settlement of
judgment could be rendered cancelling its certificate of registration. In the issues raised in the aforesaid petition; 3. That, both parties have
David vs. Aguilizan it was held that a decision rendered without any threshed-out their respective disputes and have found ways and means
hearing is null and void. which would render the above-entitled case moot and academic; 4. That,
both parties are no longer interested in the outcome/result of this case
and pray of this Honorable Court to dismiss it for being moot and
Tablante v. Noriel academic."[12] The prayer is for the dismissal of the petition on the
ground that it is moot and academic.
(LABOR UNION AND GOVERNMENT REGULATION: : Cancellation of
Union Certificate Registration) WHEREFORE, this petition for certiorari is dismissed for being moot and
academic.
Petitioner Tablante-Tungol Enterprises, resolute in its determination not
to bargain collectively with private respondent, Association of
Democratic Labor Organization, has once again filed a certiorari
proceeding against respondents Director Carmelo C. Noriel, Bureau of
Labor Relations, and the Chief of its Med-Arbiter Section, Regional Office
No. 3, Eliseo Penaflor.
UST FACULTY UNION VS. BITONIO Supreme Court but the SC still favored the union. The SC remanded the
records for enforcement by respondent CIR. In examination of books, said
(UNION-MEMBER RELATIONS: Union Constitution) CIR’s Chief Examiner filed his Partial Report regarding the computation of
the benefits rendered in the case in favor of the Union.

Petitioners claim that in this computation of the Examiner did not,


FACTS: Private Respondents are duly elected officers of the UST Faculty include the claims of 70 other laborers whose total claims (for back
Union (USTFU). The union has a subsisting five-year CBA with UST. The wages), at the rate of P6,300.00 each and would be P441,000.00.
petitioners on the other hand, questioned before the Med-Arbiter, that Therefore, the correct. grand total amount due the laborers would be
the COMELEC was not constituted in accordance with USTFU’s 864,756.74.
constitution and by-laws (CBL) and that no rules had been issued to
govern the conduct of the 05 October 1996 election. Med-Arbiter issued a The trial judge took no action on the latest Urgent Motion of the union,
TRO enjoining the conduct of elections. However, a general faculty wherein it emphasized that respondent, with Court’s action rejecting its
assembly was held as scheduled. The general assembly was attended by appeal, no longer had any excuse for refusing to comply with the deposit
members of the USTFU and, as admitted by the appellants, also by “non- Order. Instead, an unscheduled conference was called and held on
USTFU members [who] are members in good standing of the UST October 31, 1963 in the chambers of the trial judge, and attended by
Academic Community Collective Bargaining Unit”. On this occasion, representatives of respondent firm, including their counsels of record and
appellants were elected as USTFU’s new set of officers by acclamation and the President of the union and 8 directors of the union. Four of these nine
clapping of hands. union representatives, including the union president himself, had no
claims or awards whatever under the judgment. Said union officials were
On 03 December 1996, appellants and UST allegedly entered into another not assisted by counsel, as petitioner Mary Concepcion, counsel of record
CBA covering the period from 01 June 1996 to 31 May 2001. Said CBA of the union, was not present, not having been notified of the conference.
was ratified by a majority of the UST faculty community.
In this conference respondent firm made again the same offer to settle
and quitclaim the judgment in favor of the union members for the same
amount of P110,000.00, which offer had already been ‘rejected by the
ISSUE: WON the election of the officers in this case was valid
union at the earlier conference held on June 25, 1963. But this time,
respondent and the directors of the union decided to settle the case
amicably with the payment by the firm of the same amount of
HELD: NO. The importance of a union’s constitution and bylaws cannot be P110,000.00 which was deposited with the Court’s disbursing officer
overemphasized. They embody a covenant between a union and its “immediately upon the signing of the settlement which will be prepared
members and constitute the fundamental law governing the members’ by the respondent firm through its counsel.”
rights and obligations. As such, the union’s constitution and bylaws
should be upheld, as long as they are not contrary to law, good morals or One of the union director together with 49 of its members questioned the
public policy. amicable settlement that took place. They claim that the Board of
Directors did not have any express authority of the members of the
A union election is held pursuant to the union’s constitution and bylaws, Santiago Labor Union to enter into any compromise for the sum of
and the right to vote in it is enjoyed only by union members. A union P110,000.00, that it was tainted by apparent bad faith on the part of the
election should be distinguished from a certification election, which is the President of the Union, that the amount of P110,000.00 is
process of determining, through secret ballot, the sole and exclusive unconscionable, considering, that the total claims of the members of the
bargaining agent of the employees in the appropriate bargaining unit, for union is more than P400,000.00.
purposes of collective bargaining. Specifically, the purpose of a
certification election is to ascertain whether or not a majority of the
employees wish to be represented by a labor organization and, in the ISSUE:
affirmative case, by which particular labor organization.

In a certification election, all employees belonging to the appropriate


bargaining unit can vote. Therefore, a union member who likewise Whether the amicable settlement is valid.
belongs to the appropriate bargaining unit is entitled to vote in said
election. However, the reverse is not always true; an employee belonging
to the appropriate bargaining unit but who is not a member of the union
cannot vote in the union election, unless otherwise authorized by the RULING:
constitution and bylaws of the union. Verily, union affairs and elections
cannot be decided in a non-union activity. NO. Petitioners were not accorded due process of law. The union was
deprived of the assistance of its counsel. The lack or due deliberation and
In both elections, there are procedures to be followed. Thus, the October caution in the trial judge’s instant approval of the settlement is seen from
4, 1996 election cannot properly be called a union election, because the the stipulations therein that the union thereby waived and quitclaimed
procedure laid down in the USTFU’s CBL for the election of officers was any and all claims which it may have against the respondent, as well as
not followed. It could not have been a certification election either, the claim of each and every one of the members of the union against
because representation was not the issue, and the proper procedure for respondent, when precisely the authority of the union board members to
such election was not followed. The participation of non-union members enter into any such compromise or settlement was under express
in the election aggravated its irregularity. challenge by petitioner Magalpo, a board member herself which the trial
judge completely disregarded.

Petitioners were deprived of the formal conference on the and of their


Heirs of Teodulo M. Cruz v. CIR right to be assisted by the union counsel as expressly requested, so that a
fair hearing could be accorded petitioners and an opportunity afforded
(UNION-MEMBER RELATIONS: Nature of Relationship) them to air their serious charges of bad faith and lack authority against
the Union leadership. Certainly, all these serious questions and charges
FACTS:
made by petitioners could have been threshed out and verified, if the
June 21, 1952, the Santiago Labor Union, composed of workers of the formal conference had been held with the presence of union counsel.
Santiago Rice Mill, a business enterprises engaged in the, buying. and
milling of palay at Santiago, Isabela, and owned and operated by King
Hong Co., Inc., filed before the respondent Court of Industrial Relations The transcript of the conference is deficient and does not reflect the
(CIR) cases for petition for overtime pay, premium pay for night, Sunday actual discussions and proceedings. This is to be deplored, for in a matter
and holiday work, and for reinstatement of workers illegally laid off. CIR of such great importance, especially where the union officials were
favored the union by a split decision of 3-2 vote. The case reached the
unassisted by counsel in an unscheduled conference, care should be taken fact and extent of the authority of the union leadership to execute any
by the trial judge that the proceedings are faithfully recorded. compromise or settlement of the judgment on behalf of the individual
workers who are the real judgment creditors.
We find the forcing through of the settlement arbitrary, unfair and
unconscionable. The settlement in the case at bar was precipitately approved without
verification of the union boards authority to execute the compromise
Another reason for striking down the settlement is the lack of any express settlement and that there was no such authority.
or specific authority of the president and majority. of the union board of
directors to execute the same and scale down the estimated P423,756.74 WHEREFORE, the respondent Court’s Orders are hereby declared null
judgment liability of respondent firm in favor of the individual union and void and set aside.
members to P110,000.00. On the contrary, petitioner board member
Magalpo timely challenged the authority or the union board to execute
any such settlement, expressly informing the trial judge that the union
had specifically appointed an entity in Manila, the “CREAM, Inc.,” as its Salunga v. CIR
attorney in-fact and “exclusive authorized representative for the
(UNION-MEMBER RELATIONS: Issue on Admission and Discipline)
evaluation, adjustment and -liquidation or its claim against respondent.
These union members have repudiated the former union president, FACTS:
Maylem and his board of directors, for having betrayed the union
members, and the new union leadership. 1) San Miguel Brewery, Inc (Company) entered with the Union, of
which respondent John de Castillo is the president, into a CBA.
The authority of the union, to execute a settlement of the judgment award
in favor of the individual union members, cannot be presumed but must
be expressly granted.
Section 3 of the CBA reads: The company agrees to require as a condition
Just as this Court has stricken down unjust exploitation of laborers by of employment of those workers covered by this agreement who either
oppressive employers, so will it strike down their unfair treatment by are members of the UNION on the date of the signing of this agreement,
their own unworthy leaders. The Constitution enjoins the State to afford or may join the UNION during the effectivity of this agreement, that they
protection to labor. Fair dealing is equally demanded of unions as well as shall not voluntarily resign from the UNION earlier than thirty (30) days
of employers in their dealings with employees. The union has been before the expiry date of this agreement as provided in Article XIII hereof,
evolved as an organization of collective strength for the protection of provided, however, that nothing herein contained shall be construed to
labor against the unjust exactions of capital, but equally important is the require the company to enforce any sanction whatsoever against any
requirement of fair dealing between the union and its members, which is employee or worker who fails to retain his membership in the UNION as
fiduciary in nature, and arises out of two factors: “one is the degree of hereinbefore stated, for any cause other than voluntary resignation or
dependence of the individual employee on the union organization; the non-payment of regular union dues on the part of said employee or
other, a corollary of the first, is the comprehensive power vested in the worker.
union with respect to the individual.” The union may, be considered but
the agent of its members for the purpose of securing for them fair and .
just wages and good working conditions and is subject to the obligation of
giving the members as its principals all information relevant to union and 2) Petitioner Francisco Salunga was a member of the National Brewery
labor matters entrusted to it. and Allied Industries Labor Union of the Philippines (PAFLU) since 1953.
On August 18, 1961, he tendered his resignation from the Union. The
The union leadership in the case at bar was recreant in its duty towards Union accepted the resignation, and transmitted it to the Company, with a
the union members in apparently having failed to disclose to the union request for the immediate implementation of said Section 3.
members the full situation of their judgment credit against respondent, to
wit, that they were in the advantageous position of being able to require 3) The Company informed petitioner that his resignation would result
enforcement of the respondent court’s P200,000.00-deposit order, and in in the termination of his employment, in view of Section 3
presuming that it had authority to waive and quitclaim the estimated
P423,756.74-judgment credit of the union members for the 4) Petitioner wrote to the Union a letter withdrawing or revoking his
unconscionable amount of P110,000.00, which had already been resignation and advising the Union to continue deducting his monthly
previously rejected by the workers. Respondent firm could not claim that union dues.
it dealt in good faith with the union officials, for it hastily executed the
purported settlement notwithstanding the serious charges of bad faith 5) The Union told the Company that petitioner's membership could not
against the union leadership, and the non-holding of the scheduled be reinstated and insisted on his separation from the service,
conference where the union leaders, at their express request, could be conformably with the stipulation above-quoted. The Company replied:
duly assisted by union counsel. The interests of the individual worker can Mr. Salunga told us that he did not realize that he would be losing his job
be better protected on the whole by a strong union aware of its moral and if he were to resign from the Union. We did not at any time ask or urge
legal obligations to represent the rank and file faithfully and secure for him to withdraw his resignation; neither are we now asking or insisting
them the best wages and working terms and conditions in the process of that you readmit him into your membership. We thought that informing
collective bargaining. As has been aptly pointed out, the will of the him of the consequences of his resignation from the Union, was the only
majority must prevail over that of the minority in the process, for “under humane thing to do under the circumstances. Nevertheless, if
the philosophy of collective responsibility, an employer who bargains in notwithstanding our foregoing clarification you still consider him as
good faith should be entitled to rely upon the promises and agreements of having actually resigned from your organization, and you insist that we
the union representatives with whom he must deal. under the dismiss him from the service in accordance with Sec. 3, Article II of our
compulsion of, law and contract. The ‘collective bargaining process agreement, we will have no alternative but to do so. The Company
should be carried on between parties who can mutually respect and rely notified petitioner that, in view of said letter and the aforementioned
upon the authority of each other.” Where, however, collective bargaining section, "we regret we have to terminate your employment for cause.”
process is not involved, and what is at stake are back wages already Petitioner was discharged from the employment of the Company.
earned by the individual workers by way of overtime, premium and
6) A prosecutor of the Court of Industrial Relations commenced the
differential pay, and final judgment has been rendered in their favor, as in
present proceedings for unfair labor practice against the Union, its
the present case, the real parties in interest with direct material interest,
president, respondent John de Castillo, respondent Cipriano Cid, as
as against the union which has only served as a vehicle for collective
PAFLU president, the Company, and its aforementioned Vice-President
action to enforce their just claims, are the individual workers themselves.
Miguel Noel.
Authority of the union to waive or quitclaim all or part of the judgment
award in favor of the individual workers cannot be lightly presumed but 7) The trial Judge rendered a decision directing them to readmit and to
must be expressly granted, and the employer, as judgment debtor, must continue the membership of Salunga in the membership rolls of the union
deal in all good faith with the union as the agent of the individual after paying all union dues
workers. The Court in turn should certainly verify and assure itself of the
8) This decision was reversed by the CIR — sitting en banc. Hence, this implementation of Section 3 of the bargaining contract, the Company
appeal by the petitioner. advised petitioner of the provision thereof, thereby intimating that he had
to withdraw his resignation in order to keep his employment. Besides, the
Company notified the Union that it (the Company) would not take any
action on the case and would consider the petitioner, "still a member" of
ISSUE: WON petitioner should be readmitted. the Union. When the latter, thereafter, insisted on petitioner's discharge,
the Company still demurred and explained it was not taking sides and
RULING: YES. Having been denied readmission into the Union and having
that its stand was prompted merely by "humane" considerations,
been dismissed from the service owing to an unfair labor practice on the
springing from the belief that petitioner had resigned from the Union
part of the Union, petitioner is entitled to reinstatement as member of the
without realizing its effect upon his employment. And, as the Union
Union.
reiterated its demand, the Company notified petitioner that it had no
The appeal is well taken, for, although petitioner had resigned from the other alternative but to terminate his employment, and dismissed him
Union and the latter had accepted the resignation, the former had, soon from the service, although with "regret".
later — upon learning that his withdrawal from the Union would result in
his separation from the Company, owing to the closed-shop provision
above referred to — revoked or withdrawn said resignation, and the Under these circumstances, the Company was not "unfair" to the
Union refused to consent thereto without any just cause therefor. petitioner. At the same time, the Company could not safely inquire into
the motives of the Union officers, in refusing to allow the petitioner to
withdraw his resignation. The arbitrary nature of the decision of said
The Union had not only acted arbitrarily in not allowing petitioner to officers was not such as to be apparent and to justify the company in
continue his membership. The trial Judge found said refusal of the Union regarding said decision unreasonable. Moreso, the petitioner had
officers to be due to his critical attitude towards certain measures taken appealed to the National Officers of the PAFLU and the latter had
or sanctioned by them. As set forth in the decision of the trial Judge: sustained the Union. The Company was justified in presuming that the
PAFLU had inquired into all relevant circumstances, including the
. . . Prior to August, 1961, he had been criticizing and objecting to what he motives of the Union Officers.
believed were illegal or irregular disbursements of union funds. Salunga
was later removed by the union from his position as steward without his
knowledge, and that the union did not honor the of attorney executed in
Having been denied readmission into the Union and having been
his favor by Alejandro Miranda, a co-worker, for the collection of
dismissed from the service owing to an unfair labor practice on the part
Miranda's indebtedness of P60.00 to him.
of the Union, petitioner is entitled to reinstatement as member of the
Union and to his former or substantially equivalent position in the
Company, without prejudice to his seniority and/or rights and privileges,
The officers of the Union tried to justify themselves by characterizing said and with back pay, which back pay shall be borne exclusively by the
criticisms as acts of disloyalty to the Union, which, of course, is not true, Union.
not only because the criticism assailed, not the Union, but certain acts of
its officers, and, indirectly, the officers themselves, but also because the
Constitution and By-laws of the Union explicitly recognize the right of its Villar v. Inciong
members to give their views on "all transactions made by the Union."
(UNION-MEMBER RELATIONS: Right to Discipline)

FACTS:
Although, generally, a state may not compel ordinary voluntary
associations to admit thereto any given individual, because membership 1) San Miguel Brewery, Inc (Company) entered with the Union, of
therein may be accorded or withheld as a matter of privilege, the rule is which respondent John de Castillo is the president, into a CBA.
qualified in respect of labor unions holding a monopoly in the supply of
labor, either in a given locality, or as regards a particular employer with
which it has a closed-shop agreement. The reason is that
Section 3 of the CBA reads: The company agrees to require as a condition
. . . The closed shop and the union shop cause the admission requirements of employment of those workers covered by this agreement who either
of trade union to become affected with the public interest. Likewise, a are members of the UNION on the date of the signing of this agreement,
closed shop, a union shop, or maintenance of membership clauses cause or may join the UNION during the effectivity of this agreement, that they
the administration of discipline by unions to be affected with the public shall not voluntarily resign from the UNION earlier than thirty (30) days
interest. before the expiry date of this agreement as provided in Article XIII hereof,
provided, however, that nothing herein contained shall be construed to
require the company to enforce any sanction whatsoever against any
employee or worker who fails to retain his membership in the UNION as
Consequently, it is well settled that such unions are not entitled to
hereinbefore stated, for any cause other than voluntary resignation or
arbitrarily exclude qualified applicants for membership, and a closed-
non-payment of regular union dues on the part of said employee or
shop provision would not justify the employer in discharging, or a union worker.
in insisting upon the discharge of, an employee whom the union thus
refuses to admit to membership, without any reasonable ground therefor. .
Needless to say, if said unions may be compelled to admit new members,
who have the requisite qualifications, with more reason may the law and 2) Petitioner Francisco Salunga was a member of the National Brewery
the courts exercise the coercive power when the employee involved is a and Allied Industries Labor Union of the Philippines (PAFLU) since 1953.
long standing union member, who, owing to provocations of union On August 18, 1961, he tendered his resignation from the Union. The
officers, was impelled to tender his resignation, which he forthwith Union accepted the resignation, and transmitted it to the Company, with a
withdrew or revoked. Surely, he may, at least, invoke the rights of those request for the immediate implementation of said Section 3.
who seek admission for the first time, and can not arbitrarily he denied
readmission. 3) The Company informed petitioner that his resignation would result
in the termination of his employment, in view of Section 3

4) Petitioner wrote to the Union a letter withdrawing or revoking his


We cannot agree, however, with the finding of the trial Judge to the effect resignation and advising the Union to continue deducting his monthly
that the Company was guilty of unfair labor practice. The Company was union dues.
reluctant — if not unwilling — to discharge the petitioner. When the
Union first informed the Company of petitioner's resignation and urged
5) The Union told the Company that petitioner's membership could not labor, either in a given locality, or as regards a particular employer with
be reinstated and insisted on his separation from the service, which it has a closed-shop agreement. The reason is that
conformably with the stipulation above-quoted. The Company replied:
Mr. Salunga told us that he did not realize that he would be losing his job . . . The closed shop and the union shop cause the admission requirements
if he were to resign from the Union. We did not at any time ask or urge of trade union to become affected with the public interest. Likewise, a
him to withdraw his resignation; neither are we now asking or insisting closed shop, a union shop, or maintenance of membership clauses cause
that you readmit him into your membership. We thought that informing the administration of discipline by unions to be affected with the public
him of the consequences of his resignation from the Union, was the only interest.
humane thing to do under the circumstances. Nevertheless, if
notwithstanding our foregoing clarification you still consider him as
having actually resigned from your organization, and you insist that we
Consequently, it is well settled that such unions are not entitled to
dismiss him from the service in accordance with Sec. 3, Article II of our
arbitrarily exclude qualified applicants for membership, and a closed-
agreement, we will have no alternative but to do so. The Company
shop provision would not justify the employer in discharging, or a union
notified petitioner that, in view of said letter and the aforementioned
in insisting upon the discharge of, an employee whom the union thus
section, "we regret we have to terminate your employment for cause.”
Petitioner was discharged from the employment of the Company. refuses to admit to membership, without any reasonable ground therefor.
Needless to say, if said unions may be compelled to admit new members,
6) A prosecutor of the Court of Industrial Relations commenced the who have the requisite qualifications, with more reason may the law and
present proceedings for unfair labor practice against the Union, its the courts exercise the coercive power when the employee involved is a
president, respondent John de Castillo, respondent Cipriano Cid, as long standing union member, who, owing to provocations of union
PAFLU president, the Company, and its aforementioned Vice-President officers, was impelled to tender his resignation, which he forthwith
Miguel Noel. withdrew or revoked. Surely, he may, at least, invoke the rights of those
who seek admission for the first time, and can not arbitrarily he denied
7) The trial Judge rendered a decision directing them to readmit and to readmission.
continue the membership of Salunga in the membership rolls of the union
after paying all union dues

8) This decision was reversed by the CIR — sitting en banc. Hence, this We cannot agree, however, with the finding of the trial Judge to the effect
appeal by the petitioner. that the Company was guilty of unfair labor practice. The Company was
reluctant — if not unwilling — to discharge the petitioner. When the
Union first informed the Company of petitioner's resignation and urged
implementation of Section 3 of the bargaining contract, the Company
ISSUE: WON petitioner should be readmitted. advised petitioner of the provision thereof, thereby intimating that he had
to withdraw his resignation in order to keep his employment. Besides, the
Company notified the Union that it (the Company) would not take any
action on the case and would consider the petitioner, "still a member" of
RULING: YES. Having been denied readmission into the Union and having the Union. When the latter, thereafter, insisted on petitioner's discharge,
been dismissed from the service owing to an unfair labor practice on the the Company still demurred and explained it was not taking sides and
part of the Union, petitioner is entitled to reinstatement as member of the that its stand was prompted merely by "humane" considerations,
Union. springing from the belief that petitioner had resigned from the Union
without realizing its effect upon his employment. And, as the Union
reiterated its demand, the Company notified petitioner that it had no
The appeal is well taken, for, although petitioner had resigned from the other alternative but to terminate his employment, and dismissed him
from the service, although with "regret".
Union and the latter had accepted the resignation, the former had, soon
later — upon learning that his withdrawal from the Union would result in
his separation from the Company, owing to the closed-shop provision
above referred to — revoked or withdrawn said resignation, and the Under these circumstances, the Company was not "unfair" to the
Union refused to consent thereto without any just cause therefor. petitioner. At the same time, the Company could not safely inquire into
the motives of the Union officers, in refusing to allow the petitioner to
withdraw his resignation. The arbitrary nature of the decision of said
The Union had not only acted arbitrarily in not allowing petitioner to officers was not such as to be apparent and to justify the company in
continue his membership. The trial Judge found said refusal of the Union regarding said decision unreasonable. Moreso, the petitioner had
officers to be due to his critical attitude towards certain measures taken appealed to the National Officers of the PAFLU and the latter had
or sanctioned by them. As set forth in the decision of the trial Judge: sustained the Union. The Company was justified in presuming that the
PAFLU had inquired into all relevant circumstances, including the
. . . Prior to August, 1961, he had been criticizing and objecting to what he motives of the Union Officers.
believed were illegal or irregular disbursements of union funds. Salunga
was later removed by the union from his position as steward without his
knowledge, and that the union did not honor the of attorney executed in
Having been denied readmission into the Union and having been
his favor by Alejandro Miranda, a co-worker, for the collection of
dismissed from the service owing to an unfair labor practice on the part
Miranda's indebtedness of P60.00 to him.
of the Union, petitioner is entitled to reinstatement as member of the
Union and to his former or substantially equivalent position in the
Company, without prejudice to his seniority and/or rights and privileges,
The officers of the Union tried to justify themselves by characterizing said and with back pay, which back pay shall be borne exclusively by the
criticisms as acts of disloyalty to the Union, which, of course, is not true, Union.
not only because the criticism assailed, not the Union, but certain acts of
its officers, and, indirectly, the officers themselves, but also because the
Constitution and By-laws of the Union explicitly recognize the right of its
Kapisanan ng mga Manggagawa sa MRR v. Bugay, 4 SCRA 487 [G.R.
members to give their views on "all transactions made by the Union."
No. L-13093, Feb. 28, 1962]

(UNION-MEMBER RELATIONS: Due Process Rule)


Although, generally, a state may not compel ordinary voluntary
associations to admit thereto any given individual, because membership
therein may be accorded or withheld as a matter of privilege, the rule is - Facts
qualified in respect of labor unions holding a monopoly in the supply of
o Paulino Bugay filed against the Kapisanan Ng Mga petitioners staged a strike inside the company premises. After four (4)
Manggagawa Sa Manila Railroad Company before the Court of First days the strike was settled. An agreement was entered into by the
Instance of Manila an action for moral damages arising out of an unfair representatives of the management, Lacanilao group and the Tancinco
labor practice allegedly committed by said union which was the subject of group the relevant terms of which are as follows:
a decision rendered by the Court of Industrial Relations finding said
union guilty as charged.

- Issue “1. That all monthly-paid employees shall be United under one union, the
ITM Monthly Employees Association (ITM-MEA), to be affiliated with
o Was Bugay’s expulsion illegal because of the irregularities ANGLO;
committed in his investigation?
2. That the management of ITM recognizes ANGLO as the sole and
- Held exclusive bargaining agent of all the monthly-paid employees;

o Yes 3. That an election of union officers shall be held on 26 May l986, from
8:00 a.m. to 5:00 p.m.;
o The finding that defendant union was guilty of the unfair labor
practice preferred against it by plaintiff, the Court of Industrial Relations 4. That the last day of filing of candidacy shall be on l9 May l986 at 4:00
made the following comment: p.m.;

"It is to be noted that both in the investigation held by the 5. That a final pre-election conference to finalize the list of qualified
investigation committee of the Kapisanan and in the board meeting of voters shall be held on 19 May 1986, at 5:00 p.m.;”
June 14, 1953, where the committee's report recommending expulsion
was approved, Bugay was not present.

As has been pointed out earlier, the reason for Bugay's failure A pre-election conference was held, but the parties failed to agree on the
to attend the investigation does not appear of record. list of voters. During the May 21, 1986 pre-election conference attended
by MOLE officers, ANGLO through its National Secretary, a certain Mr.
On the other hand, during the board meeting, the committee of Cornelio A. Sy made a unilateral ruling excluding some 56 employees
three board members assigned to summon Bugay failed to serve notice consisting of the Manila office employees, members of Iglesia ni Kristo,
upon him because he was then in Lucena, Quezon. non-time card employees, drivers of Mrs. Salazar and the cooperative
employees of Mrs. Salazar. Prior to the holding of the election of union
Why all these proceedings were continued by the respondents officers petitioners, 2 through a letter addressed to the Election
inspite of Bugay's absence remains unexplained in the record. Supervisor, MOLE San Fernando Pampanga, protested said ruling but no
action was taken. On May 26, 1986, the election of officers was conducted
But one thing is certain, whatever might be the merits of the under the supervision of MOLE wherein the 56 employees in question
charge filed by respondent Olazo against him, Bugay did not have participated but whose votes were segregated without being counted.
sufficient opportunity to defend himself. Lacanilao’s group won. Lacanilao garnered 119 votes with a margin of
three (3) votes over Tancinco prompting petitioners to make a protest.
Such proceedings, being violative of the elementary rule of
Thereafter, petitioners filed a formal protest with the Ministry of Labor
justice and fair play, can not give validity to any act done pursuant
Regional Office in San Fernando, Pampanga 3 claiming that the
thereto.
determination of the qualification of the 56 votes is beyond the
o The fact remains that the CIR and the Supreme Court have competence of ANGLO. Private respondents maintain the contrary on the
found that his expulsion was illegal because of the irregularities premise that definition of union’s membership is solely within their
committed in his investigation. jurisdiction.

o In effect, it was found that not only has he not been given an On the basis of the position papers submitted by the parties MOLE’s Med
opportunity to defend himself but his expulsion was not submitted to the Arbiter 4 issued an order dated July 25, 1986 directing the opening and
different chapters of the union as required by its constitution and by- counting of the segregated votes. 5 From the said order private
laws. respondents appealed to the Bureau of Labor Relations (BLR) justifying
the disenfranchisement of the 56 votes. Private respondents categorized
the challenged voters into four groups namely, the Manila Employees,
that they are personal employees of Mr. Lee; the Iglesia ni Kristo, that
TANCINCO vs. PURA FERER-CALLEJA allowing them to vote will be anomalous since it is their policy not to
participate in any form of union activities; the non-time card employees,
(UNION-MEMBER RELATIONS: Issues on Election of Officers – that they are managerial employees; and the employees of the
Qualification, Manner of Election, Tenure and Compensation: Voter’s cooperative as non-ITM employees. 6 On December 10, 1986, BLR
List) rendered a decision 7 holding the exclusion of the 56 employees as
arbitrary, whimsical, and wanting in legal basis 8 but set aside the
challenged order of July 26, 1986 on the ground that 51 ** of 56
challenged voters were not yet union members at the time of the election
FACTS: per April 24, 1986 list submitted before the Bureau. 9 The decision
directed among others the proclamation of Lacanilao’s group as the duly
elected officers and for ITM-MEA to absorb in the bargaining unit the
This special civil action for certiorari seeks to annul the Resolution and challenged voters unless proven to be managerial employees. 10
the Decision of the Bureau of Labor Relations setting aside the order Petitioners’ motion for reconsideration was likewise denied.
which decreed the inclusion and counting of the 56 segregated votes for
Dissatisfied with the turn of events narrated above petitioners elevated
the determination of the results of the election of officers of Imperial
the case to this Court by way of the instant petition for certiorari under
Textile Mills Inc. Monthly Employees Association (ITM-MEA).
Rule 65 of the Rules of Court.
Private respondents are the prime organizers of ITM-MEA. While said
Petitioners allege that public respondent director of Labor Relations
respondents were preparing to file a petition for direct certification of the
committed grave abuse of discretion in ordering the Med-Arbiter to
Union as the sole and exclusive bargaining agent of ITM’s bargaining unit,
disregard the 56 segregated votes and proclaim private respondents as
the union’s Vice-President, Carlos Dalmacio was promoted to the position
the duly elected officers of ITM-MEA whereas said respondent ruled that
of Department Head, thereby disqualifying him for union membership.
the grounds relied upon by ANGLO for the exclusion of voters are
Said incident, among others led to a strike spearheaded by Lacanilao
arbitrary, whimsical and without legal basis.
group, respondents herein. Another group however, led by herein
(UNION-MEMBER RELATIONS: Issues on Election of Officers –
Qualification, Manner of Election, Tenure and Compensation:
ISSUE: Disqualification of Candidate)

Facts: The petitioners were disqualified from running as candidates in


the election of APCWU officers by the Med-Arbiter, which election had
Whether the order of segregation (exclusion) of 56 votes of the theretofore been scheduled for November 17, 1981 but was enjoined and
petitioner’s union by the respondent is tantamount to grave abuse of ordered reset. However, on appeal, said order was set aside by the
discretion. Director of the Bureau of Labor. Thereafter, the election of officers and
board members of the union was held, with the candidates of the
petitioners, that is, Manalad, Leano and Puerto, winning over those of the
HELD: private respondents, who were Babula, Mijares and Navarro, for the
positions of president, treasurer and auditor, respectively. As a
The petition is impressed with merit. The record of the case shows that consequence, the latter group filed a petition for review with this Court
public respondent categorically declared as arbitrary, whimsical and assailing the aforesaid order of October 31, 1984 of the Bureau of Labor
without legal basis the grounds 11 relied upon by ANGLO in Relations which had declared the aforesaid petitioners eligible to run for
disenfranchising the 56 voters in question. However, despite said finding said union offices.
public respondent ruled to set aside the Resolution of July 25, 1986 of the
Med-Arbiter based on its own findings 12 that 51 of the 56 The SC in a previous case entitled “Associated Port Checkers and Workers
disenfranchised voters were not yet union members at the time of the Union, et al. vs. Ricardo R. Manalad, et al.” however declared vacant all
election of union officers on May 26, 1986 on the ground that their names positions and ordered the winners of the elections to stop acting as
officers, and to turn-over all union funds.
do not appear in the records of the Union submitted to the Labor
Organization Division of the Bureau of Labor on April 24, 1986.
Pursuant thereto, the Director of the Bureau of Labor Relations issued an
The finding does not have a leg to stand on. Submission of the employees order on July 10, 1985 to the effect that he was taking over the
names with the BLR as qualified members of the union is not a condition management of the affairs of said union, ordering private respondents
sine qua non to enable said members to vote in the election of union’s Babula and all other persons to cease acting as officers of the union, and
officers. It finds no support in fact and in law. Per public respondent’s requiring them to turn over the union funds to said director.
findings, the April 24, 1986 list consists of 158 union members only 13 Subsequently, the Court’s aforesaid resolution of July 3, 1985 was
wherein 51 of the 56 challenged voters’ names do not appear. Adopting modified on July 17, 1985 by providing that the special election scheduled
however a rough estimate of a total number of union members who cast on July 20, 1985 shall be held under the personal supervision of
their votes of some 333 14 and excluding therefrom the 56 challenged respondent Director Trajano. Private respondents still won the elections,
votes, if the list is to be the basis as to who the union members are then despite attempts from petitioners to have them disqualified for their
public respondent should have also disqualified some 175 of the 333 alleged refusal to comply with the resolution of the SC.
voters. It is true that under article 242(c) of the Labor Code, as amended,
Director Trajano issued a resolution proclaiming private respondents as
only members of the union can participate in the election of union
the winners in the special election and duly elected officers of APCWU,
officers. The question however of eligibility to vote may be determined
with the following observation: “The submission that Mr. Babula failed to
through the use of the applicable payroll period and employee’s status
completely turn over management of the union to the undersigned is
during the applicable payroll period. The payroll of the month next
within the competence and authority of the Supreme Court to pass upon
preceding the labor dispute in case of regular employees 15 and the considering that the mandate for such a turn-over came from the Court.
payroll period at or near the peak of operations in case of employees in
seasonal industries. Meanwhile, the three-year term of the private respondents under the
disputed July 20, 1985 elections expired on July 20, 1988.
In the case before Us, considering that none of the parties insisted on the
use of the payroll period-list as voting list and considering further that However petitioners insist that they be declared the winners in said
the 51 remaining employees were correctly ruled to be qualified for election (in 1985) with their terms of three (3) years to commence from
membership, their act of joining the election by casting their votes on the time they assume office in execution of a final and executory
May 26, 1986 after the May 10, 1986 agreement is a clear manifestation resolution of the SC.
of their intention to join the union. They must therefore be considered
ipso facto members thereof Said employees having exercised their right Issue: WON respondents are disqualified
to unionism by joining ITM-MEA their decision is paramount. Their
names could not have been included in the list of employee submitted on
April 24, 1986 to the Bureau of Labor for the agreement to join the union
was entered into only on May 10, 1986. Indeed the election was Held: It is pointless and unrealistic to insist on annulling an election of
supervised by the Department of Labor where said 56 members were officers whose terms had already expired. The SC would have thereby a
allowed to vote. Private respondents never challenged their right to vote judgment on a matter which cannot have any practical legal effect upon a
then. controversy, even if existing, and which, in the nature of things, cannot be
enforced.
The Solicitor General in his manifestation agreed with petitioners that
public respondent committed a grave abuse of discretion in deciding the We agree with the petitioners that disobedience to a resolution of this
issue on the basis of the records of membership of the union as of April Court should not be left unpunished. However, before the alleged
24, 1986 when this issue was not put forward in the appeal. disobedient party may be cited for contempt, the allegations against him
should be clearly established. The contentions of petitioners, even
WHEREFORE, premises considered, the petition for certiorari is disregarding some evidential deficiencies, do not adequately establish the
GRANTED. The temporary restraining order issued by this Court is basis for contempt. On the contrary, respondents have satisfactorily
hereby made permanent. The questioned Resolution and the Decision are answered the averments thereon.
hereby set aside for being null and void and the Order of the Mediator
Arbiter is hereby declared immediately executory. At this juncture, it would further be appropriate to remind petitioners
that even if the disqualification of private respondents could be justified,
Cost against private respondents. the candidates of petitioners certainly cannot be declared as the winners
in the disputed election. The mere fact that they obtained the second
highest number of votes does not mean that they will thereby be
considered as the elected officers if the true winners are disqualified.
Manalad v. Trajano
(UNION-MEMBER RELATIONS: Issues on Election of Officers –
Qualification, Manner of Election, Tenure and Compensation: Invalid
Election)
Kapisanan ng Manggagawang Pinagyakap (KMP) vs. Trajano

(UNION-MEMBER RELATIONS: Issues on Election of Officers – FACTS:


Qualification, Manner of Election, Tenure and Compensation:
Expulsion Remedy) 1. The consolidated cases involve disputes among employees of the
Philippines Long Distance Telephone Company (PLDT), who are
members of the same union, the Free Telephone Workers Union (FTWU).

FACTS: On June 30, 1981, a written request for accounts examination of 2. The disputes concern the validity of the general elections for union
the financial status of KMP Labor Union, the existing labor union at officers in 1986, and the increase of union dues adopted and put into
Franklin Baker Company in San Pablo City, was filed by private effect by the incumbent officers subsequent to said elections.
respondent Silvestre and 13 other employees, who are members of the
union. Acting on said request, Union Account Examiner Vicedo of the 3. Assailed by the petitioners in this case are (1) the decision of the
MOLE conducted the investigation and thereafter submitted a report. Director of Labor Relations (BLR) annulling the elections of officers of the
Based on the revelations, private respondents filed with the Regional labor union, the Free Telephone Workers Union (FTWU), and (2) the
Office QC, MOLE, a petition for the expulsion of the union officers.They resolution denying their motion for reconsideration of the decision.
committed gross violation of the Labor Code, specifically pars. (a), (b), (g),
(h), (j), and (k) of Article 242; and, the constitution and by-laws of the 4. The union's by-laws provide for the election of officers every three (3)
union, Sections 6 and 7 years, in the month of July.

Union Officers: Denied imputation and argued that the disallowed 5. Pursuant thereto, the union's Legislative Council set the provincial
expenditures were made in good faith; that the same conduced the elections for its officers on July 14 to 18, 1986, and those for Metro Manila
benefit of members, they are willing to reimburse the same from their on July 25, 1986.
own personal funds,they should not be held accountable for the non-
production of books of accounts of the Union for years 1977, 1978, and 6. The same Council also quite drastically raised the fees for the filing of
1979 because they were not the officers then and not one of the former certificates of candidates which had therefore ranged from P75.00 to
officers of the Union had turned over to them the records, And non- P100.00.
ratification of the constitution and by-laws of the Union and the non-
7. The filing fee for each candidate for president of the labor
segregation of the Union funds occurred before they became officers and
that they have already been correcting the same. organization was increased to P3,000; that for each candidate for vice-
president, secretary general, treasurer and auditor, to P2,000.00; and that
Med-Arbiter Cabibihan ordered the holding of a referendum, to be for assistant secretary, assistant treasurer and assistant auditor, to
conducted under the supervision of BLR. Petitioners appealed the order P1,000.00 each.
to respondent Trajano of BLR the disallowed expenditures of P1,278.00
were made in good faith and not used for the personal benefit of herein
union officers but, instead, contributed to the benefit of the members they ISSUES
were elected in 1980 only and, therefore, they could not be made
responsible for the omissions of their predecessors who failed to turn 1. Whether or not there is any grave abuse of discretion tainting the
over union records for the questioned period there would be a general adjudgment of respondent Director of Labor Relations that the general
election on Oct. 4, 1982, at which time, both the election and the desired elections for union officers held in 1986 were attended by grave
referendum could be undertaken to determine the membership at irregularities, rendering the elections invalid
minimum expense they prayed that resolution on the issue be held in
abeyance.Respondents claimed that Med-Arbiter erred in calling a 2. Whether or not the Matter of 30%-Support for Complaints for
referendum to decide issue; the appropriate action should be the Violations of Union Membership Rights was mandatory
expulsion of union officers. Trajano dismissed both appeals and affirmed
in toto the order of Med-Arbiter 3. Whether or not the resolution increasing the union dues must
therefore be struck down, as illegal and void
ISSUE: Whether or not union officers were guilty of the alleged acts
imputed against them thus expulsion was proper

HELD: NO RULING:

If herein union officers (also petitioners) were guilty of the alleged acts 1. No, there is no grave abuse of discretion on the part of respondent
imputed against them, said public respondent pursuant to Article 242 of Director of Labor Relations.
the New Labor Code and in the light of Our ruling in Duyag vs. Inciong, 98
SCRA 522, should have meted out the appropriate penalty on them, i.e., to No, because a review of the record fails to disclose any grave abuse of
expel them from the Union, as prayed for, and not call for a referendum to discretion tainting the adjudgment of respondent Director of Labor
decide the issue; the alleged falsification and misrepresentation of herein Relations that the general elections for union officers held in 1986 were
union officers were not supported by substantial evidence. The fact that attended by grave irregularities, rendering the elections invalid. That
they disbursed the amount of P1,278.00 from Union funds and later on finding must thus be sustained.
was disallowed for failure to attach supporting papers thereon did not of
itself constitute falsification and/or misrepresentation. The expenditures 2. No, pursuant to Article 242 of the Labor Code.
appeared to have been made in good faith and the amount spent for the
No, because the respondent Director's ruling, however, that the assent of
purpose mentioned in the report, if concurred in or accepted by the
30% of the union membership, mentioned in Article 242 of the Labor
members, are reasonable; and the repudiation of both private
Code, was mandatory and essential to the filing of a complaint for any
respondents to the highly sensitive position of auditor at the October 4,
violation of rights and conditions of membership in a labor organization
1982 election, is a convincing manifestation and demonstration of the
(such as the arbitrary and oppressive increase of union dues here
union membership's faith in the herein officers' leadership on one hand
complained of), cannot be affirmed and will be reversed.
and a clear condonation of an act they had allegedly committed.
The very article relied upon militates against the proposition. It states
that a report of a violation of rights and conditions of membership in a
Rodriguez v. DIR labor organization maybe made by "(a)t least thirty percent (30%) of all
the members of a union or any member or members specially concerned."
The use of the permissive "may" in the provision at once negates the title to a tract of land in Caloocan, Rizal. The parcel of land was eventually
notion that the assent of 30% of all the members is mandatory. More registered in the name of the Union.
decisive is the fact that the provision expressly declares that the report
may be made, alternatively by "any member or members specially The Union, through Atty. Pineda, filed an urgent motion with the Ministry
concerned." of Labor and Employment (MOLE) requesting for authority to sell and
dispose of the property.
And further confirmation that the assent of 30% of the union members is
not a factor in the acquisition of jurisdiction by the Bureau of Labor Atty. Espinas, (the original counsel) established the award of 897
Relations is furnished by Article 226 of the same Labor Code, which workers' claim. When Atty. Pineda appeared for the Union in these cases,
grants original and exclusive jurisdiction to the Bureau, and the Labor still an associate of the law firm, his appearance carried the firm name
Relations Division in the Regional Offices of the Department of Labor, “B.C. Pineda and Associates," giving the impression that he was the
over "all inter-union and intra-union conflicts, and all disputes, principal lawyer in these cases.
grievances or problems arising from or affecting labor management
relations," making no reference whatsoever to any such 30 % support Atty. Pineda, without authority from the Supreme Court but relying on
requirement. the earlier authority given him by the Ministry of Labor, filed another
urgent motion, praying that the Union be authorized to sell the lot. The
Indeed, the officials are given the power to act "on all inter-union and sale was finally consummated, resulting in the execution of an escrow
intra-union conflicts (1) "upon request of either or both parties" as well agreement.
as (2) "at their own initiative."
Issue:

Whether or not Atty. Pineda and Arbiter Valenzuela should be held in


3. Yes, the resolution increasing the union dues must therefore be struck contempt.
down, as illegal and void, arbitrary and oppressive.
Held:

YES. Contempt of court is a defiance of the authority, justice or dignity of


Yes, because the respondent Director found that the resolution of the the court; such conduct as tends to bring the authority and
union's Legislative Council to this effect does not bear the signature of at administration of the law into disrespect or to interfere with or prejudice
least two-thirds (2/3) of the members of the Council, contrary to the parties’ litigant or their witnesses during litigation.
requirement of the union constitution and by-laws; and that proof is
wanting of proper ratification of the resolution by a majority of the The power to punish for contempt is inherent in all courts and is essential
general union membership at a plebiscite called and conducted for that to the preservation of order in judicial proceedings and to the
purpose, again in violation of the constitution and by-laws. The resolution enforcement of judgments, orders, and mandates of the court, and
increasing the union dues must therefore be struck down, as illegal and consequently, to the due administration of justice.
void, arbitrary and oppressive. The collection of union dues at the
The Court may suspend or disbar a lawyer for any conduct on his part
increased rates must be discontinued; and the dues thus far improperly
showing his unfitness for the confidence and trust which characterize the
collected must be refunded to the union members or held in trust for
attorney and client relations, and the practice of law before the courts, or
disposition by them in accordance with their charter and rules, in line
with this Court's ruling in a parallel situation, viz: showing such a lack of personal honesty or of good moral character as to
render him unworthy of public confidence.
... All amounts already collected must be credited accordingly in favor of
In the case, the expeditious manner by which Arbiter Valenzuela granted
the respective members either for their future legal dues or other
Atty. Pineda's motion for such authority to sell the property makes the
assessments or even delinquencies, if any. And if this arrangement
entire transaction dubious and irregular.
regarding the actual refund of what might be excessive dues is not
acceptable to the majority of the members, the matter may be decided in Significantly Atty. Pineda's act of filing a motion praying for authority to
a general meeting called for the purpose. sell was by itself an admission on his part that he did not possess the
authority to sell the property. He could not and did not even wait for valid
DISPOSITIVE: WHEREFORE, in G.R. Nos. 76579-82, the petition for
authority but instead previously obtained the same from the labor arbiter
certiorari is DISMISSED, no grave abuse of discretion or other serious
whom he knew was not empowered to so authorize.
error having been shown in the decision of the respondent Director of
Labor Relations, said decision — ordering the holding of new elections Atty. Pineda is found guilty of indirect contempt of court for which he is
for officers of the Free Telephone Worker Union — being on the contrary sentenced to imprisonment and directed to show because why he should
in accord with the facts and the law. not be disbarred.

DOCTRINE: It goes without saying that free and honest elections are Pacific Banking Corp. v. Clave
indispensable to the enjoyment by employees and workers of their
constitutionally protected right to self-organization. That right "would be (UNION-MEMBER RELATIONS: Major Policy Matters: Source of
diluted if in the choice of the officials to govern ... (union) affairs, the Payment of Attorney’s fees)
election is not fairly and honestly conducted," and the labor officers
concerned and the courts have the duty "to see to it that no abuse is
committed by any official of a labor organization in the conduct of its
affairs. Facts: This case is about the legality of deducting from the monetary
benefits awarded in a collective bargaining agreement the attorney's fees
of the lawyer who assisted the union president in negotiating the
agreement. It also involves the jurisdiction of the Office of the President
HALILI VS CIR of the Philippines to order such deduction.
(UNION-MEMBER RELATIONS: Major Policy Matters: Legal Counsel) Since January, 1979, there had been negotiations between the Pacific
Banking Corporation and the Pacific Banking Corporation Employees
Facts:
Organization (PABECO) for a collective bargaining agreement for 1979 to
1981. Because of a deadlock, the Minister of Labor assumed jurisdiction
The cases involve disputes regarding claims for overtime of more than
over the controversy. On July 10, 1979, the Deputy Minister rendered a
five hundred bus drivers and conductors of Halili Transit. The disputes
decision directing the parties to execute a CBA in accordance with the
were eventually settled when the contending parties reached an
terms and conditions set forth in his decision.
Agreement where the Administratrix would transfer to the employees the
The union was represented in the negotiations by its president, Paula S. Moreover, the case is covered squarely by the mandatory and explicit
Paug, allegedly assisted as consultant by Jose Umali, Jr., the president of prescription of article 222 which is another guarantee intended to protect
the National Union of Bank Employees (NUBE) with which it was the employee against unwarranted practices that would dimi-nish his
formerly affiliated (p. 209, Rollo). Lawyer Juanito M. Saavedra's earliest compensation without his knowledge and consent. (See National Power
recorded parti-cipation in the case was on July 15 and 27, 1979 when he Corporation Supervisors' Union vs. National Power Corporation, L-28805,
filed a motion for reconsideration and a supplemental motion. No action August 10, 1981, 106 SCRA 556). Other provisions of the Labor Code
was taken on said motions animated by the same intention are the following:

Even before the formalization of the CBA on June 3, 1980, Saavedra on "ART. 242. Rights and conditions of membership in a labor organization. -
March 24, 1980 filed in the case his notice of attorney's lien. On May 20, The following are the rights and conditions of membership in a labor
1980, the bank's vice-president in a reply to the letter of the union organization:
president stated that he had serious doubts about paying the attorney's
fees

The union officials requested the bank to with-hold around P345,000 out "(n) No special assessment or other extraordinary fees may be levied
of the total benefits as ten percent attorney's fees of Saavedra. At first, the upon the members of a labor organization unless authorized by a written
bank interposed no objection to the request in the interest of harmonious resolution of a majority of all the members at a general membership
labor-management relations meeting duly called for the purpose. The secretary of the organization
shall record the minutes of the meeting including the list of all members
For nearly a year, the Office of the President in four resolutions wrestled present, the votes cast, the purpose of the special assessment or fees and
with the propriety of Saavedra's ten percent attorney's fees. In a the recipient of such assessment or fees. The record shall be attested to
resolution dated May 29, 1980, Presidential Executive Assistant Jacobo C. lay the president;
Clave refused to intervene in the matter. He ruled that the payment of
attorney's fees was a question that should be settled by the union and its "(o) Other than for mandatory activities under the Code, no special
lawyer themselves. assessment, attorney's fees, negotiation fees or any other extraordinary
fees may be checked off from any amount due an employee without an
then, he "clarified" that ruling in a second resolution wherein he directed individual written authorization duly signed by the employee. The
that the attorney's fees may be deducted from the total benefits and paid authorization should specifically state the amount, purpose and
to Saavedra in accordance with the following provisions of the Labor beneficiary of the deduction; and
Code:
There is no doubt that lawyer Saavedra is entitled to the payment of his
fees but article 222 ordains that union funds should be used for that
purpose. The amount of P345, 000 does not constitute union funds. It is
"Art. 111. Attorney's fees. - (a) In cases of unlawful withholding of wages money of the employees. The union, not the employees, is obligated to
the culpable party may be assessed attorney's fees equivalent to ten Saavedra.
percent of the amount of wages recovered.

CONSOLIDATED WORKERS UNION v. CIR


"(b) It shall be unlawful for any person to demand or accept, in any
judicial or administrative proceedings for the recovery of wages, (UNION-MEMBER RELATIONS: Major Policy Matters: Deposit Funds)
attorney's fees which exceed ten percent of the amount of wages
recovered." Facts: The herein respondent, Coto Labor Union, composed of workers in
the mines operated by the Benguet Consolidated, Inc., in Coto, Masinloc,
Zambales, filed with the Court of Industrial Relations on 3 March 1964 a
complaint for unfair labor practice against the said company and the
Article 111 is implemented in Rule VIII, Book III of the Implementing herein petitioner, Consolidated Workers Union, a registered labor union
Rules and Regulations as follows: whose members are also workers of the company

alleging inter alia the following:


"Sec. 11. Attorney's fees. - Attorney's fees in any judicial or administrative
proceedings for the recovery of wages shall not exceed 10% of the
amount awarded. The fees may be deducted from the total amount due "1. That complainant is a labor organization duly registered with the
the winning party." Department of Labor under Reg. No. 1450-IP ....................................;

Presidential Executive Assistant Clave should have noticed that article "2. ..................................................;
111 refers to a proceeding for the recovery of wages and not to CBA
negotiations. The two are different or distinct proceedings. Not satisfied "3. ..................................................;
with the clarificatory resolution, Clave issued a third resolution wherein
he held that it is the legal obligation of the bank to turn over to the union "4. That on several occasions in July, 1963 and before August 11, 1963
treasurer ten percent of the award as Saavedra's fees. respondent Norberto Corpus (liaison officer of the company), told
Marciano Cruz, then a candidate for president of complainant union, not
to proceed with the complainant union's election on August 11, 1963, join
the res-pondent union and not to run against Juan Calbone of respondent
Issue: WON that attorney's fees may be charged against union funds? union and promised special benefits and privileges to Marciano Cruz if
the latter will agree;

"5. That during the second and third week of September, October and
Held: We hold that, under the circumstances, the Office of the President November, 1963 and, again, on the third week of December, 1963
had no jurisdiction to make an adjudication on Saavedra's attorney's fees. respondent Nor-berto Corpus uttered in front of many workers that the
The case was appealed with respect to the CBA terms and conditions, not complainant union is not recognized by the company and that the union
with respect to attorney's fees. Although the fees were a mere incident, compensation will be turned over to the respondent union,[1] causing by
nevertheless, the jurisdiction to fix the same and to order the payment such ut-terances, the members to be disgusted with complainant union,
thereof was outside the pale of Clave's appellate jurisdiction. He was right making many of them willing and did join the res-pondent union and thus
in adopting a hands-off attitude in his first resolution and holding that the helping said union;
payment of the fees was a question between the lawyer and the union.
"6. That on several instances in July, August, September, October and
November, 1963 respondent Pablo Pabilona (department supervisor of
the company) asked Marciano Cruz to stop the latter's union activi-ties union and order it disestablished, the result would be that petitioner
with a promise of giving special benefits and privi-leges as that granted would have to return all moneys and dues collected by it from the
by respondent company to Juan Calbone of respondent union; workers, since the union would have had no authority to represent the
workers. Because the moneys collected might be improperly spent by the
"7. That since the election of Marciano Cruz on August 11, 1963 as union time the disestablishment order is made, it was logical for the court
president respondent company, thru Leon E. Charlot (the company's below to order that such moneys be deposited in the meantime.
general superin-tendent), refused, in spite of demands, to deliver union
compensation to the complainant union in violation of the existing If the petitioner should need any funds to continue its existence and
collective bargaining agreement and instead, since the third week of operations until judgment on the disestablishment issue is rendered, its
December, 1963 turned over said amounts to the respondent union remedy would lie in applying to the Industrial Court to release, after due
thereby assisting the same and in order to discourage membership to the hearing, so much of the impounded funds as may be strictly necessary for
com-plainant union; the purpose, and not to seek the invalidation of the appealed order.

"8. That respondent Rodrigo Ebuengan (the company's treasurer), upon It was error for the labor court not to require the complainant-movant,
the instruction of Leon E. Charlot and Juan Calbone refused, in spite of Coto Labor Union (herein respondent), to file a bond for the issuance of
demands, to turn over to the complainant union the union properties and preliminary injunction. The posting of a bond under Section 4 of Rule 58
records in his possession; of the Rules of Court is a condition sine qua non in order that the writ of
preliminary injunction may issue (Siva, et al. vs. Reyes, et al., 83 Phil. 416;
"9. That on or after August 16, 1963 Leon E. Charlot threatened Marciano Villarosa vs. Teodoro, 100 Phil. 24).
Cruz with dismissal because the latter asked why the former interfered
with union matters by instructing Rodrigo Ebuengan not to turn over the "The justice of this requirement of bond as a condition to the issuance of
union properties to complainant union and on the same occasion the 'preliminary injunction is beyond question and the Rules of Court
respondent Norberto Corpus told Mar-ciano Cruz to stop his union makes its compliance necessary whether the injunction sought is the
activities in order to be steady in his job." main objective or merely an auxiliary thereto. It has been said that, an
undertaking is the price of injunction and in the case of Tucker v. New
and praying that the respondents therein be declared guilty of unfair Brunswick Trading Co., 44 Ch. D. 249, 253, it was held that 'if a man gets
labor practice; that the Consolidated Workers Union be ordered dis- the injunction he must pay the price.'" (Commentaries & Jurisprudence on
established, as company-dominated, and ordering delivery of union Injunction, Laureta & Nolledo, pages 58-59)
properties and records to the complainant union.

The respondents moved to dismiss on the ground of lack of legal capacity


to sue; upon denial thereof, they filed their answers.

Almost a year after the filing of the complaint, complainant Coto Labor Rodriguez v. DIR
Union filed, on 27 February 1965, an urgent motion for preliminary
injunction to enjoin Benguet Consolidated Inc., and Consolidated Workers (UNION-MEMBER RELATIONS: Major Policy Matters: Union dues)
Union from negotiating and concluding, between themselves, a new
collective bargaining agreement; to enjoin the corporation from turning FACTS:
over to the latter union, and for the latter to receive from the former, the
1. The consolidated cases involve disputes among employees of the
dues collected from the workers, and from disbursing said dues; and for
Philippines Long Distance Telephone Company (PLDT), who are
the deposit into court of all dues col-lected or which may thereafter be
members of the same union, the Free Telephone Workers Union (FTWU).
collected. In the said motion, complainant - movant union alleged its
willingness and ability to post a bond therefor.

On 19 March 1965, Consolidated Workers Union filed, in accordance with 2. The disputes concern the validity of the general elections for union
its reservation, a Supplementary Agreement, dated 15 April 1964, officers in 1986, and the increase of union dues adopted and put into
entered into between the company and the said union, purporting to be effect by the incumbent officers subsequent to said elections.
"formerly the Coto Labor Union", and the agreement being one to
supplement the collective bargaining agreement then existing between
Coto Labor Union and the company since 8 April 1959 and containing a
check-off provision in favor of Coto Labor Union. This supplementary 3. Assailed by the petitioners in this case are (1) the decision of the
agreement provided, among other stipulations, that the employees who Director of Labor Relations (BLR) annulling the elections of officers of the
have been paying union compensation through payroll de-ductions on the labor union, the Free Telephone Workers Union (FTWU), and (2) the
date of effectivity of said agreement "shall be considered as having resolution denying their motion for reconsideration of the decision.
authorized the COMPANY to pay the said amount to the CONSOLIDATED
WORKERS UNION (formerly Coto Labor Union) as Union Dues". The
Consolidated Workers Union also filed a Consent Election Agreement
executed on 28 Sept-ember 1964 (after the initial charge was filed) by 4. The union's by-laws provide for the election of officers every three (3)
and between Coto Labor Union, Consolidated Workers Union and years, in the month of July.
Benguet Consolidated, Inc., providing for the holding of an election, under
the supervision of the Bureau of Labor Relations, to determine the sole
collective bargaining agent of the workers, together with the minutes and
5. Pursuant thereto, the union's Legislative Council set the provincial
tally sheet of the election thus held on 3 October 1964, showing that the
elections for its officers on July 14 to 18, 1986, and those for Metro Manila
majority of the votes cast by secret ballot favored the petitioner,
on July 25, 1986.
Consolidated Workers Union.

6. The same Council also quite drastically raised the fees for the filing of
Issue: WON deposit of the union dues were valid?
certificates of candidates which had therefore ranged from P75.00 to
P100.00.

Held: here is no merit to the argument of herein petitioner that the order
deprives it of property without due process of law, for it assumes that the
7. The filing fee for each candidate for president of the labor
union dues ordered to be deposited are its property, which is precisely in
organization was increased to P3,000; that for each candidate for vice-
issue in the main case. Moreover, as pointed out in the appealed order, if
president, secretary general, treasurer and auditor, to P2,000.00; and that
the Industrial Court, after hearing, should ultimately find that the
Consolidated Workers Union is company dominated or is a company
for assistant secretary, assistant treasurer and assistant auditor, to
P1,000.00 each.
3. Yes, the resolution increasing the union dues must therefore be struck
down, as illegal and void, arbitrary and oppressive.

ISSUES Yes, because the respondent Director found that the resolution of the
union's Legislative Council to this effect does not bear the signature of at
1. Whether or not there is any grave abuse of discretion tainting the least two-thirds (2/3) of the members of the Council, contrary to the
adjudgment of respondent Director of Labor Relations that the general requirement of the union constitution and by-laws; and that proof is
elections for union officers held in 1986 were attended by grave wanting of proper ratification of the resolution by a majority of the
irregularities, rendering the elections invalid general union membership at a plebiscite called and conducted for that
purpose, again in violation of the constitution and by-laws. The resolution
2. Whether or not the Matter of 30%-Support for Complaints for increasing the union dues must therefore be struck down, as illegal and
Violations of Union Membership Rights was mandatory void, arbitrary and oppressive. The collection of union dues at the
increased rates must be discontinued; and the dues thus far improperly
3. Whether or not the resolution increasing the union dues must
collected must be refunded to the union members or held in trust for
therefore be struck down, as illegal and void
disposition by them in accordance with their charter and rules, in line
with this Court's ruling in a parallel situation, viz:

RULING: ... All amounts already collected must be credited accordingly in favor of
the respective members either for their future legal dues or other
assessments or even delinquencies, if any. And if this arrangement
regarding the actual refund of what might be excessive dues is not
1. No, there is no grave abuse of discretion on the part of respondent acceptable to the majority of the members, the matter may be decided in
Director of Labor Relations. a general meeting called for the purpose.

No, because a review of the record fails to disclose any grave abuse of DISPOSITIVE: WHEREFORE, in G.R. Nos. 76579-82, the petition for
discretion tainting the adjudgment of respondent Director of Labor certiorari is DISMISSED, no grave abuse of discretion or other serious
Relations that the general elections for union officers held in 1986 were error having been shown in the decision of the respondent Director of
attended by grave irregularities, rendering the elections invalid. That Labor Relations, said decision — ordering the holding of new elections
finding must thus be sustained. for officers of the Free Telephone Worker Union — being on the contrary
in accord with the facts and the law.

2.No, pursuant to Article 242 of the Labor Code.

DOCTRINE: It goes without saying that free and honest elections are
No, because the respondent Director's ruling, however, that the assent of indispensable to the enjoyment by employees and workers of their
30% of the union membership, mentioned in Article 242 of the Labor constitutionally protected right to self-organization. That right "would be
Code, was mandatory and essential to the filing of a complaint for any diluted if in the choice of the officials to govern ... (union) affairs, the
violation of rights and conditions of membership in a labor organization election is not fairly and honestly conducted," and the labor officers
(such as the arbitrary and oppressive increase of union dues here concerned and the courts have the duty "to see to it that no abuse is
complained of), cannot be affirmed and will be reversed. committed by any official of a labor organization in the conduct of its
affairs.

The very article relied upon militates against the proposition. It states
that a report of a violation of rights and conditions of membership in a Gabriel, et al v. Secretary of Labor
labor organization maybe made by "(a)t least thirty percent (30%) of all
the members of a union or any member or members specially concerned." (UNION-MEMBER RELATIONS: Major Policy Matters: Source of
Payment and Special Assessment)

The use of the permissive "may" in the provision at once negates the
notion that the assent of 30% of all the members is mandatory. More FACTS: Petitioners compromise the Executive Board of Solidbank union,
decisive is the fact that the provision expressly declares that the report the collective bargaining agent for the Solidbank Corporation. Private
may be made, alternatively by "any member or members specially respondents are members of said union. The union’s EB decided to retain
concerned." the services of their counsel in connection with negotiations for a new
CBA. A general membership meeting was called where majority of union
members approved a resolution confirming the decision to engage the
services of the union’s counsel, Atty. Lacsina. The resolution provided
And further confirmation that the assent of 30% of the union members is that 10% of the total economic benefits that may be secured be given to
not a factor in the acquisition of jurisdiction by the Bureau of Labor the counsel at attorney’s fees. Also it contained an authorization for
Relations is furnished by Article 226 of the same Labor Code, which Solidbank Corporation to check-off said attorney’s fees the first lump sum
grants original and exclusive jurisdiction to the Bureau, and the Labor of payment of benefits under the new CBA. Private respondents issued a
Relations Division in the Regional Offices of the Department of Labor, complaint for illegal deduction.
over "all inter-union and intra-union conflicts, and all disputes,
grievances or problems arising from or affecting labor management
relations," making no reference whatsoever to any such 30 % support
requirement.

ISSUE: W/N the union may check-off attorney’s fees.

Indeed, the officials are given the power to act "on all inter-union and
intra-union conflicts (1) "upon request of either or both parties" as well
as (2) "at their own initiative."
HELD: No. Article 241 has 3 requisites for the validity of the special
assessment for union’s incidental expenses, attorney’s fees and
representation expenses. They are: The Petition is granted.

1. authorization by a written resolution of majority of all the members at 1. YES.


the general membership meeting called for the purpose.

2. secretary’s record of the minutes of the meeting


It is clear from Article 241 (o) of the Labor Code that attorney’s fees may
3. individual written authorization for check-off duly signed by the not be deducted or checked off from any amount due to an employee
employees concerned. without his written consent except for mandatory activities under the
Code. A mandatory activity is a judicial process of settling dispute laid
Such requirements were not complied with as there were no individual down by the law. Here, the amicable settlement entered into by the
written check-off authorizations; thus, the employer cannot legally management and the union cannot be considered as a mandatory activity.
deduct thus the assessment. The union should be made to shoulder the Although, the union filed a claim for ECOLA and other benefits before the
expenses incurred for the services of a lawyers and accordingly, Ministry of Labor, however, this case never reached its conclusion in view
reimbursement should be charged to the union’s general fund or account. of the parties’ agreement.
No deduction can be made from the salaries of the concerned employees
other than those mandated by law.

Vengco vs Trajano Timbungco maintains that the "Kapasiyahan" or resolutions gave him the
authority to make the deduction. However, contrary to his claim, the
FACTS: "Kapasiyahan" or resolutions did not confer upon him the power to
deduct the 10% despite the alleged approval of the majority of the union
workers. Thus, considered defective due to: (1) not only because it was
undated but also, the signatories were not apprised since the pages were
A compromise agreement was entered into by the management of the not captioned and did not state the very purpose for which it was
Anglo-American Tobacco Corporation and the Kapisanan ng Manggagawa prepared; (2) they never authorized, more so, they had no knowledge of
sa Anglo-American Tobacco Corporation (FOI-TAF) whereby the the deduction of 10% attorney’s fees until it was actually effected; and (3)
company will pay to the union members the sum of P150,000.00 for their the resolutions merely indicated their intention to get their claim and to
claims arising from the unpaid ECOLA and other benefits which were the inform Timbungco of such. Thus, absent such authority, Timbungco
subject of their complaint before the Ministry of Labor. Emmanuel should have turned over the money to the Union Treasurer.
Timbungo, union president, received the said amount in installments and
distributed the same among the union members.

Further, Book III, Rule VIII, Section II of the Implementing Rules which
dispenses with the required written authorization from the employees
Petitioners alleged that Timbungco was not authorized to receive the concerned does not apply in this case. This provision envisions a situation
money and the 10% attorney’s fees was deducted without their written where there is a judicial or administrative proceedings for recovery of
authorization. So, they demanded to Timbungco an accounting. However, wages. Upon termination of the proceedings, the law allows a deduction
Timbungco did not give in to their demand and alleged that (1) he is for attorney’s fees of 10% from the total amount due to a winning party.
authorized by virtue of resolutions signed by the majority of the union Unfortunately, in the present case, the fringe benefits received by the
members; (2) 10% attorney’s fees was in relation to the claim of the local union members consist of back payments of their unpaid ECOLA which
union for payment of ECOLA which is totally distinct and separate from are totally distinct from their wages and such payment was effected
the negotiation of the CBA; and (3) 10% deduction was in accordance through an amicable settlement and not in an administrative proceeding.
with Section II, Rule No. VIII, Book No. III of the Rules and Regulations
implementing the Labor Code and therefore, no authorization from the
union members is required. Thus, a complaint was filed against
Timbungco before the Ministry of Labor. Therefore, the submission by Timbungco of an accounting report on the
distribution of P150,000.00 is of no moment since the deduction of 10%
attorney’s fees is illegal and void for failure to comply with the
requirements of the law, thereby, his expulsion as union president is
Med-Arbiter issued an Order dismissing the complaint. On appeal to the justified.
BLR, Med-Arbiter’s Order was set aside, and ordered Timbungco to
render full accounting of the P150,000.00 and his expulsion as union
president. Timbungco filed a MR and consequently an order was issued
setting aside the BLR’s Order and ordered an audit examination of the 2. YES.
Books of Account of FOITAF. Petitioners then filed their MR but it was
denied.
Orders were issued with grave abuse of discretion.

ISSUE:

1. Whether Timbungco is guilty of illegally deducting 10% The controversy involves the propriety of the 10% deduction from the
attorney’s fees. fringe benefits of the union workers which they received from the
management in settlement of their claims. Such issue does not touch on
2. Whether BLR Director Cresenciano Trajano gravely abused his union dues or funds. Besides, the sum of P150,000.00 was not entered
discretion amounting to lack of jurisdiction in ordering examination of into the records of the Union since the money was not turned over by
union books instead of affirming his previous Order expelling Timbungco Timbungco to the Union Treasurer. Consequently, the said Orders have
from the union and ordering him to render an accounting of P150,000.00 no basis.
received by him.
Galvadores v. Trajano

(UNION-MEMBER RELATIONS: Major Policy Matters: CBA


RULING: negotiation)
Facts: Petitioner employees of the Philippine Long Distance Telephone collective bargaining agreement shall be imposed on any individual
Company (PLDT) and members of respondent Free Telephone Workers member of the contracting union; Provided, however, that attorney's fees
Union, now the Manggagawa ng Komunikasyon sa Pilipinas (simply may be charged against union funds in an amount to be agreed upon by
referred to hereinafter as the Union), question the legality of the check-off the parties. Any contract, agreement or arrangement of any sort to the
for attorney's fees amounting to P1M, more or less, of respondent Atty. contrary shall be null and void.
Jose C. Espinas (hereinafter referred to as "Respondent Counsel") from
the monetary benefits awarded to PLDT employees in a deadlocked While Article 242 of the same Code reads:
collective bargaining agreement negotiations between the PLDT and the
Union. Art. 242. Rights and conditions of membership in a labor organization.
The following are the rights and conditions of membership in a labor
Respondent Counsel has been the legal counsel of respondent Union since organization:
1964. For his services, he was hired on a case to case contingent fee basis.
When respondent receives a letter from the Union President stating that xxx xxx xxx
his services are needed and that the Union binds itself to compensate him
(o) Other than for mandatory activities under the Code, no special
with the amount of 10% of any improvement, with retroactive effect, of
assessment, attorney's fees, negotiation fees or any other extraordinary
the PLDT's last offer to the deadlock in CBA negotiations which they
fees may be checked off "from any amount due an employee without
know will result in a compulsory arbitration. PLDT's "last offer" referred
individual written authorization duly signed by the employee. The
to on the wage increases was: P230 for the first year of the proposed CBA;
P100 for the second year; and P90 for the third year authorization should specifically state the amount, purpose and
beneficiary of the deduction.
the Minister of Labor awarded across-the-board wage increases of P
The Omnibus Rules Implementing the Labor Code also provide that
330/month effective November 9, 1982; P155/month effective November
deductions from wages of the employees may only be made by the
9, 1983, and P155/month effective November 9, 1984, in addition to the
employer in cases authorized by law, including deductions for insurance
Christmas bonus of 1/2 month pay per employee effective December,
premiums advanced by the employer on behalf of the employees as well
1983, and other fringe benefits. As will be noted, there were
as union dues where the right to check-off is authorized in writing by the
improvements obtained from PLDT's "last offer."
individual employee himself.
On October 29, 1983, the Executive Board of the Union passed a
The provisions are clear. No check-offs from any amounts due employees
resolution requesting PLDT to deduct P115.00 per employee for the legal
services extended to the Union by respondent Counsel. may be effected without individual written authorizations duly signed by
the employee specifically stating the amount, purpose and beneficiary of
Petitioners initially numbering 600 and finally 5,258, filed a letter- the deduction. The required individual authorizations in this case are
complaint before the MOLE through their authorized representative, wanting. In fact, petitioner employees are vigorously objecting. The
petitioner Carlos Galvadores assailing the imposition of P130.00 (later question asked in the plebiscite, besides not being explicit, assumed that
corrected to P155.00) per employee as attorney's fees of respondents there was no dispute relative to attorney's fees.
counsel. Petitioners took the position that the attorney's fees of
Contrary to respondent Union's and Counsel's stand, the benefits
respondent counsel were not only unreasonable but also violative of
awarded to PLDT employees still formed part of the collective bargaining
Article 242(o) of the Labor Code; and that he deductions cannot given
negotiations although placed already under compulsory arbitration. This
legal effect by a mere Board resolution but needs the ratification by the
general membership of the Union. is not the "mandatory activity" under the Code which dispenses with
individual written authorizations for check-offs, notwithstanding its
the Union filed a Manifestation to the effect that about 6,067 members of "compulsory" nature. It is a judicial process of settling disputes laid down
the Union ratified the October 29, 1983 resolution of the legislative by law. Besides, Article 222(b) does not except a CBA, later placed under
council in a plebiscite called for that purpose. On the basis thereof, compulsory arbitration, from the ambit of its prohibition. The cardinal
Counsel moved for the payment of his legal fees under the September 7, principle should be borne in mind that employees are protected by law
1983 contract. from unwarranted practices that diminish their compensation without
their knowledge and consent.
Petitioners questioned the plebiscite on the ground that Question No. 2,
which reads:

Question No. 2. Do you approve of the use of P1 million (P500,000.00 to


be withdrawn from PECCI and another P500,000.00 from IBAA) from our
CONTINENTAL CEMENT CORPORATION LABOR UNION (NLU) v.
CBA negotiation fund together with the attorney's fees (P1 million) that
was collected and to be loaned to the MKP/FTWU as our counterpart of CONTINENTAL CEMENT CORP.
the seed money to start the housing program as agreed by the PLDT
management and our union panel and included in the award of the (UNION-MEMBER RELATIONS: Major Policy Matters: Union Officer
MOLE? Obligation)
was misleading and deceptive as it assumed that there was no dispute FACTS:
regarding the deduction of attorney's fees from the monetary benefits
awarded to PLDT employees.

On February 18, 1985, respondent Director of the Bureau of Labor • In 1975, the NLRC issued an award regarding the working
Relations dismissed petitioners' complaint. terms and conditions in the respondent’s establishment. But due to the
delay caused by disagreements, petitioner staged a strike for the
Issue: WON attorney’s fees can be part of CBA negotiation and can be immediate compliance of the said award.
deducted to salary of employees.
• It was, however, lifted after the private respondent agreed to
Held: NO. pay the disputed employee s' leaves in three installments.
Article 222(b) of the Labor Code provides: • Meanwhile, private respondent sought clarification from the
labor arbiter on whether a group of 91 workers who were unable to
Article 222. Appearance and Fees. complete 300 days of work within a 12-month period was entitled to
proportionate payment of vacation and sick leave benefits, in which it
xxx xxx xxx
ruled in favor of herein petitioner.
(b) No attorney's fees, negotiation fees or similar charges of any kind
• On Jun 12, 1976, when the date for the first installment arrived,
arising from any collective bargaining negotiations or conclusion of the
petitioner staged a strike.
• On the next day, the Minister of Labor issued an order thru the ALDOVINO vs NLRC
Director of the Bureau of Labor Relations, directing the striking workers
to resume work under the terms and conditions prevailing prior to the (UNION-MEMBER RELATIONS: Major Policy Matters: Employee
work stoppage. The order was served on the parties in the afternoon of Individual Rights)
the same date.
FACTS: Petitioner Anacleto G. Pimentel started work as a lay-out man on
• Only 11 out of the total work force of about 120 workers in one 25 April 1985 assigned at AG & P San Roque, Bauan, Batangas, while
shift reported for work and were admitted by the company. petitioner Gaudencio A. Aldovino was hired in June 1989 as an electrician
at the AG & P Batangas Marine and Fabrication Yard (BMFY) in Bauan,
• Petitioner filed a motion for reconsideration of the return-to- Batangas. Pimentel and Aldovino acquired the status of regular
work order or its suspension pending compliance by private respondent employees on 1 December 1990 and 1 February 1991, respectively, and
with the NLRC award in favor of petitioner. became members in good standing of the employees' union, herein
petitioner United Rank and File Association (URFA), then the recognized
• Some 110 striking workers did not return to work. and exclusive collective bargaining agent for all regular rank-and-file
Consequently, private respondent filed with the Department of Labor employees of AG & P. In 1994 Aldovino and Pimentel instituted separate
reports on the dismissal of those who failed to comply with the return-to- but similar complaints against AG & P for unfair labor practice, illegal
work order. layoff, illegal dismissal and non-payment of CBA increases and benefits.
They prayed for reinstatement with back wages, interests, CBA wage
• Three days later, the president of petitioner and 7 other increases, benefits and attorney's fees. The two (2) cases were thereafter
officers requested admission to work but were informed that their consolidated.
employment had been terminated by the company.
On 12 August 1994, finding that the complainants were illegally
• NLRC ruled for the respondent which was affirmed by the dismissed, Labor Arbiter Ernesto S. Dinopol rendered a decision[12] in
Minister of Labor. the two (2) cases ordering AG & P to reinstate Aldovino and Pimentel as
regular employees and to pay back wages and attorney's fees. He
ISSUES:
explained that the financial situation of AG & P was not bleak as was
(1) WON the strike staged by petitioner on June 12, 1976 until its lifting pictured in its position paper, which was why the extended temporary
was illegal; and layoff status of Aldovino and Pimentel was unjustified and "akin to illegal
dismissal." The Labor Arbiter however rejected the charge of unfair labor
(2) In the affirmative, whether or not the penalties meted out by the practice and the claims for damages and other monetary benefits for lack
NLRC to the Union officers and the members are warranted by the of evidence.
circumstances and the law.
AG & P appealed to the NLRC protesting that the issue of the validity of
HELD: (1) Yes, it is illegal. the temporary layoff had already been decided in its favor by a voluntary
arbitrator, hence, was res judicata. Aldovino and Pimentel also appealed,
PD 823 Sec 1, as amended, provides that all forms of strikes, picketing although partially, questioning the Labor Arbiter's computation of their
and lockouts are hereby strictly prohibited in vital industries. The Letter back wages and the denial of their claim for CBA increases and benefits.
of Instruction No. 368 of the President considers cement companies or
firms as vital industries. Resolving the appeal on 18 February 1995, the NLRC set aside the 12
August 1994 decision of the Labor Arbiter and agreed with AG & P that
Private respondent was engaged in the manufacture of cement which is the principle of res judicata was applicable in petitioners' case, citing the
no doubt a vital industry in which a strike or lockout is prohibited under decision of Voluntary Arbitrator Batino which upheld the validity of the
the foregoing aforestated decree. And even assuming that private 17 September 1991 temporary layoffs. It also alluded to its decision in
respondent was not engaged in a vital industry, the strike that was staged Revidad v. AG & P of Manilapromulgated on 14 July 1993which already
by petitioner was nonetheless illegal. It was not in connection with any established the law of the case. In its resolution of 30 March 1995, the
unresolved economic issue in collective bargaining which is the only NLRC denied reconsideration.
ground for which a lawful strike can be held.
ISSUE:
The issue between the petitioner and the private respondent at the time
of the strike concerned merely the implementation of an arbitration The NLRC misapplied Art. 286 of the Labor Code because at the time AG &
award of the NLRC. The petitioner had a remedy by applying for a writ of P asked petitioners if they were willing to extend their layoff status, there
execution to enforce that award. Its resort to a strike was without lawful was yet no resumption of operations in the particular work unit or area
basis. to which they were previously assigned.

(2) YES. The strikers in question did not only violate the no-strike policy Held:
of the state in regard to vital industries; instead, they repeatedly defied
the orders of the Director of Labor Relations and the Minister of Labor for The right of URFA as a legitimate labor union to represent its members is
them to return to work. Their dismissal was recommended by the labor expressly guaranteed under Art. 242 of the Labor Code.[19] This right,
arbiter. However, out of compassion, the NLRC and the Minister of Labor however, does not deprive its individual members of their concomitant
only suspended them. right to file a case in their own names, nor of their right to withdraw from
any case filed by the union in their behalf. More importantly, the
Petitioner then contends that the separation from work of the officers of individual member may seasonably exercise his option to withdraw from
the union is quite severe. The officers had the duty to guide their a case filed by his union if he does not want to be bound thereby. In
members to respect the law. Instead, they urged them to violate the law Philippine Land-Air-Sea Labor Union (PLASLU), Inc. v. CIR,[20] this Court
and defy the duly constituted authorities. Their responsibility is greater ruled that only those members of the petitioning union who did not
than that of the members. Their dismissal from the service is a just signify their intention to withdraw from the case before its trial and
penalty for their unlawful acts. judgment on the merits are bound by the outcome of the case. Since it has
not been shown that Aldovino and Pimentel withdrew from the case
Pursuant to Art. 242 (p) of the labor code, It is within the power of the undergoing voluntary arbitration, it stands to reason that both are bound
NLRC to order the removal of the officers of petitioner, by the decision rendered thereon. This obtaining, there is no doubting the
identity of parties between the arbitrated case and that brought by
The officers of petitioner misinformed the members and led them into petitioners before the Labor Arbiter.
staging an illegal strike. If the NLRC is to attain the objective of the Labor
Code to ensure a stable but dynamic and just industrial peace, the Petitioners are now barred by prior judgment from raising in this case
removal of undesirable labor leaders must be effected. the same issue of the legality of their layoffs. The test to determine
whether there is identity of causes of action is to ascertain whether the
same evidence necessary to sustain the second action would have been
sufficient to authorize a recovery in the first, even if the forms or nature
of the two actions be different. For NLRC to allow Aldovino and Pimentel Held: YES. It is true that disaffiliation from a labor union is not open to
to prove that they were illegally dismissed as a result of the extended legal objection. It is implicit in the freedom of association ordained by the
layoff period would be to relitigate the validity and reasonableness of the Constitution. But this Court has laid down the ruling that a closed shop is
retrenchment program of AG & P, a matter already resolved by Voluntary a valid form of union security, and such provision in a collective
Arbitrator Batino and likewise settled in Revidad. It is to the interest of bargaining agreement is not a restriction of the right of freedom of
the public that there should be an end to litigation by the same parties association guaranteed by the Constitution.
and their privies over a subject once fully and fairly adjudicated.Interest
republicae ut sit finis litium. In the case at bar, the Company and the Amigo Employees Union-PAFLU
entered into a Collective Bargaining Agreement with a union security
clause provided for in Article XII thereof which is a reiteration of the
same clause in the old CBA. The quoted stipulation for closed-shop is
Villiar v. Inciong clear and unequivocal and it leaves no room for doubt that the employer
is bound, under the collective bargaining agreement, to dismiss the
(UNION-MEMBER RELATIONS: Major Policy Matters: Jurisdiction- employees, herein petitioners, for non- union membership. Petitioners
Exhaustion of Internal Remedies) became non-union members upon their expulsion from the general
membership of the Amigo Employees Union-PAFLU on March 15, 1977
Facts: Petitioners were members of the Amigo Employees Union-PAFLU, pursuant to the Decision of the PAFLU national president.
a duly registered labor organization which, at the time of the present
dispute, was the existing bargaining agent of the employees in private The Court rejected petitioners' theory that their expulsion was not valid
respondent Amigo Manufacturing, Inc. (hereinafter referred to as upon the grounds adverted to earlier in this Decision. That PAFLU had the
Company). The Company and the Amigo Employees Union-PAFLU had a authority to investigate petitioners on the charges filed by their co-
collective bargaining agreement governing their labor relations, which employees in the local union and after finding them guilty as charged, to
agreement was then about to expire on February 28, 1977. Within the last expel them from the roll of membership of the Amigo Employees Union-
sixty (60) days of the CBA, events transpired giving rise to the present PAFLU is clear under the constitution of the PAFLU to which the local
dispute. union was affiliated. And pursuant to the security clause of the new CBA,
reiterating the same clause in the old CBA, PAFLU was justified in
Upon written authority of at least 30% of the employees in the company, applying said security clause.
including the petitioners, the Federation of Unions of Rizal (hereinafter
referred to as FUR) filed a petition for certification election with the Med- Petitioners insist that their disaffiliation from PAFLU and filing a petition
Arbiter's Office, of the Ministry of Labor and Employment. The petition for certification election are not acts of disloyalty but an exercise of their
was, however, opposed by the Philippine Association of Free Labor right to self-organization. They contend that these acts were done within
Unions (hereinafter referred to as PAFLU) with whom, as stated earlier, the 60-day freedom period when questions of representation may freely
the Amigo Employees Union was at that time affiliated. PAFLU's be raised. Under the peculiar facts of the case, We find petitioners'
opposition cited the "Code of Ethics" governing inter-federation disputes insistence untenable.
among and between members of the Trade Unions Congress of the
Philippines (hereinafter referred to as TUCP). Consequently, the Med- In the first place, had petitioners merely disaffiliated from the. Amigo
Arbiter indorsed the case to TUCP for appropriate action but before any Employees Union-PAFLU, there could be no legal objections thereto for it
such action could be taken thereon, the petitioners disauthorized FUR was their right to do so. But what petitioners did by the very clear terms
from continuing the petition for certification election for which reason of their "Sama-Samang Kapasiyahan" was to disaffiliate the Amigo
FUR withdrew the petition. The same employees who had signed the Employees Union-PAFLU from PAFLU, an act which they could not have
petition filed by FUR signed a joint resolution disaffiliating from PAFLU. done with any effective consequence because they constituted the
minority in the Amigo Employees Union-PAFLU.
Dolores Villar, representing herself to be the authorized representative of
the Amigo Employees Union, filed a petition for certification election in Extant from the records is the fact that petitioners numbering ten (10),
the Company. The Amigo Employees Union-PAFLU intervened and moved were among the ninety-six (96) who signed the "Sama-Samang
for the dismissal of the petition for certification election filed by Villar, on Kapasiyahan" whereas there are two hundred thirty four (234) union
the ground, among others that Villar had no legal personality to sign the members in the Amigo Employees Union-PAFLU. Hence, petitioners
petition since she was not an officer of the union nor is there factual or constituted a small minority for which reason they could not have
legal basis for her claim that she was the authorized representative of the successfully disaffiliated the local union from PAFLU. Since only 96
local union. Med-Arbiter dismissed the petition filed by Villar, which wanted disaffiliation, it can be inferred that the majority wanted the
dismissal is still pending appeal before BLR. Amigo Employees Union- union to remain an affiliate of PAFLU and this is not denied or disputed
PAFLU called a special meeting of its general membership. A Resolution by petitioners. The action of the majority must, therefore, prevail over
was thereby unanimously approved which called for the investigation by that of the minority members
the PAFLU national president, of all of the petitioners and one Felipe
Manlapao, for continuously maligning the union spreading false A closed-shop is a valid form of union security, and a provision therefor in
propaganda that the union officers were merely appointees of the a collective bargaining agreement is not a restriction of the right of
management; and for causing divisiveness in the union. freedom of association guaranteed by the Constitution. (Manalang, et al.
vs. Artex Development Co., Inc., et al., L-20432, October 30, 1967, 21 SCRA
PAFLU formed a Trial Committee to investigate the local union's charges 561). Where in a closed-shop agreement it is stipulated that union
against the petitioners for acts of disloyalty. PAFLU and the Company members who cease to be in good standing shall immediately be
concluded a new CBA which also reincorporated the same provisions of dismissed, such dismissal does not constitute an unfair labor practice
the existing CBA, including the union security clause. PAFLU President exclusively cognizable by the Court of Industrial Relations.
rendered a decision finding the petitioners guilty of the charges. PAFLU
demanded the Company to terminate the employment of the petitioners
pursuant to the security clause of the CBA. Acting on PAFLU's demand,
the Company informed PAFLU that it will first secure the necessary Kapisanan ng Manggagawang Pinagyakap (KMP) vs. Trajano
clearances to terminate petitioners. PAFLU requested the Company to put
petitioners under preventive suspension pending the application for said (UNION-MEMBER RELATIONS: Major Policy Matters: Remedies)
clearances to terminate the petitioners. The Company filed the request
FACTS: On June 30, 1981, a written request for accounts examination of
for clearance to terminate the petitioners before DOLE which was
the financial status of KMP Labor Union, the existing labor union at
granted. DOLE Secretary Inciong denied the appeal, hence, this petition
Franklin Baker Company in San Pablo City, was filed by private
for review.
respondent Silvestre and 13 other employees, who are members of the
union. Acting on said request, Union Account Examiner Vicedo of the
MOLE conducted the investigation and thereafter submitted a report.
Issue: WON the termination of petitioners are proper. Based on the revelations, private respondents filed with the Regional
Office QC, MOLE, a petition for the expulsion of the union officers.They
committed gross violation of the Labor Code, specifically pars. (a), (b), (g),
(h), (j), and (k) of Article 242; and, the constitution and by-laws of the 3. Assailed by the petitioners in this case are (1) the decision of the
union, Sections 6 and 7 Director of Labor Relations (BLR) annulling the elections of officers of the
labor union, the Free Telephone Workers Union (FTWU), and (2) the
Union Officers: Denied imputation and argued that the disallowed resolution denying their motion for reconsideration of the decision.
expenditures were made in good faith; that the same conduced the
benefit of members, they are willing to reimburse the same from their
own personal funds,they should not be held accountable for the non-
production of books of accounts of the Union for years 1977, 1978, and 4. The union's by-laws provide for the election of officers every three (3)
1979 because they were not the officers then and not one of the former years, in the month of July.
officers of the Union had turned over to them the records, And non-
ratification of the constitution and by-laws of the Union and the non-
segregation of the Union funds occurred before they became officers and
that they have already been correcting the same. 5. Pursuant thereto, the union's Legislative Council set the provincial
elections for its officers on July 14 to 18, 1986, and those for Metro Manila
Med-Arbiter Cabibihan ordered the holding of a referendum, to be on July 25, 1986.
conducted under the supervision of BLR. Petitioners appealed the order
to respondent Trajano of BLR the disallowed expenditures of P1,278.00
were made in good faith and not used for the personal benefit of herein 6. The same Council also quite drastically raised the fees for the filing of
union officers but, instead, contributed to the benefit of the members they certificates of candidates which had therefore ranged from P75.00 to
were elected in 1980 only and, therefore, they could not be made P100.00.
responsible for the omissions of their predecessors who failed to turn
over union records for the questioned period there would be a general
election on Oct. 4, 1982, at which time, both the election and the desired
referendum could be undertaken to determine the membership at 7. The filing fee for each candidate for president of the labor
minimum expense they prayed that resolution on the issue be held in organization was increased to P3,000; that for each candidate for vice-
abeyance.Respondents claimed that Med-Arbiter erred in calling a president, secretary general, treasurer and auditor, to P2,000.00; and that
referendum to decide issue; the appropriate action should be the for assistant secretary, assistant treasurer and assistant auditor, to
expulsion of union officers. Trajano dismissed both appeals and affirmed P1,000.00 each.
in toto the order of Med-Arbiter

ISSUE: Whether or not union officers were guilty of the alleged acts
imputed against them thus expulsion was proper ISSUES

HELD: NO 1. Whether or not there is any grave abuse of discretion tainting the
adjudgment of respondent Director of Labor Relations that the general
If herein union officers (also petitioners) were guilty of the alleged acts elections for union officers held in 1986 were attended by grave
imputed against them, said public respondent pursuant to Article 242 of irregularities, rendering the elections invalid
the New Labor Code and in the light of Our ruling in Duyag vs. Inciong, 98
SCRA 522, should have meted out the appropriate penalty on them, i.e., to 2. Whether or not the Matter of 30%-Support for Complaints for
expel them from the Union, as prayed for, and not call for a referendum to Violations of Union Membership Rights was mandatory
decide the issue; the alleged falsification and misrepresentation of herein
union officers were not supported by substantial evidence. The fact that 3. Whether or not the resolution increasing the union dues must
they disbursed the amount of P1,278.00 from Union funds and later on therefore be struck down, as illegal and void
was disallowed for failure to attach supporting papers thereon did not of
itself constitute falsification and/or misrepresentation. The expenditures
appeared to have been made in good faith and the amount spent for the
RULING:
purpose mentioned in the report, if concurred in or accepted by the
members, are reasonable; and the repudiation of both private
respondents to the highly sensitive position of auditor at the October 4,
1982 election, is a convincing manifestation and demonstration of the 1. No, there is no grave abuse of discretion on the part of respondent
union membership's faith in the herein officers' leadership on one hand Director of Labor Relations.
and a clear condonation of an act they had allegedly committed.

No, because a review of the record fails to disclose any grave abuse of
Rodriguez v. CIR discretion tainting the adjudgment of respondent Director of Labor
Relations that the general elections for union officers held in 1986 were
(UNION-MEMBER RELATIONS: Major Policy Matters: Invalidity) attended by grave irregularities, rendering the elections invalid. That
finding must thus be sustained.

Topic: Enforcement and Remedies – Intra-union Disputes; Jurisdiction;


Procedure and Sanctions 2.No, pursuant to Article 242 of the Labor Code.
FACTS:

1. The consolidated cases involve disputes among employees of the No, because the respondent Director's ruling, however, that the assent of
Philippines Long Distance Telephone Company (PLDT), who are 30% of the union membership, mentioned in Article 242 of the Labor
members of the same union, the Free Telephone Workers Union (FTWU). Code, was mandatory and essential to the filing of a complaint for any
violation of rights and conditions of membership in a labor organization
(such as the arbitrary and oppressive increase of union dues here
2. The disputes concern the validity of the general elections for union complained of), cannot be affirmed and will be reversed.
officers in 1986, and the increase of union dues adopted and put into
effect by the incumbent officers subsequent to said elections.
The very article relied upon militates against the proposition. It states
that a report of a violation of rights and conditions of membership in a
labor organization maybe made by "(a)t least thirty percent (30%) of all
the members of a union or any member or members specially concerned."

Sugbuanon v Laguesma
The use of the permissive "may" in the provision at once negates the
notion that the assent of 30% of all the members is mandatory. More (UNION AFFILIATION: LOCAL AND PARENT RELATIONS: Affiliation:
decisive is the fact that the provision expressly declares that the report Purpose of; Nature of Relations)
may be made, alternatively by "any member or members specially
concerned."
FACTS: Petitioner Sugbuanon Rural Bank, Inc., (SRBI, for brevity) is a
duly-registered banking institution with principal office in Cebu City and
And further confirmation that the assent of 30% of the union members is a branch in Mandaue City. Private respondent SRBI Association of
not a factor in the acquisition of jurisdiction by the Bureau of Labor Professional, Supervisory, Office, and Technical Employees Union
Relations is furnished by Article 226 of the same Labor Code, which (APSOTEU) is a legitimate labor organization affiliated with the Trade
grants original and exclusive jurisdiction to the Bureau, and the Labor Unions Congress of the Philippines (TUCP).1âwphi1.nêt
Relations Division in the Regional Offices of the Department of Labor,
over "all inter-union and intra-union conflicts, and all disputes,
grievances or problems arising from or affecting labor management
On October 8, 1993, the DOLE Regional Office in Cebu City granted
relations," making no reference whatsoever to any such 30 % support
requirement. Certificate of Registration No. R0700-9310-UR-0064 to APSOTEU-TUCP,
hereafter referred to as the union.

Indeed, the officials are given the power to act "on all inter-union and
On October 26, 1993, the union filed a petition for certification election of
intra-union conflicts (1) "upon request of either or both parties" as well
as (2) "at their own initiative." the supervisory employees of SRBI. It alleged, among others, that: (1)
APSOTEU-TUCP was a labor organization duly-registered with the Labor
Department; (2) SRBI employed 5 or more supervisory employees; (3) a
majority of these employees supported the petition: (4) there was no
3. Yes, the resolution increasing the union dues must therefore be struck existing collective bargaining agreement (CBA) between any union and
down, as illegal and void, arbitrary and oppressive. SRBI; and (5) no certification election had been held in SRBI during the
past 12 months prior to the petition.

Yes, because the respondent Director found that the resolution of the
union's Legislative Council to this effect does not bear the signature of at On October 28, 1993, the Med-Arbiter gave due course to the petition.
least two-thirds (2/3) of the members of the Council, contrary to the The pre-certification election conference between SRBI and APSOTEU-
requirement of the union constitution and by-laws; and that proof is TUCP was set for November 15, 1993.
wanting of proper ratification of the resolution by a majority of the
general union membership at a plebiscite called and conducted for that
purpose, again in violation of the constitution and by-laws. The resolution
On November 12, 1993, SRBI filed a motion to dismiss the union’s
increasing the union dues must therefore be struck down, as illegal and
petition. It sought to prevent the holding of a certification election on two
void, arbitrary and oppressive. The collection of union dues at the
grounds. First, that the members of APSOTEU-TUCP were in fact
increased rates must be discontinued; and the dues thus far improperly
managerial or confidential employees.
collected must be refunded to the union members or held in trust for
disposition by them in accordance with their charter and rules, in line
with this Court's ruling in a parallel situation, viz:
ISSUES:
... All amounts already collected must be credited accordingly in favor of
the respective members either for their future legal dues or other
assessments or even delinquencies, if any. And if this arrangement
regarding the actual refund of what might be excessive dues is not (1) Whether or not the members of the respondent union are managerial
acceptable to the majority of the members, the matter may be decided in employees and/or highly-placed confidential employees, hence
a general meeting called for the purpose. prohibited by law from joining labor organizations and engaging in union
activities.

DISPOSITIVE: WHEREFORE, in G.R. Nos. 76579-82, the petition for


certiorari is DISMISSED, no grave abuse of discretion or other serious (2) Whether or not the Med-Arbiter may validly order the holding of a
error having been shown in the decision of the respondent Director of certification election upon the filing of a petition for certification election
Labor Relations, said decision — ordering the holding of new elections by a registered union, despite the petitioner’s appeal pending before the
for officers of the Free Telephone Worker Union — being on the contrary DOLE Secretary against the issuance of the union’s registration.
in accord with the facts and the law.

RULING:

DOCTRINE: It goes without saying that free and honest elections are
indispensable to the enjoyment by employees and workers of their (1) Petitioner’s explanation does not state who among the employees has
constitutionally protected right to self-organization. That right "would be access to information specifically relating to its labor to relations policies.
diluted if in the choice of the officials to govern ... (union) affairs, the Even Cashier Patricia Maluya, who serves as the secretary of the bank’s
election is not fairly and honestly conducted," and the labor officers Board of Directors may not be so classified.
concerned and the courts have the duty "to see to it that no abuse is
committed by any official of a labor organization in the conduct of its
affairs.
Confidential employees are those who (1) assist or act in a confidential any such agreement is entered into beyond six months, the parties shall
capacity, in regard (2) to persons who formulate, determine, and agree on the duration of retroactivity thereof. In case of a deadlock in the
effectuate management policies [specifically in the field of labor renegotiation of the collective bargaining agreement, the parties may
relations].9 The two criteria are cumulative, and both must be met if an exercise their rights under this Code.
employee is to be considered a confidential employee — that is, the
confidential relationship must exist between the employee and his Petitioner argues that in granting retroactive effect to the enforceability
superior officer; and that officer must handle the prescribed of the CBA, public respondent committed an act contrary to the above
responsibilities relating to labor relations. provision of law, pointing out that the old CBA expired on July 30, 1990
and the questioned order was issued on January 28, 1991. Petitioner
theorizes that following Article 13 of the Civil Code which provides that
there are 30 days in one month, the questioned Order of January 28, 1991
Art. 245 of the Labor Code does not directly prohibit confidential was issued beyond the six-month period, graphically shown thus:
employees from engaging in union activities. However, under the
doctrine of necessary implication, the disqualification of managerial July 30, 1990 Expiration
employees equally applies to confidential employees. The confidential-
employee rule justifies exclusion of confidential employees because in the July 31 = 1 day
normal course of their duties they become aware of management policies
relating to labor relations. It must be stressed, however, that when the August 1-31, 1990 = 31 days
employee does not have access to confidential labor relations
September 1-30, 1990 = 30 days
information, there is no legal prohibition against confidential employees
from forming, assisting, or joining a union. October 1-31, 1990 = 31 days

November 1-30, 1990 = 30 days


(2) One of the rights of a legitimate labor organization under Article December 1-31, 1990 = 31 days
242(b) of the Labor Code is the right to be certified as the exclusive
representative of all employees in an appropriate bargaining unit for January 1-28, 1991 = 28 days
purposes of collective bargaining. Having complied with the
requirements of Art. 234, it is our view that respondent union is a —————————
legitimate labor union. Article 257 of the Labor Code mandates that a
certification election shall automatically be conducted by the Med-Arbiter TOTAL = 182 days
upon the filing of a petition by a legitimate labor organization.16 Nothing
is said therein that prohibits such automatic conduct of the certification (6 months and 2 days)
election if the management appeals on the issue of the validity of the
union’s registration. On this score, petitioner’s appeal was correctly Private respondent agrees with the Labor Secretary's view that Article
dismissed. 253-A of the Labor Code does not apply to arbitral awards such as those
involved in the instant case. According to private respondent, Article 253-
A of the Labor Code is clear and plain on its face as referring only to
collective bargaining agreements entered into by management and the
ST. LUKE’S MEDICAL CENTER, INC. VS. TORRES, certified exclusive bargaining agent of all rank-and-file employees therein
within six (6) months from the expiry of the old CBA.
(UNION AFFILIATION: LOCAL AND PARENT RELATIONS: Effect on
Legal Personality) ISSUE: Whether or not the CBA should be given retroactive effect.

FACTS: HELD: The effectivity of the Order of January 28, 1991, must retroact to
the date of the expiration of the previous CBA, contrary to the position of
Private respondent SLMCEA-AFW brought to the attention of petitioner petitioner. Under the circumstances of the case, Article 253-A cannot be
via a letter dated July 4, 1990 that the 1987-1990 was about to expire, property applied to herein case. As correctly stated by public respondent
and manifested in the process that private respondent wanted to renew in his assailed Order of April 12, 1991 dismissing petitioner's Motion for
the CBA. This development triggered round-table talks on which Reconsideration —
occasions petitioner proposed, among other items, a maximum across-
the-board monthly salary increase of P375.00 per employee, to which Anent the alleged lack of basis for the retroactivity provisions awarded,
proposal private respondent demanded a P1,500.00 hike or 50% increase we would stress that the provision of law invoked by the Hospital, Article
based on the latest salary rate of each employee, whichever is higher. 253-A of the Labor Code, speak of agreements by and between the
parties, and not arbitral awards . . .
A deadlock on issues, especially that bearing on across-the-board
monthly and meal allowances followed and to pre-empt the impending Therefore, in the absence of a specific provision of law prohibiting
strike as voted upon by a majority of private respondent's membership, retroactivity of the effectivity of arbitral awards issued by the Secretary
petitioner lodged the petition below. The Secretary of Labor immediately of Labor pursuant to Article 263 (g) of the Labor Code, such as herein
assumed jurisdiction and the parties submitted their respective involved, public respondent is deemed vested with plenary and
pleadings. discretionary powers to determine the effectivity thereof.

On January 28, 1991, public respondent Secretary of Labor issued the


Order now under challenge. Said Order contained a disposition on both
the economic and non-economic issues raised in the petition. One of the PAMBANSANG KAPATIRAN NG MGA ANAK PAWIS vs. SECRETARY OF
rulings in the order is the granting of the retroactive effect to the LABOR
enforceability of the CBA.
(UNION AFFILIATION: LOCAL AND PARENT RELATIONS: Supervisor -
Petitioner argues that the Order of January 28, 1991 is violative of Article Rank and File Union Affiliation: Rule on Affiliation)
253-A of the Labor Code, particularly its provisions on retroactivity. Said
Article pertinently provides:

xxx xxx xxx FACTS: The rank and file workers of Formey Plastic, Inc. (FORMEY),
formed a local union known as Pambansang Kapatiran ng mga Anak
Any agreement on such other provisions of the collective bargaining Pawis sa Formey Plastic (KAPATIRAN) under the auspices of the National
agreement entered into within six (6) months from the date of expiry of WorkersBrotherhood (NWB). They ratified their Constitution and By-
the term of such other provisions as fixed in the collective bargaining Laws on 4 April 1993.
agreement, shall retroact to the day immediately following such date. If
On 22 April 1993 KAPATIRAN filed a Petition for Certification Election respond to the return-to-work notice were sent termination letters dated
alleging that there was no existing andeffective CBA between FORMEY 17 May 1989.
and any union; neither was there any recognized union within the
company. FORMEY moved to dismiss the petition while Kalipunan ng - Issue
Manggagawang Pilipino (KAMAPI) intervened and likewise moved to
dismiss on the ground that there was already a duly registered CBA o Can company officials be held personally liable?
covering period Jan. 1,1992 to Dec. 31, 1996, therefore the contract bar
- Held
rule will apply. KAPATIRAN opposed both motion to dismiss claiming
that the CBA executed between FORMEY and KAMAPI was fraudulently o No, company officials cannot be held personally liable for
registered with the DOLE and that it was defective since what was damages on account of the employees’ dismissal because the employer
certified as bargaining agent was KAMAPI which as federation only corporation has a personality separate and distinct from its officers who
served as mere agent of the local union and without any legal personality merely acted as its agents
to sign in behalf of the latter. Med-Arbiter found that there is a valid and
existing CBA between FORMEY and KAMAPI which effectively barred the o However, the dismissal was invalidated in this case because of
filing for petition for certification election. KAPATIRAN appealed respondent company’s failure to accord petitioners with due process, that
imputing grave abuse of discretion to the Med-Arbiter in applying the is, notice and hearing prior to their termination.
contract bar rule. Secretary of Labor upheld the decision of Med-Arbiter.
KAPATIRAN filed a motion for reconsideration which was likewise o Also, said dismissal was invalidated because the reason relied
denied. upon by respondent Federation was not valid.

o Nonetheless, the dismissal still does not constitute unfair labor


practice.
ISSUES:
o It has come to the attention of this Court that the 30-day prior
1. Whether or not the petition for certification election was properly filed. notice requirement for the dismissal of employees has been repeatedly
violated and the sanction imposed for such violation enunciated in
2. Whether or not there was a valid CBA between FORMEY and KAMAPI. Wenphil Corporation vs. NLRC32 has become an ineffective deterrent.

o Thus, the Court recently promulgated a decision to reinforce


and make more effective the requirement of notice and hearing, a
HELD:
procedure that must be observed before termination of employment can
1. No, the petition for certification election was not properly filed. The be legally effected.
CBA entered into between FORMEYand KAMAPI was made effectively Jan.
o In Ruben Serrano vs. NLRC and Isetann Department Store (G.R.
1, 1992 and will expire Dec. 31, 1996. The petition forcertification
No. 117040, January 27, 2000), the Court ruled that an employee who is
election was filed on April 22, 1993 which was filed before the so-called
60-day freedomperiod dismissed, whether or not for just or authorized cause but without prior
notice of his termination, is entitled to full backwages from the time he
was terminated until the decision in his case becomes final, when the
dismissal was for cause; and in case the dismissal was without just or
2. Yes, the court affirmed that there was a valid CBA between FORMEY valid cause, the backwages shall be computed from the time of his
and KAMAPI. Art. 253-A of the dismissal until his actual reinstatement.

labor code provides that “no petition questioning the majority status of o In the case at bar, where the requirement of notice and hearing
the incumbent bargaining agent was not complied with, the aforecited doctrine laid down in the Serrano
case applies.
shall be entertained and no certification election shall be conducted by
the DOLE outside the 60-dayperiod immediately before the date of expiry o WHEREFORE, the Petition is GRANTED; the decision of the
of such 5 year term of the CBA. National Labor Relations Commission in Case No. NCR-00-09-04199-89 is
REVERSED and SET ASIDE

WHEREFORE, the petition is DENIED. The decision of the Secretary of


Labor and Employment dated 15 August1993 sustaining the order of the ATLAS LITHOGRAPHIC SERVICES INC., VS LAGUESMA
Med-Arbiter dated 31 May 1993 is AFFIRMED.
(UNION AFFILIATION: LOCAL AND PARENT RELATIONS: Supervisor -
Rank and File Union Affiliation: Rule on Affiliation)

Malayang Samahan v. Ramos, 326 SCRA 428 [G.R. No. 113907, Feb.
28, 2000]
FACTS:
- Facts
On July 16, 1990, the supervisory, administrative personnel, production,
o At bar is a Petition for Certiorari under Rule 65 of the Revised accounting and confidential employees of Atlas Lithographic Services Inc
Rules of Court to annul the decision of the National Labor Relations (ALSI) affiliated with Kaisahan ng Manggagawang Pilipinom a national
Commission in an unfair labor practice case instituted, by a local union labor organization. The local union adopted the name ALSI-SAPPACEA-
against its employer company and the officers of its national federation. KAMPIL, which shall hereafter refer to as the “supervisors” union.

o The official declaration of strike was at around 3:30 p.m. of 14


March 1989. The strike was attended with violence, force and
intimidation on both sides resulting to physical injuries to several Kampil-Katipunan filed on behalf of the “supervisors” union a petition for
employees, both striking and non-striking, and damage to company certification election so that it could be the sole and exclusive bargaining
properties. The employees who participated in the strike and allegedly agent of the supervisory employees. ALSI opposed the petition claiming
figured in the violent incident were placed under preventive suspension that under Art. 245 of the Labor Code, Kampil-Katipunana cannot
by the company. The company also sent return to-work notices to the represent the supervisory employees for collective bargaining purposes
home addresses of the striking employees thrice successively, on March because it also represents the rank-and-file employees’ union.
27, April 8 and April 31, 1989, respectively. However, only 261
employees were eventually accepted back to work. Those who did not
On September 18, 1990, the Med-Arbiter issued an order allowing the Article 212 (k) therein became eligible to form, to join or assist a rank-
certification election. ALSI appealed but such appeal was denied. Hence, and-file union.
this petition for certiorari.
A revision of the Labor Code undertaken by the bicameral Congress
ISSUE(S): brought about the enactment of Rep. Act No. 6715 in March 1989 in
which employees were reclassified into three groups, namely: (1) the
1. WON, under Art. 245 of the Labor Code, a local union of managerial employees; (2) supervisors; and (3) the rank and file
supervisory employees may be allowed to affiliate with a national employees. Under the present law, the category of supervisory employees
federation of labor organization of rank-and-file employees where such is once again recognized. Hence, Art. 212 (m) states:
federation represents its affiliates in the collective bargaining negotiation
with the same employer of the supervisors and in the implementation of (m) . . . Supervisory employees are those who, in the interest of the
the CBAs. employer, effectively recommend such managerial actions if the exercise
of such authority is not merely routinary or clerical in nature but requires
HELD: NO, supervisors are not prohibited from forming their own union. the use of independent judgment. . . .
What the law prohibits is their membership in a labor organization of
rank-and-file employees or their joining in a federation of rank-and-file The rationale for the amendment is the government's recognition of the
employees that includes the very local union which they are not allowed right of supervisors to organize with the qualification that they shall not
to directly join. join or assist in the organization of rank-and-file employees. The reason
behind the Industrial Peace Act provision on the same subject matter has
RATIO: been adopted in the present statute. The interests of supervisors on the
one hand, and the rank-and-file employees on the other, are separate and
ALSI’s arguments: distinct. The functions of supervisors, being recommendatory in nature,
are more identified with the interests of the employer. The performance
1. KAMPIL-KATIPUNAN already represents its rank-and-file
of those functions may, thus, run counter to the interests of the rank-and-
employees and, therefore, to allow the supervisors of those employees to file.
affiliate with the private respondent is tantamount to allowing the
circumvention of the principle of the separation of unions under Article This intent of the law is made clear in the deliberations of the legislators
245 of the Labor Code. on then Senate Bill 530 now enacted as Rep. Act No. 6715.
2. It further argues that the intent of the law is to prevent a single The definition of managerial employees was limited to those having
labor organization from representing different classes of employees with authority to hire and fire while those who only recommend effectively the
conflicting interests. hiring or firing or transfers of personnel would be considered as closer to
rank-and-file employees. The exclusion, therefore, of middle level
KAMPIL-KATIPUNAN’s arguments:
executives from the category of managers brought about a third
1. Despite affiliation with a national federation, the local union classification, the supervisory employees. These supervisory employees
does not lose its personality which is separate, and distinct from the are allowed to form their own union but they are not allowed to join the
national federation. [Adamson & Adamson vs. CIR (1984)] rank-and-file union because of conflict of interest (Journal of the Senate,
First Regular Session, 1987, 1988, Volume 3,
2. It maintains that Rep. Act No. 6715 contemplates the principle
laid down by this Court in the Adamson case interpreting Section 3 of p. 2245).
Rep. Act No. 875 (the Industrial Peace Act) on the right of a supervisor's
In terms of classification, however, while they are more closely identified
union to affiliate. The private respondent asserts that the legislature must
with the rank-and-file they are still not allowed to join the union of rank-
have noted the Adamson ruling then prevailing when it conceived the
and-file employees. The peculiar role of supervisors is such that while
reinstatement in the present Labor Code of a similar provision on the
they are not managers, when they recommend action implementing
right of supervisors to organize.
management policy or ask for the discipline or dismissal of subordinates,
DISCUSSION: they identify with the interests of the employer and may act contrary to
the interests of the rank-and-file.
The basis of the Adamson case is R.A. No. 875 (Industrial Peace Act)
where employees were classified into three groups, namely: 1) The Court agrees with ALSI’s contention that a conflict of interest may
managerial employees; 2) supervisors; and 3) rank-and-file employees. arise in the areas of discipline, collective bargaining and strikes. Members
Supervisors who were considered employees in relation to their of the supervisory union might refuse to carry out disciplinary measures
employer could join a union but not a union of rank-and-file employees. against their co-member rank-and-file employees. And also, in the event
of a strike, the national federation might influence the supervisors’ union
With the enactment in 1974 of the Labor Code (Pres Decree No. 442), to conduct a sympathy strike on the sole basis of affiliation.
employees were classified into managerial and rank-and-file employees.
Neither the category of supervisors nor their right to organize under the The Court construes Article 245 to mean that, as in Section 3 of the
old statute were recognized. So that, in Bulletin Publishing Corporation v. Industrial Peace Act, supervisors shall not be given an occasion to bargain
Sanchez (144 SCRA 628 [1986]), the Court interpreted the superseding together with the rank-and-file against the interests of the employer
labor law to have removed from supervisors the right to unionize among regarding terms and conditions of work.
themselves. The Court ruled:
The Court emphasizes that the limitation is not confined to a case of
In the light of the factual background of this case, We are constrained to supervisors wanting to join a rank-and-file local union. The prohibition
hold that the supervisory employees of petitioner firm may not, under the extends to a supervisors' local union applying for membership in a
law, form a supervisors union, separate and distinct from the existing national federation the members of which include local unions of rank-
bargaining unit (BEU), composed of the rank-and-file employees of the and-file employees. The intent of the law is clear especially where, as in
Bulletin Publishing Corporation. It is evident that most of the private the case at bar, the supervisors will be co-mingling with those employees
respondents are considered managerial employees. Also, it is distinctly whom they directly supervise in their own bargaining unit.
stated in Section 11, Rule II, of the Omnibus Rules Implementing the
There is no question about this intendment of the law. There is, however,
Labor Code, that supervisory unions are presently no longer recognized
in the present case, no violation of such a guarantee to the employee.
nor allowed to exist and operate as such. (pp. 633, 634)
Supervisors are not prohibited from forming their own union. What the
In Section 11, Rule II, Book V of the Omnibus Rules implementing Pres. law prohibits is their membership in a labor organization of rank-and-file
Decree No. 442, the supervisory unions existing since the effectivity of the employees (Art. 245, Labor Code) or their joining a national federation of
New Code in January 1, 1975 ceased to operate as such and the members rank-and-file employees that includes the very local union which they are
not allowed to directly join.
who did not qualify as managerial employees under this definition in
NOTE: Before this case was resolved, ALSI caved in to the pressure and their check-off authorization. Thereafter, ALUMETAL wrote respondent
was no longer interested to pursue this case. SC just said the employer is companies advising them to continue deducting union dues and remitting
free to grant whatever concession it wishes to give to its employees them to said federation.
unilaterally or through negotiations. However, the resolutions issued by
DOLE were still struck down.

WHEREFORE, the petition is hereby GRANTED. The private respondent is Respondent companies, then sought the legal opinion of the BLR. Med-
disqualified from affiliating with a national federation of labor Arbiter issued a Resolution finding the disaffiliation legal and gave its
organizations which includes the petitioner's rank-and-file employees. opinion that VLU’s members should continue paying their dues to
ALUMETAL in the concept of agency fees. However, on appeal to the
BLR’s Director, said resolution was reversed. VLU appealed to the
Secretary of Labor, however treating the same as MR, referred the same
Adamson v. CIR back to BLR. BLR denied the appeal.

(UNION AFFILIATION: LOCAL AND PARENT RELATIONS: Supervisor -


Rank and File Union Affiliation: Rule on Affiliation)
Secretary of Labor issued an execution of the BLR’s Director’s Order
stating that the same had become final and executory and ordering the
respondent companies to turn over the dues to ALUMETAL in accordance
Facts: with the check-off provision in the CBA.

Court of Industrial Relations (CIR) holding that the Adamson and ISSUES:
Adamson Inc. Supervisory Union (FFW) can legally represent supervisors
of the petitioner corporation... notwithstanding the affiliation of the rank
and file union of the same company with the same labor federation, the
Federation of Free Workers. 1. Whether VLU’s disaffiliation from ALUMETAL valid.

2. Whether respondent companies have the right to effect the


union dues collections despite revocation by the employees of the check-
Issues: off authorization.

3. Whether ALUMETAL is entitled to union dues payment


notwithstanding their disaffiliation.
COURT OF INDUSTRIAL RELATIONS ERRED IN SUSTAINING THE
ELIGIBILITY OF THE RESPONDENT UNION TO REPRESENT THE
PETITIONER'S SUPERVISORY EMPLOYEES NOTWITHATANDING THE
AFFILIATION OF THE SAID UNION WITH THE SAME NATIONAL RULING:
FEDERATION... whether or not a supervisor's union may affiliate with a
federation with which unions of rank-and-file employees of the same
employer are also affiliated.
The Resolutions of the BLR are REVERSED and SET ASIDE.

Ruling:
1. YES.

The supervisory employees of an employer cannot... join any labor


organization of employees under their supervision but may validly form a A local union, being a separate and voluntary association, is free to serve
separate organization of their own the interest of all its members including the freedom to disaffiliate when
circumstances warrant. This right is consistent with the Constitutional
The Adamson and Adamson Supervisory Union and the Adamson and guarantee of freedom of association. A disaffiliation does not disturb the
Adamson, Inc., Salesmen Association (FFW), have their own respective enforceability and administration of a collective agreement; it does not
constitutions and by-laws occasion a change of administrators of the contract nor even an
amendment of the provisions thereof." But nowhere in the record does it
There could be no employer influence on rank-and-file organizational appear that the contract entered into by the petitioner and ALUMETAL
activities nor there could be any rank and file influence on the prohibits the withdrawal of the former from the latter.
supervisory functions of the supervisors because of the representation
sought to be proscribed.
2. NO.

Volkschel Labor Union v. BLR,

(UNION AFFILIATION: LOCAL AND PARENT RELATIONS: Local Union The obligation of an employee to pay union dues is coterminous with his
Disaffiliation: Nature of Right) affiliation or membership. The employees' check-off authorization, even if
declared irrevocable, is good only as long as they remain members of the
union concerned. A contract between an employer and the parent
organization as bargaining agent for the employees is terminated by the
FACTS: disaffiliation of the local of which the employees are members.
Respondent companies therefore were wrong in continuing the check-off
in favor of respondent federation since they were duly notified of the
disaffiliation and of petitioner's members having already rescinded their
Volkschel Labor Union (VLU) was once affiliated with Associated Labor check-off authorization.
Union for Metal Workers (ALUMETAL), jointly entered into a collective
bargaining agreement (one of the subjects dealt with payment of union
dues) with respondent companies. However, VLU decided to disaffiliate,
and a resolution was adopted and signed by VLU’s members revoking 3. NO.
work shall be entitled to reinstatement without loss of seniority rights
and other privileges and to his full backwages, inclusive of allowances,
ALUMETAL is not entitled to union dues payments from petitioner's and to his other benefits or their monetary equivalent computed from the
members. A local union which has validly withdrawn from its affiliation time his compensation was withheld from him up to the time of his actual
with the parent association and which continues to represent the reinstatement. And as implemented by Section 3, Rule 8 of the 1990 New
employees of an employer is entitled to the check-off dues under a Rules of Procedure of the National Labor Relations Commission, it would
collective bargaining contract. seem that the Mercury Drug Rule (Mercury Drug Co., Inc. vs. Court of
Industrial Relations, 56 SCRA 694 [1974]) which limited the award of
back wages of illegally dismissed workers to three (3) years "without
deduction or qualification" to obviate the need for further proceedings in
Thus, ALUMETAL was ordered to return to VLU all the union dues
the course of execution, is no longer applicable.
enforced and collected through the execution of BLR’s Director’s Order.
As to the propriety of the dismissal, the court said that while it is true that
the CBA between OFC and the SAMAHAN provided for the dismissal of
Alex FERRER VS NLRC employees who have not maintained their membership in the union, the
manner in which the dismissal was enforced left much to be desired in
(UNION AFFILIATION: LOCAL AND PARENT RELATIONS: Rules on terms of respect for the right of petitioners to procedural due process.
Legality of Act of Disaffiliation) SAMAHAN did not conduct any investigation and hearing where
petitioners could have defended themselves. Moreover, the company
summarily dismissed petitioners upon request of the union officers
without conducting their own investigation.
Facts:

Petitioners were regular and permanent employees of the Occidental


Foundry Corporation (OFC). They had been in the employ of OFC for LIBERTY COTTON MILLS WORKERS UNION, RAFAEL NEPOMUCENO,
about ten (10) years at the time of their dismissal in 1989 as piece MARIANO CASTILLO, NELLY ACEVEDO, RIZALINO CASTILLO and
workers. The Samahang Manggagawa ng Occidental Foundry RAFAEL COMBALICER, petitioners,
Corporation-Federation of Free Workers (SAMAHAN) and the OFC
entered into a collective bargaining agreement (CBA). The agreement vs.
provides that a union member who fails to retain a membership of good
standing may be dismissed by the employer upon written request by the LIBERTY COTTON MILLS, INC., PHILIPPINE ASSOCIATION OF FREE
union. Pursuant to this provision, herein petitioners were dismissed from LABOR UNION (PAFLU) and the COURT OF INDUSTRIAL RELATIONS,
employment on the ground of failure to retain membership in good respondents.
standing. It was later on found out that the dismissal was due to an intra-
(UNION AFFILIATION: LOCAL AND PARENT RELATIONS: Rules on
union squabble arising out of the attempt by the petitioners to oust the
Legality of Act of Disaffiliation)
elected union officials. Upon knowledge of their dismissal, petitioners
volunteered to be admitted as members of the Federation of Democratic
Labor Unions (FEDLU) who represented them before the DOLE in the
complaint for illegal dismissal against the company, SAMAHAN and FFW.

Issue: Facts: Liberty Cotton Mills Workers Union, hereinafter referred to as the
Union, adopted its Constitution and By-laws on January 1, 1959.1 Among
Whether or not petitioners failed to maintain membership in good other things, the said Constitution provided:
standing by committing acts of disloyalty against SAMAHAN
ARTICLE I — NAME AND DOMICILE.
Held:
xxx xxx xxx
No. Petitioners sought the help of the FEDLU only after they had learned
of the termination of their employment. Their alleged application with ARTICLE X — UNION AFFILIATION
federations other than the FFW can hardly be considered as disloyalty to
the SAMAHAN, nor may the filing of such applications denote that Section 1. The Liberty Cotton Mills Workers Union-Paflu shall be affiliated
petitioners failed to maintain in good standing their membership in the with the Philippine Association of Free Labor Unions, otherwise known as
SAMAHAN. The SAMAHAN is a different entity from FFW, the federation PAFLU, and shall remain an affiliate as long as ten or more of its members
to which it belonged. Neither may it be inferred that petitioners sought evidence their desire to continue the said local union's affiliation, in
disaffiliation from the FFW for petitioners had not formed a union accordance with the Paflu Constitution, Article XI-Paragraph 11:15
distinct from that of the SAMAHAN. Parenthetically, the right of a local thereof;
union to disaffiliate from a federation in the absence of any provision in
the federation's constitution preventing disaffiliation of a local union is ARTICLE XIII — CHARGES, TRIALS, AND IMPEACHMENT OF OFFICERS
legal. Such right is consistent with the constitutional guarantee of
freedom of association. AND MEMBERS: APPEALS.

Hence, while petitioners' act of holding a special election to oust Capitle, Section 1. Any member or officer of the Liberty Cotton Mills Workers
et al. may be considered as an act of sowing disunity among the Union-Paflu may be charged, tried or impeached if an officer, in
SAMAHAN members, and, perhaps, disloyalty to the union officials, which accordance with this and the PAFLU CONSTITUTION.
could have been dealt with by the union as a disciplinary matter, it
On October 1, 1959, a Collective Bargaining Agreement was entered into
certainly cannot be considered as constituting disloyalty to the union.
by and between the Company and the Union represented by PAFLU.
Faced with a SAMAHAN leadership which they had tried to remove as
officials, it was but a natural act of self-preservation that petitioners fled Collective Bargaining Agreement was amended on February 28, 1964,
to the arms of the FEDLU after the union and the OFC had tried to thus:
terminate their employment. Petitioners should not be made accountable
for such an act. Article III. UNION SECURITY
With the passage of Republic Act No. 6715 which took effect on March 21, Additional Clause
1989, Article 279 of the Labor Code was amended to read as follows:
The Company agrees to encourage casual workers and non-union
Security of Tenure. — In cases of regular employment, the employer shall members to join the Union which is the sole and exclusive agent for all
not terminate the services of an employee except for a just cause or when the employees covered by this Agreement.
authorized by this Title. An employee who is unjustly dismissed from
Article XI. DURATION The disaffiliation, coming as it did from the greater majority of its
members, is more than enough to the collective desire of the members of
The Duration of this Agreement shall be for two (2) years, that is from the Liberty Cotton Mills Workers Union to sever their relations from the
November 2, 1963 up to November, 1965. mother federation. The right of disaffiliation is inherent in the compact
and such act should not have been branded as an act of disloyalty,
The Agreements aforementioned bore the signatures of representatives especially considering the cause which impelled the union to take such a
of both the Company and the PAFLU, and the incumbent President of the step.
local union.
Lastly, we will take up the process by which the workers were dismissed.
While the Collective Bargaining Agreement was in full force, Marciano We find that it was hastily and summarily done.
Castillo and Rafael Nepomuceno, President and Vice-President,
respectively, of the local union, wrote PAFLU, its mother federation,
complaining about the legal counsel assigned by the PAFLU to assist them
in a ULP case (Case No. 4001) they filed against the Company. In said Philippine Skylanders vs NLRC
letter, the local union expressed its dissatisfaction and loss of confidence
in the PAFLU lawyers, claiming that PAFLU never lifted a finger regarding (UNION AFFILIATION: LOCAL AND PARENT RELATIONS: Rules on
this particular complaint. Legality of Act of Disaffiliation)

On May 17, 1964, thirty two (32) out of the 36 members of the local union Facts:
disaffiliated themselves from respondent PAFLU pursuant to their local
union's Constitution and By-Laws, specifically Article X thereof, supra (p. In November 1993 the Philippine Skylanders Employees Association
12 Record). A copy of the signed resolution of disaffiliation was furnished (PSEA), a local labor union affiliated with the Philippine Association of
the Company as well as the Bureau of Labor Relations. The following day, Free Labor Unions (PAFLU), won in the certification election conducted
the local union wrote the Company and required the turn-over of the among the rank and file employees of Philippine Skylanders, Inc. (PSI). Its
checked-off dues directly to its Treasurer. rival union, Philippine Skylanders Employees Association-WATU (PSEA-
WATU) immediately protested the result of the election before the
PAFLU then wrote a letter to the company stating that they received the Secretary of Labor.
letter signed by 32 persons and informing them of their desire to
disaffiliate the local union from the mother federation — PAFLU. And also Several months later, PSEA sent PAFLU a notice of disaffiliation.
stated, that the members and officers who made the letter have no right
PSEA subsequently affiliated itself with the National Congress of Workers
to do the same under our existing contract and under the PAFLUs
(NCW), changed its name to Philippine Skylanders Employees Association
Constitution and By-Laws. PAFLU wrote the Company again, this time
– National Congress of Workers (PSEA-NCW), and to maintain continuity
quoting en toto Article III of the Collective Bargaining Agreement on
within the organization, allowed the former officers of PSEA-PAFLU to
"Union Security" and requesting the termination of the employment of
continue occupying their positions as elected officers in the newly-
Rafael Nepomuceno, Marciano Castillo, Nelly Acevedo, Enrique Managan,
formed PSEA-NCW.
Rizalino Castillo and Rafael Combalicer, all petitioners herein. PAFLU at
the same time expelled the aforementioned workers from their' union
membership in the mother federation for allegedly "instigating union
disaffiliation." On 17 March 1994 PSEA-NCW entered into a collective bargaining
agreement with PSI which was immediately registered with the
The Company terminated the employment of the members expelled by Department of Labor and Employment.
the PAFLU

Issue: whether or not the dismissal of the complaining employees,


petitioners herein, was justified or not. Meanwhile, apparently oblivious to PSEA’s shift of allegiance, PAFLU
Secretary General Serafin Ayroso wrote Mariles C. Romulo requesting a
Held: NO. The resolution of this question hinges on a precise and careful copy of PSI’s audited financial statement. On 30 July 1994 PSI through its
analysis of the Collective Bargaining Agreements. (Exhs. "H' and "I") In personnel manager Francisco Dakila denied the request citing as reason
these contracts it appears that PAFLU has been recognized as the sole PSEA’s disaffiliation from PAFLU and its subsequent affiliation with NCW.
bargaining agent for all the employees of the Company other than its
supervisors and security guards. Moreover it likewise appears that
"PAFLU, represented in this Act by its National Treasurer, and duly
authorized representative, ... (was) acting for and in behalf of its affiliate, Issue: WON PSEA’s disaffiliation is legitimate.
the Liberty Cotton Mills Workers Union and the employees of the
Company, etc.' In other words, the PAFLU, acting for and in behalf of its
affiliate, had the status of an agent while the local union remained the
basic unit of the association free to serve the common interest of all its Held:
members including the freedom to disaffiliate when the circumstances
warrant. This is clearly provided in its Constitution and By-Laws, At the outset, let it be noted that the issue of disaffiliation is an inter-
specifically Article X on Union Affiliation. union conflict the jurisdiction of which properly lies with the Bureau of
Labor Relations (BLR) and not with the Labor Arbiter.
For while it is correct to say that a union security clause did exist, this
clause was limited by the provision in the Unions' Constitution and By-
Laws, which states:
We upheld the right of local unions to separate from their mother
That the Liberty Cotton Mills Workers Union-PAFLU shall be affiliated federation on the ground that as separate and voluntary associations,
with the PAFLU, and shall remain an affiliate as long as ten (10) or more local unions do not owe their creation and existence to the national
of its members evidence their desire to continue the said local unions federation to which they are affiliated but, instead, to the will of their
affiliation. members. Yet the local unions remain the basic units of association, free
to serve their own interests subject to the restraints imposed by the
Record shows that only four (4) out of its members remained for 32 out constitution and by-laws of the national federation, and free also to
of the 36 members of the Union signed the resolution of disaffiliation on renounce the affiliation upon the terms laid down in the agreement which
May 17, 1964, triggered by the alleged negligence of PAFLU in attending brought such affiliation into existence.
to the needs of its local union, particularly its failure to assign a
conscientious lawyer to the local to attend to the ULP case they filed
against the Company. The disaffiliation was, therefore, valid under the
There is nothing shown in the records nor is it claimed by PAFLU that the
local's Constitution and By-Laws which, taken together with the
local union was expressly forbidden to disaffiliate from the federation nor
Collective Bargaining Agreement, is controlling.
were there any conditions imposed for a valid breakaway. As such, the
pendency of an election protest involving both the mother federation and
the local union did not constitute a bar to a valid disaffiliation. 2) directing respondent Manila Bay Spinning Mills, Inc. and J.P.
Coats to stop remitting to ANGLO federation dues and instead to remit
the whole amount of union dues to the treasurer of petitioner union; and

It was entirely reasonable then for PSI to enter into a collective


bargaining agreement with PSEA-NCW. As PSEA had validly severed itself
from PAFLU, there would be no restrictions which could validly hinder it 3) enjoining ANGLO-KMU from interfering in the affairs of
from subsequently affiliating with NCW and entering into a collective petitioner union.
bargaining agreement in behalf of its members.

SO ORDERED.1
Policy considerations dictate that in weighing the claims of a local union
as against those of a national federation, those of the former must be
preferred. Parenthetically though, the desires of the mother federation to
ANGLO filed a motion for reconsideration but the same was denied for
protect its locals are not altogether to be shunned. It will however be to
lack of merit. Hence, this petition for certiorari under Rule 65.
err greatly against the Constitution if the desires of the federation would
be favored over those of its members. That, at any rate, is the policy of the
law. For if it were otherwise, instead of protection, there would be
disregard and neglect of the lowly workingmen. The petition calls upon us to resolve two issues, to wit:

Alliance of Nationalist Union vs Samana Bay 1) whether the disaffiliation was valid; and
(Period)

2) whether petitioner can validly oust individual private


respondents from their positions.

Petitioner Alliance of Nationalist and Genuine Labor Organization


(ANGLO for brevity) is a duly registered labor organization while We rule for the respondents.
respondent union Samahan Ng Mga Manggagawang Nagkakaisa sa Manila
Bay Spinning Mills and J.P. Coats (SAMANA BAY for brevity) is its affiliate.
In representation of SAMANA BAY, ANGLO entered and concluded a
Collective Bargaining Agreement (CBA) with Manila Bay Spinning Mills For clarity, we shall first consider the issue respecting the validity of the
and J.P. Coats Manila Bay, Inc. (hereinafter referred to as the disaffiliation.
corporations) on November 1, 1991. On December 4, 1993, the Executive
Committee of SAMANA BAY decided to disaffiliate from ANGLO in view of
the latter's dereliction of its duty to promote and advance the welfare of
SAMANA BAY and the alleged cases of corruption involving the federation Petitioner ANGLO wants to impress on us that the disaffiliation was
officers. Said disaffiliation was unanimously confirmed by the members invalid for two reasons, namely: that the procedural requirements for a
of SAMANA BAY. valid disaffiliation were not followed; and that it was made in violation of
P.D 1391.

On April 4, 1994, a petition to stop remittance of federation dues to


ANGLO was filed by SAMANA BAY with the Bureau of Labor Relations on Anent the first ground, we reiterate the rule that all employees enjoy the
the ground that the corporations, despite having been furnished copies of right to self-organization and to form and join labor organizations of their
the union resolution relating to said disaffiliation, refused to honor the own choosing for the purpose of collective bargaining. This is a
same. ANGLO counter-acted by unseating all officers and board members fundamental right of labor and derives its existence from the
of SAMANA BAY and appointing, in their stead, a new set of officers who Constitution. In interpreting the protection to labor and social justice
were duly recognized by the corporations. provisions of the Constitution and the labor laws, rules or regulations, we
have always adopted the liberal approach which favors the exercise of
labor rights. 2

In its position paper, ANGLO contended that the disaffiliation was void
considering that a collective bargaining agreement is still existing and the
freedom period has not yet set in. The Med-Arbiter resolved that the This Court is not ready to bend this principle to yield to a mere
disaffiliation was void but upheld the illegality of the ouster of the officers procedural defect, to wit: failure to observe certain procedural
of SAMANA BAY. Both parties filed their respective appeals with the requirements for a valid disaffiliation. Non-compliance with the
Department of Labor and Employment. In a resolution dated September procedure on disaffiliation, being premised on purely technical grounds
23, 1994, herein public respondent modified the order and ruled in favor cannot rise above the fundamental right of self-organization. 3
of respondent union, disposing as follows:

We quote, with approval, the findings of herein public respondent, that:


WHEREFORE, the appeal of respondent ANGLO is hereby denied for lack
of merit while the appeal of petitioners is hereby granted. Accordingly,
the order of the Med-Arbiter is modified by: . . . the resolution of the general membership ratifying the disaffiliation
action initiated by the Board, substantially satisfies the procedural
requirements for disaffiliation. No doubt was raised on the support of the
1) declaring the disaffiliation of petitioner union from respondent majority of the union members on the decision to
ANGLO as valid;
disaffiliate. 4
This, to our mind, is clearly supported by the evidence. ANGLO's alleged
acts inimical to the interests of respondent union have not been
sufficiently rebutted. It is clear under the facts that respondent union's
members have unanimously decided to disaffiliate from the mother
federation and ANGLO has nothing to offer in dispute other than the law
prohibiting the disaffiliation outside the freedom period.

In the same wise, We find no ground for ruling against the validity of the
disaffiliation in the light of recent jurisprudential rules.

Although P.D. 1391 provides:

Item No. 6. No petition for certification election, for intervention and


disaffiliation shall be entertained or given due course except within the
60-day freedom period immediately preceeding the expiration of a
collective bargaining agreement,

said law is definitely not without exceptions. Settled is the rule that a local
union has the right to disaffiliate from its mother union when
circumstances warrant. 5 Generally, a labor union may disaffiliate from
the mother union to form a local or independent union only during the
60-day freedom period immediately preceding the expiration of the CBA.
However, even before the onset of the freedom period, disaffiliation may
be carried out when there is a shift of allegiance on the part of the
majority of the members of the union. 6

Coming now to the second issue, ANGLO contends that individual private
respondents were validly ousted as they have ceased to be officers of the
incumbent union (ANGLO-KMU) at the time of disaffiliation. In order to
fill the vacuum, it was deemed proper to appoint the individual
replacements so as not to put in disarray the organizational structure and
to prevent chaos and confusion among the general membership and
within the company.

The contention is bereft of merit. A local labor union is a separate and


distinct unit primarily designed to secure and maintain an equality of
bargaining power between the employer and their employee-members. A
local union does not owe its existence to the federation with which it is
affiliated. It is a separate and distinct voluntary association owing its
creation to the will of its members. 7 The mere act of affiliation does not
divest the local union of its own personality, neither does it give the
mother federation the license to act independently of the local union. It
only gives rise to a contract of agency 8 where the former acts in
representation of the latter.

By SAMANA BAY's disaffiliation from ANGLO, the vinculum that


previously bound the two entities was completely severed. ANGLO was
divested of any and all power to act in representation of SAMANA BAY.
Thus, any act performed by ANGLO affecting the interests and affairs of
SAMANA BAY, including the ouster of herein individual private
respondent, is rendered without force and effect.

WHEREFORE, premises considered, the petition is hereby DISMISSED.

Das könnte Ihnen auch gefallen